Anda di halaman 1dari 227

NGUYN TH CHI (CH BIN)

V THU AN CHU THU LAN - TRN THU NGA

BI TP TRC NGHIM

TING ANH
(DNG CHO HC SINH LP 12
V N LUYN THI VO I HC CAO NG)

NH XUT BN GIO DC

ENGLISH OBJECTIVE TEST 1


Time allowed: 90 Min
Part A. PHONETICS
I. Find a word in each line which is pronounced differently in the part underlined
1. A. stage

B. village

C. manage

D. baggage

1. A. supply

B. superb

C. support

D. suppose

2. A. eight

B. weight

C. height

D. freight

1. A. game

B. gist

C. gift

D. forgive

3. A. phase

B. danger

C. bookcase

D. literature

II. Find the word which has different stress pattern from the other three.
A. sincerity

B. remarkable

C. curriculum

D. combination

4. A. demanding

B. concerning

C. finishing

D. beginning

2. A. parliament

B. position

C. successful

D. abolish

3. A. referee

B. voluntary

C. paradise

D. parachute

4. A. appearance

B. herbivore

C. visible

D. magazine

1.

Part B. LEXICO - GRAMMAR


I. Choose the correct preposition or adverb for each sentence.
1. When you have thought .. what I have said, you will understand.

A. of

B. over

C. about

D. again

C. between

D. up

1. To come .. means to meet by chance.

A. into

B. across

2. I could tell he was pleased .. the expression on his face.

A. by

B. for

C. at

D. in

3. It isn't quite certain that he will turn .. at the meeting.

A. up

B. back

C. down

D. on

4. One of the primary causes of skin cancer is overexposure .. the sun without the use of

sunscreen.
A. in

B. under

C. with

D. to

II. Choose the correct answer for each of the following sentences.
16. You'll get a better . of exchange at another bank.
A. rate

B. value

C. worth

D. charge

17. Science has a language that is . in every country.


A. alike

B. similar

C. same

D. the same

18. I lost too much money betting at the races last time, so you won't. me to go again.
A. convince

B. impress

C. persuade

D. urge

19. Last year the potato harvest was very disappointing, but this year it looks as though we
shall have a better.
A. product

B. outcome

C. amount

D. crop

20. The shop assistant was . helpful, but she felt he could have given her more advice.
A. entirely

B. exactly

C. quite

D. totally

21. It's an awful. Jane couldn't come. I was looking forward to meeting her.
A. harm

B. sorrow

C. shame

D. shock

22. The new manager explained to the staff that she hoped to . new procedures to save
time and money.
A. restore

B. control

C. establish

D. manufacture

C. imaginable

D. imagination

23. I like his essay because it is very .


A. imaginary

B. imaginative

24. Everyone knows about pollution problems, but not many people have . any solutions.
A. looked into

B. thought over

C. got round to D. come up with

25. I don't know what was Tong with her this morning. She is usually quite
A. glad

B. pleased

C. cheerful

D. satisfied

26. The air in the house felt cold and . after weeks of bad weather.
A. wet

B. moist

C. damp

D. watery

27. He has a very. temper and often says things he regrets later.
A. angry

B. quick

C. warm

D. bad

28. Don't forget your appointment. You'd better put it in your.


A. agenda

B. diary

C. calendar

D. directory

29. You know I'll always stand. you if you are in trouble.

A. by

B. with

C. for

D. up

30. The taxi drew.at the gate promptly at six o'clock.

A. up

B. along

C. outside

D. over

III. Decide the underlined word or phrase (marked A, B, C, or D) in each sentence that
needs correcting.
29. Candles A have been use B since the C prehistoric era. D
30. Noise pollution in big, cities can be controlled in the number of ways.
31. Sweetly smelling A perfumes B are added C to soap to make it appealing. D
31. Many people believe that food additive A and other chemicals B used in the production C

and processing D of foods harm the body.


32. Many elephants they die A after they B lose their teeth C and can no longer D chew food.
33. Calculus was invented A to deal with B problems that could not be solving C using

algebra or D geometry.
34. Davis devoted his last year A to write B at his home C in Biloxi, near the D Gulf of

Mexico.
35. Often the bottom of a fan A or skillet becomes black B when it is C placed among a fire.

D
36. Isabella danced A in her bare feet B and wore loose-fitting clothing garments C that

allowed her D freedom of movement.


37. Light rays A that enter the eve B must come to a point focus C for a clear vision to form.

D
IV. Choose the correct word form for each of the numbered gaps.
E-MAIL
There can't be many people who are (41) . of e-mail, even if they have never
actually sent one. Although there are some (42) . between e-mail and letters, there are
also many differences. The first is that e-mail is delivered (43) . so it can be a very
(44) . means of communication when speed is important. This speed means that email is more (45) . for communicating over large distances. Another difference is that
e-mail tends to be (46) . informal. People arc much more likely to. use language
which they would consider (47) . for a formal letter. Words spelled (48) . in
an e-mail are less likely to be checked than in a letter. One explanation for this is that an email seems to be less permanent than something (49) . on paper. We can be sure that
the future development of e-mail will have all kinds of (50) .

effects on the way

we communicate.
41. A. aware

B. unaware

C. awareness

D. unawareness

42. A. similar

B. similarity

C. similarities

D. dissimilar

43. A. instant

B. instance

C. instances

D. instantly

44. A. effect

B. effecting

C. effective

D. effected

45. A. practice

B. practiced

C. practical

D. impractical

46. A. related

B. relating

relative

D. relatively

47. A. suited

B. suitable

C. unsuitable

D. suitably

48. A. correct

B. incorrect

C. correctly

D. incorrectly

49. A. written

B. writing

C. to write

D. being written

50. A. expected

B. unexpected

C. expecting

D. unexpectedly

Part C. READING
I. Read the two passages, then choose the correct answer for each question.
Noah Webster's goal in life was to promote the adoption of an American language. He

wanted to free Americans from British English as they had freed themselves from the British
crown. To this end he published a series of three textbooks: a speller in 1783, a grammar in
1784, and a reader in 1785. Webster objected to the way certain words had been borrowed
from other languages, but had not been respelled. The result, he claimed, was a confusing
mixture of letters, many of which were not pronounced the way they looked, and others of
which were not pronounced at all. Webster urged Americans to simplify their spelling. For
example, he argued that "head" should be spelled "hed" and "bread" should be spelled "bred".
Most of Webster's suggestions did not catch on, but his textbooks sold millions of copies.
51. Which of the following is the best title for the passage?
A. Three books of Noah Webster
B. Noah Webster and the Adoption of an American Language
C. Simplification of Spelling
D. Noah Webster and the British Crown

52. According to Webster, Americans should .


A. avoid reading his three books
B. be ruled from England
C. simplify their spelling
D. not borrow words from other languages

53. In the last paragraph, the phrase "did not catch on" means .
A. were not appreciated

C. did not please anyone

B. did not become popular

D. were not intelligent

54. Webster's books were very innovative, specially when we consider that they were written
in the .
A. late eighteenth centuryC. mid-seventeenth century
B. early eighteenth century

D. late seventeenth century

55. The problems that Webster tried to solve did NOT include .
A. unreliable spelling

C. grammar errors

B. mispronounced words D. poor sales of his textbooks

All humans do not have the same type of blood. In different types of blood, certain
antibodies and antigens may or may not be present. There are different systems for classifying
blood, and one of the systems is the ABO system. In this system, a person's blood is classified
as either type A, type B, type AB, or type O. The purpose of this system is to describe which
types of blood are compatible. This means which types of blood can be taken from or given to
a person. There are three principles that govern which types of blood are compatible.
The first principle is that a particular blood type is always compatible with itself. This

means, for example, that a person with type A blood can receive type A blood and that a
person with type B can accept type B blood. It also means that a person with type AB blood
can receive type AB blood and that a person with type O blood can receive type O blood.
The second principle is that type O blood can be given to any of the other blood groups.
Type O can be given to a patient with types A, B, or AB blood. For this reason, type O is
called the universal donor.
The last of the principles is that patients with type AB blood can receive blood from
type A, B, or O. This means that patients with type AB blood can compatibly receive any
other type of blood.
56. The topic of this passage is
A. a particular type of blood
A. a system for classifying types of blood
B. examples of problems with blood
C. the principle of universal donor

57. It is stated in the passage that certain antibodies and antigens in the blood
A. are omitted from the ABO system

C. are the universal donors

B. have not been classified

D. may not be in all blood

58. Look at the word principles in paragraph 1. Principles are


A. leaders

B. effects

C. rules

D. trials

59. It can be inferred from the passage that type A blood can
A. accept types A or 0 blood

C. accept types A or AB blood

B. accept types 0 or AB blood

D. accept types A or B blood

60. Look at the word donor in paragraph 3. A donor is most likely someone who
A. receives

B. gives

C. shows

D. takes

Choose the correct option to put in each of the numbered gaps.


The history of the bicycle goes back more than 200 years. In 1791, Count de Sivrac
. (61) onlookers in a park in Paris as he showed off his two-wheeled invention, a
machine called the 'celerifere'. It was basically an . (62) version of a children's toy
which had been in . (63) many years. Sivrac's `celerifere' had a wooden frame, made in
. (64) of a horse, which was mounted on a wheel at either end. To ride you sat on a
small seat, just like a modern bicycle, and pushed . (65) against the .(66) with
your legs there were no pedals. It was impossible to steer a 'celerifere' and it had no brakes,
but despite these problems the invention very much.(67) to the fashionable young men
of Paris. Soon they were . (68) races up and down the streets. Minor . (69)
were common as riders attempted a final burst of (70) Controlling he machine was

difficult as the only way to change . (71) was to pull up the front of the "celerifere"
and.(72) it round while the front wheel was. (73) in the air.
"Celerifere" were not popular for long, however, as the . (74) of no springs, no
steering and rough roads made riding them very uncomfortable. Even so, the wooden
'celerifere' was the. (75) of the modern bicycle.
61. A. delighted

B. cheered

C. appreciated

D. overjoyed

62. A. increased

B. enormous

C. extended

D. enlarged

63. A. use

B. play

C. operation

D. service

64. A. resemblance

B. shape

C. body

D. appearance

65. A. fast

B. deeply

C. heavily

D. hard

66. A. surface

B. ground

C. earth

D. floor

67. A. attracted

B. appealed

C. took

D. called

68. A. going

B. getting

C. holding

D. making

69. A. wounds

B. trips

C. injuries

D. breaks

70. A. velocity

B. energy

C. pace

D. speed

71. A. direction

B. route

C. heading

D. way

72. A. roll

B. drive

C. turn

D. revolve

73. A. cycling

B. circling

C. winding

D. spinning

74. A. mixture

B. link

C. combination

D. union

75. A. origin

B. design

C. model

D. introduction

III. Choose the correct connectives for the gaps in the following passage.
Our weather is not beautiful all the time. Perhaps you can remember a day. (76)..
bad weather made you afraid. One kind of bad weather (77) .. scares many people is
called a thunderstorm.
This is (78) .. happens when there is a thunderstorm. (79) .. you see a sudden
flash of bright light. (80) .. a few seconds you hear a loud rumbling sound. This quick
flash is called lightning, (81) .. the loud sound is called thunder. Lightning is colorful;
(82) .., it can cause serious problems. Lightning is electricity (83) .. is moving very
rapidly. It may be moving between a cloud and the ground, between two parts of the same
cloud. The lightning heats the air around it. This hot air expands, (84) .. gets bigger, and
it causes the air to move in waves. The air waves pass you in a series, one after another. (85)
.. you may hear many rumbles and not just one sound.
76. A. when

B. which

C. that

D. what

77. A. when

B. who

C. that

D. what

78. A. when

B. whose

C. that

D. what

79. A. First

B. Firstly

C. To begin

D. At the beginning

80. A. Within

B. Without

C. With

D. Before

8 l. A. but

B. and

C. or

D. furthermore

82. A. furthermore

B. however

C. but

D. and

83. A. when

B. where

C. that

D. what

84. A. but

B. however

C. or

D. moreover

85. A. That's why

B. The result

C. So

D. So that

Part D. WRITING
I. Choose the sentence nearest in meaning to the one in italics.
86. I wish she was going to the party.
A. I hope she'll come to the party.
B. I'm sorry she isn't going to the party.
C. I'm sorry she didn't come to the party.
D. I'm pleased she might come to the party.

87. He asked her if she would meet him that evening.


A. He said to her: " Did you meet me that evening?"
A.If he asks her, she'll meet him this evening.
B. He said: "Would you meet me one evening if I asked you?"
C. He said to her: "Will you meet me this evening?"

88. It was not as far to the town as we thought.


A. As we thought, the town wasn't far away.
A. We thought the town was far, and it was.
B. The town was nearer than we thought.
B. We thought the town was nearer than it was.

89. Do you realize that I would have been given that job if you had not been silent?
A. As you did not talk I did not get the job.
A. Because you asked I did not get the job.
B. Although you asked I got the job.
B. Although you did not talk I got the job.

90. When the power Jailed, we went to a restaurant for dinner.


A. We had dinner at the restaurant because of a power cut.
A. The power failed just as we were going out for dinner.
B. We were having dinner at a restaurant when the power failed
C. There was going to be a power cut, so we went out for a dinner.

91. It was unfair that Paul failed the driving test.

A. Paul failed the driving test because he was unfair.


A. Paul didn't deserve to fail the driving test.
B. It was unfair of Paul to fail the driving test.
C. Paul was not fair in doing the driving test.

92. It is possible Hoa didn't hear her name being called.


A. It is possible for Hoa to hear them call her name.
B. Hoa was not able to hear her name being called.
C. Hoa might not have heard them call her name.
D. Hoa might not hear her name being called.

93. I regret not visiting Washington when I was in America.


A. 1 regret to say that 1 didn't like to visit Washington when I was in America.
B. I visited Washington when I was in America but now I regret it.
C. I don't regret visiting Washington when I was in America.
D. I now wish I had visited Washington when I was in America.

94. Could you connect me to the director's office, please?


A. Could you direct me to the director's office, please?
B. Could you put me through to the director's office, please?
C. Could you come with me to the director's office, please?
D. Could you allow me to talk to the director, please? I am afraid we have run out of

Guardian newspapers.
A. I am afraid we have run into a Guardian.
B. There are not any Guardian newspapers left, I am afraid.
C. There won't be any Guardian newspapers to be published, I am afraid.
D. I am afraid we can't run Guardian newspapers any more.

II. Decide the sentence A, B, C, or D that is different in meaning to the original sentence.
96. It was my refusal to obey the policeman that caused my arrest.
A. It was because I refused to obey the policeman that I was arrested.
B. My refusal to obey the policeman resulted in my arrest.
C. I didn't obey the policeman, so I was arrested.
D. I was arrested, so I refused to obey the policeman.

97. I am sure it was by mistake that he took your calculator.


A. He must have taken your calculator by mistake.
B. I am sure he didn't take your calculator deliberately.
C. It is certain he didn't take your calculator on purpose.
D. It is a mistake that he took your calculator.

98. For fear of waking up the family, Trung tiptoed upstairs.


A. Trung tiptoed upstairs because he didn't want to wake up the family.
A. Trung tiptoed upstairs so as not to wake up the family.
B. The family feared when Trung tiptoed upstairs.
B. Trung tiptoed upstairs lest the family should be woken up.

99. But for the principal's help, we would have got into serious trouble.
A. If the principal hadn't helped us, we would have got into serious trouble.
A. Without the principal's help, we would have been in serious trouble.
A. Thanks to the principal's assistance, we managed to avoid serious trouble.
B. We got into serious trouble, but the principal refused to help us.

100. It would he a good idea if you went and asked her yourself
A. You had .better go and ask her yourself.
B. I advise you to go and ask her yourself.
C. I suggest you go and ask her yourself.
D. You went and asked her yourself and that was good.

English objective test 2


Time allowed: 90 min.
Part A. PHONETICS
Choose the best answer among A, B, C or D for each of the following.
1. In three of the four words, the underlined part is pronounced the same; in the fourth word,
the underlined part is pronounced differently. Which one is it?
A. bone

B. prone

C. phone

D. gone

2. Which of the following words has the underlined part pronounced [v] as in dove?
A. fern

B. of

C. sniff

D. finger

3. Which of the following words contains the sound [au] as in house?


A. haunt

B. astound

C. fought

D. draught

4. Which of the following words contains the sound [el] as in base?


A. aisle

B. vocabulary

C. praise

D. surface

5. Which of the following words doesn't contain the sound [w] as in wing?
A. swear

B. witch

C. aware

D. sword

6. Which of the following words has the first syllable stressed ?


A. common

B. begin

C. require

D. around

7. Decide the word which has the last syllable stressed.


A. enemy

B. entertain

C. technician

D. assemble

8. Decide the word which has main stress on the second syllable.
A. surgery

B. vacancy

C. prevention

D. preservation

9. Find the word whose main stress doesn't fall on the second syllable.
A. economy

B. political

C. believable

D. architecture

10. Find the word whose main stress doesn't fall on the third syllable.
A. philosophy

B. engineering

C. adaptation

D. influential

Part B. LEXICO - GRAMMAR


1. Choose the verb or phrasal verb which best completes each sentence.
11. I've been trying to ring him all evening but I can't ..
A. get over

B. get through

C. get across

D. get by

12. The doctor.. him to smoke a few cigarettes each day.


A. let

B. made

C. stopped

D. allowed

13. Father would not .. us to go there for the weekend.


A. let

B. permit

C. agree

D. consent

14. Grandfather has never really.. from his nervous breakdown.


A. mended

B. cured

C. repaired

D. recovered

15. All traffic is being.. because of the military parades.


A. diverted

B. converted

C. changed

D. altered

16. Angela has-grown up but she hasn't .. her shyness yet.


A. got through

B. got under

C. got over

D. got away from

17. Your progress will be .. in six months' time.


A. counted

B. valued

C. admired

D. evaluated

18. The little girl .. over a stone and fell flat on her face.
A. tripped

B. dripped

C. stepped

D. walked

19. There was so much noise that we could hardly .. what the speaker was saying.
A. take in

B. take up

C. bring in

D. bring up

20. I am sorry to hear that Peter and Dick have.. They were such good friends.
A. fallen against

B. fallen out

C. dropped out

D. dropped against

II. Choose the word or phrase which best completes each sentence.
21. Paul was so.. with his examination results that he didn't smile all week.
A. disappointed

B. delighted

C. proud

D. satisfying

C. quietly

D. widely

22. She smiled.. at the joke.


A. loudly

B. strongly

23. Which would you.. have, the red or the blue one?
A. better

B. prefer

C. rather

D. choose

24. We're going on a day.. to the island tomorrow.


A. visit

B. voyage

C. journey

D. trip

25. The article was.. the front page of the newspaper.


A. on

B. at

C. in

D. of

26. I've always posted my letters first class, .. it costs more.


A. because

B. since

C. while

D. although

27. When 1 first started work, I found her advice..

A. priceless

B. invaluable

C. worthy

D. irreplaceable

28. In order to achieve the desired result in this experiment, it is necessary that he .. as

fast as he can.
A. work

B. works

C. worked

D. will work

29. I am sure we can believe her. She is usually very..

A. true

B. truthful

C. trustful

D. trustworthy

30. The number of.. in the factory has increased so much that the police have been asked

to investigate.
A. thieves

B. thefts

C. robbers

D. robberies

31. The smell of floor polish still brings.. memories of my old school.

A. up

B. on

C. back

D. over

32. Youths have to .. military service in our country.

A. do

B. make

C. join

D. attend

33. At the end of the winter, the price of winter clothes in the shops usually

A. lowers

B. drops

C. falls

D. reduces

C. Do

D. Tidy

C. remembers

D. recollects

34. Your room is a mess! .. it up at once.

A. Arrange.

B. Make

27. That song.. me of my childhood.

A. recalls

B. reminds

35. When you retire, you will receive a(n) .. from the government.

A. allowance

B. benefit

C. grant

D. pension

28. The proportion of elderly people in the population is steadily.. as they live longer.

A. growing up

B. rising

C. falling

D. raising

36. Early retirement is a.. of reducing the workforce while avoiding redundancy.

A. device

B. source

C. proposal

D. means

37. In the .. of proof, the police could not take action against the man.

A. shortage

B. lack

C. want

D. absence

38. My uncle took .. golf when he retired from work.

A. up

B. over

C. on

D. after 16

III. Choose the alternative which is CLOSEST in meaning to the italicized word
41. The performance thrilled the audience.

A. scene

B. sight

C. show

D. actor

B. displeased

C. naughty

D. serious

B. natural

C. valuable

D. expensive

42. They tried hard not to be discontented.

A. discouraged
43. It's imitation leather.

A. man-made

44. They grumbled about the food.

A. argued

B. thought

C. complained D. bargained

45. He was astounded at his bad luck.

A. unhappy

B. frightened

C. astonished

D. amused

C. opinion

D. proposal

C. reluctant

D. unhappy

41. They accepted his offer.

A. gift

B. invitation

42. The children were anxious to go camping.

A. afraid

B. eager

43. The farmer will harvest the rice soon.

A. plant

B. store

C. visit

D. gather

44. This shop sells small articles.

A. pictures

B. ornaments

C. goods

D. papers

45. They fixed the date for the wedding.

A. specified

B. fastened

C. approved of

D. discussed

Part C: READING
I. Choose the correct article for each numbered space.
A skeleton is (51)

framework of bones. If we had no (52). bones, we could not

stand up. There are over (53) . 200 bones in (54) . human body. Our (55) .
most important bone is the back bone. The bone in our head is called (56) . skull. The
ribs protect (57) . heart and lungs. The outside of (58) . bone is hard but (59) .
inside is softer. The inside is called the marrow. Young children have (60) . soft bones
which keep on growing. When we are about 20 years old, our bones stop growing.
51. A.

B. a

C. an

D. the

52. A.

B. a

C. an

D. the

53. A.

B. a

C. an

D. the

54. A.

B. a

C. an

D. the

55. A.

B. a

C. an

D. the

56. A.

B. a

C. an

D. the

57. A.

B. a

C. an

D. the

58. A.

B. a

C. an

D. the

59. A.

B. a

C. an

D. the

60. A.

B. a

C. an

D. the

II. Read the following text, then circle the most correct answer for each question.
The general principles of dynamics are rules that demonstrate a relationship between the
motions of bodies and the forces that produce those motions. Based in large part on the work
of his predecessors, Sir Isaac Newton deduced three laws of dynamics, which he published in
his famous Principia
Prior 10 Newton, Aristotle had established that the natural state of a body was a state of
rest, and that unless a force acted upon it to maintain motion. a moving body would come to
rest. Galileo had succeeded in correctly describing the behavior of falling objects and in
recording that no force was required to maintain a body in motion. He noted that the effect of
force was to change motion. Huygens recognized that a change in the direction of motion
involved acceleration, just as did a change in speed, and further, that the action of a force was
required. Kepler deduced the laws describing the' motion of planets around the sun. It was
primarily from Galileo and Kepler that Newton borrowed.
In short, Newton's laws of Motion are: (1) a body at rest remains at rest, a.-id a body in
motion remains in motion along a straight line, unless acted upon by an unbalanced force, (2)
if an unbalanced force acts upon a body, the momentum of the body changes in proportion to

the force and in the same direction as the force, (3) to every action or force, there is an equal
and opposite reaction.
61. What was the main purpose of this passage?
A. To demonstrate the development of Newton's laws
B. To establish Newton as the authority in the field of physics
C. To discredit Newton's laws of motion
D. To describe the motion of planets around the sun
62. The word 'predecessors' in line 3 refers to

A. those who came before

C. those who published their work

B. those who provided help

D. those who agreed with the ideas

63. The phrase 'prior to' in line 5 could best be replaced by which of the following?

A. before

B. after

C. with

D. simultaneously

64. Which of the following scientists established that the natural state of a body was a state of

rest?
A. Galileo

B. Kepler

C. Aristotle

D. Newton

C. state

D. motion

65. The word 'it' in line 6 refers to ....... .

A. rest

B. body

66. Who was the first scientist to correctly describe the behavior of falling objects?

A. Aristotle

B. Newton

C. Kepler

D. Galileo

67. According to Huygens, when was acceleration required?


A.

For either a change in direction or a change in speed

B.

Only for a change in speed

C.

Only for a change in direction

D.

Neither for a change in direction nor for a change in speed

68. According to this passage, Newton based his laws primarily upon the work of

A. Galileo and Copernicus

B. Ptolemy and Copernicus

C. Huygens and Kepler

D. Galileo and Kepler

69. The word 'momentum' in line 16 is closest in meaning to

A. weight

B. speed

C. shape

D. size

70. Which of the following describes Inertia, or the principles of bodies at rest?

A. Newton's first law

B. Newton's third law

C. Newton's law of motion

D. Newton's law of dynamics

III. Decide the best option (A, B, C, or D) to be used in each numbered blank.
ROBOTS

The popular idea of a robot is a machine that acts like and resembles a human being. But
the robots (71) are increasingly being used for a wide range of tasks do not look
human-like (72) all. The robots (73) work in car factory production lines
looks something like cranes. The mobile robots used (74) army bomb- disposal look
like wheel barrows on tracks. And children (75) likened a mobile robot used in school
to teach (76) computer programming to a giant sweet. Robot (77) however,
resemble human beings in the range of actions that they can carry out. Instead of repeatedly
(78) one action, like an automatic machine, a robot can perform a (79) of
different actions. It movements are (80) either by oil or by electric motors, and its
brain is a small computer that directs (81) movements. Inside the computer's memory
are the instructions for carrying (82) a task - picking chocolates from a container and
putting them in the right part of a display box, for (83) . By changing the
programme, the robot can be made to (84) the task, or do something different within
the limits of the activities it is designed (85) .
A. which

B. there

C. they

D. who

A. in

B. at

C. for

D. after

A. there

B. that

C. do

D. to

A. by

B. for

C. to

D. with

A. who

B. are

C. have

D. look

A. it

B. them

C. themselves

D. their

A. must

B. do

C. don't

D. might

A. just

B. not

C. every

D. single

A. chain

B. link

C. line

D. group

A. done

B. made

C. controlled

D. ordered

A. their

B. its

C. whose

D. which

A. out

B. off

C. away

D. with

A. sure

B. certain

C. example

D. pleasure

A. alter

B. vary

C. differ

D. change

A. from

B. of

C. for

D. by

Part D. WRITING
I. Choose the sentence nearest in meaning to the one given in italics.
86. Whoever did that must have been a very brave person.
A. A. Only a very brave person could do that.
B.

Only a very brave person could have done that.

C.

A very brave person never did that.

D. A very brave person knows who did that.

87. She doesn't realize how serious her husband's operation is going to be.
A.She realizes that her husband's operation is not going to be serious.
B. She does realize how serious her husband's operation is going to be.
C. She realizes seriously that her husband's going to have an operation.
D.Little does she realize how serious her husband's operation is going to be.

88. He decided to repair the thing himself and not to take it back to the shop.
A.Rather than take it back to the shop, he decided to repair the thing, himself.
B. He decided to repair the thing himself as he couldn't take it back to the shop.
C. He took the thing back to the shop because he wanted to have it repaired.
D.He decided to have the thing repaired and not to take it back to the shop.

89. "Please don't drive so fast!" Nicole said to her husband.


A.Nicole pleaded with her husband not to drive so fast.
B. Nicole pleased her husband by not driving so fast.
C. Nicole encouraged her husband to drive faster.
D.Nicole's husband didn't want to drive so fast as she suggested.

90. It is such a wonderful opportunity that we mustn't miss it.


A.It is too wonderful an opportunity for us to miss.
B. The opportunity is wonderful enough for us to miss.
C. It is a wonderful opportunity so that we can't miss it.
D.It is such a wonderful opportunity that we are not able to afford it.

91. You should for no reason accept their offer.


A.For no reason are you to accept their offer.
B. You should accept their offer in all circumstances.
C. You must refuse their offer for any reason.
D.You are to accept their offer for any reason.

92. The shops don't deliver now, which makes life difficult.
A.If shops don't deliver, life would be less difficult.
B. If shops delivered, life would be less difficult.
C. As shops delivered, life wouldn't be so difficult.
D. As shops don't deliver, life would be so difficult.

93. We could direct you since we had a map.


A. If we had a map, we could direct you.
B. If we didn't have a map, we couldn't direct you.
C. But for a map we couldn't direct you.

D. But for a map we couldn't have directed you.

94. My parents find fault with everything I do.


A. My parents find everything I do useful.
B. Whatever I do, my parents find it faulty.
C. Whatever I do, my parents find fault with it.
D. No matter what I do my parents tell me off for it.

95. Keeping calm is the secret of passing your driving test.


A. You should keep your passing of the driving test secret.
B. You have passed the driving test but you should keep calm.
A. As long as you keep calm, you can pass the driving test.
C. It is a secret you have passed the driving test.

II. Decide A, B, C, or D which shows the best way of combining the following pairs of
sentences.
96. It was cloudy last Sunday. The photos came out fine.
A. It was cloudy last Sunday though the photos came out fine.
A. It was cloudy last Sunday but the photos came out fine.
B. The photos came out fine whereas it was fine last Sunday.
C. The photos would have come out fine although it was cloudy last Sunday.

97. Women still cover their heads in some countries. They did so in the past.
A. Women still cover their heads in some countries as they did in the past.
A. Women still cover their heads in some countries as they did so in the past.
B. Women cover their heads in some countries similar to what they did so in the past.
B. In the past women covered their heads but they do so today in some countries.

98. She phoned him early in the morning. She wanted to make sure she caught him before he
le: ft ,for work.
A. She phoned him early in the morning so that she wanted to make sure she caught him

before he left for work.


B. She phoned him early in the morning lest she should catch him before he left for work.
C. She phoned him early in the morning because she wanted to make sure she caught him

before he left for work.


D. She phoned him early in the morning so as to want to catch him before he left for work.

99. The history of the United States is not long. It is interesting.


A. The history of the United States is not long however it is. interesting.
A. The history of the United States is not long despite interesting.
B. The history of the United States is not long but interesting.

B. The history of the United States is interesting whereas not long.

100. Tuition lees were increased. There were many objections from students and parents.
A. There were many objections from students and parents, so tuition fees were increased.
A. There were many objections from students and parents while tuition fees were increased.
B. Tuition fees were increased but there were many objections from students and parents.
B. Tuition fees were increased in spite of the fact that there were many objections from

students and parents.

English objective test 3


Time allowed: 90 min.
Part A. PHONETICS
I. Find the word (A, B, C, or D) which is pronounced differently in the pal underlined in
each line.
1. A. excursion

B. further

C. occur

D. occurrence

2. A. measure

B. division

C. precision

D. apprehension

3. A. replace

B. luggage

C. surface

D. palace

4. A. steward

B. sew

C. sewage

D. dew

5. A. fearsome

B. spear

C. gear

D. swear

II. Decide the word whose main stress falls on the second syllable.
6.

A. ingenious

B. principle

C. generous

D. volunteer

7.

A. mineral

B. nutritious

C. safeguarding

D. regulate

8.

A. decisive

B. parachute

C. aeroplane

D. mathematics

9.

A. reckon

B. protein

C. sanguine

D. technique

B. personality

C. potentially

D. territorial

10. A. inability

Part B: LEXICO - GRAMMAR


I. Circle the answer A, B, C, or D which best fits in the blank in each of the sentences.
11. For my birthday I was given a writing set. two pens, envelopes an notepapers.

A. comprising

B. involving

C. holding

D. consisting

12. One condition of this job is that you must be . to work at weekends.

A. available

B. capable

C. acceptable

D. accessible

11. I am going to. my dress dry- cleaned.

A. make

B. send

C. get

D. take

12. Don't . to bring a hairdryer. I'll lend you mine.

A. mind

B. bother

C. forget

D. worry

13. The building was badly . in the fire.

A. hurt

B. wounded

C. damaged

D. injured 24

14. He is unhappy because his parents do not . very well.

A. get over

B. get off

C. get on

D. get away .

15. . a wrong number, it is important to apologize before hanging up.

A. You dial

B. Dialing

C. If you dial

D. If it is dialed

C. give up

D. take in

16. Lots of people . yoga to relax.

A. practice

B. take up

17. Even though she hated the food, her mother . her eat it all.

A. let

B. made

C. forced

D. allowed

C. likely

D. liking

18. The twins look so . their father.

A. like

B. alike

19. It took him ages to . living in the new town.

A. use to

B. get used to

C. used to

D. accustomed

20. Maybe we should. Mr. Smith to see his rose garden.

A. call at

B. call on

C. call for

D. call out

21. Drunkenness is . for many road accidents.

A. guilty

B. responsible

C. cause

D. faulty

22. The police are said to be looking. the matter.

D. for

D. into

D. up

D. over

23. If you park there, you will have to pay a.

D. fee

B. ticket

C. fare

D. fine

II. Choose the right word to complete the following English idioms.
26. as hungry as a.
A. farmer

B. hunter

C. beggar

D. fighter

B. moon

C. picture

D. dream

B. snow

C. winter

D. water

B. rooster

C. winner

D. peacock

B. fat

C. a fish

D. an eel

B. flower

C. honey

D. a dream

B. brave

C. strong

D. courageous

B. dumb

C. quiet

D. firm

C. quiet

D. poor

C. quiet

D. poor

27. as pretty us a.
A. flower
28. as cold as.
A. ice
29. as proud as a.
A. cock
30. as slippery as.
A. oil
31. us sweet as.
A. sugar
32. as. as a lion
A. cruel
33. as. as a post
A. deaf

34. as. as a mouse


A. fast

B. wise

35. as. as a church mouse


A. fast

B. wise

III. Choose the correct verb form to be used in the numbered gaps.

There (36). an incredible evolution in the size and capabilities of


computers in the past years. Today, computers smaller than your fingernail (37) . the
same capabilities as room-sized machines of about 50 years ago. The first computers (38)
. around 1945. They were so large that they (39) . air-conditioned rooms. Then
in the 1960s, desk-sized computers were developed. This (40) . a gigantic advance.
Shortly afterward, a third generation of computers, which (41) . simple integrated
circuits and which (42) . even smaller and faster (43) . In the 1970s the first
microprocessor, less than one square centimeter in size was developed. Today electronic
engineers (44). that even smaller and more sophisticated computers (45). on the
market before the end of this decade.
36. A. be

B. is

C. was

D. has been

37. A. has

B. have

C. are having

D. had

38. A. developed

B. were developed

C. have been developed

D. have been developing

39. A. require

B. required

C. was requiring

D. have required

40. A. represents

B. was representing C. represented

D. has represented

41. A. used

B. was using

C. is using

D. had used

42. A. was

B. is

C. were

D. are

43. A. appeared

B. appearing

C. has appeared

D. had appeared

44. A. predict

B. predicted

C. are predicting

D. will predict

45. A. is

B. are

C. will be

D. will have been

IV. Choose the correct word form for each gap to complete the passage.
School inspectors have found that, contrary to all (46) . children don't dislike
homework at all. In fact, many do more than their teachers suggest, either because of (47)
. about their marks or (48) . for enjoyment. The inspectors' (49) . add to
the evidence of the powerful (50) . effects of homework on pupils' achievement. The
inspectors also recommend the of "homework chatter". These tell parents and children how
much work is expected and provide (53) . to schools to form links with parents who
can check that tasks are properly (54) . International (55) . suggest that a
typical 14-year-old does six hours homework in Britain, eight hours in Italy and almost nine
hours in Hungary, Japan and Poland.
46. A. expects

B. expectants

C. expectancy

D. expectations

47. A. anxious

B. anxiety

C. anxiousness

D. anxiously

48. A. simple

B. simply

C. simplify

D. simplification

49. A. finds

B. found

C. finding

D. findings

46. A. benefit

B. benefits

C. beneficial

D. benefitial

47. A. introduce

B. introduction

C. introducing

D. introductory

A. adding

B. added

C. additive

D. additional

53. A. courage

B. encouragement C. courageously

D. encouraging

54. A. complete

B. completed

C. completing

D. completive

A. compares

B. comparing

C. comparisons

D. comparasions

V. Decide the underlined word or phrase (marked A, B, C, or D) in each sentence that


needs correcting.
56. When too many firms enter competitive markets, their share of profits will fell.

57. The term "middle class" describes people between the upper and the low social classes.

58. Copper comes from seven types of ores that also contain the other materials.

59. Matthew Henson received many honor for his part in the expedition to the North Pole.
A

60. Silicon chips contain thousands of circuits in an area as smaller than fingernail.
A

61. The waters of the Black Sea conceal numerous archaeologist treasures.
A

62. Two human hairs found at the scene of the crime led the detectives to the eventual arrest
A

the murder.
D
59. There is no definite record of who created and first use traffic control devices.
A

64. Fortunetellers say they possess a certain power that makes them aware of events before
A

them happening.
D
65. Developing new technologies are time-consuming and expensive.
Part C. READING
I. Read the following text and choose the correct answer for each question.
The Empire State Building and its occupants suffered a serious trauma on Saturday
morning in 1945. The Empire State Building was, quite suddenly and without warning, struck

by an airplane.
That morning, an experienced army pilot named Bill Smith took off from Bedford,
Massachusetts in a B-25 bomber with two passengers on board He was headed for Newark,
New Jersey in very foggy condition. About an hour into the flight, the plane was in the middle
of New York City rather than at its intended destination. The pilot was able to maneuver the
plan around several skyscrapers before it crashed into the 79 th floor of one of the tall
buildings, the 102-floor Empire State Building.
When the plane hit the building, there was a huge amount of destruction There was a big
hole in the outside of the building, and parts of the plane crushed through interior walls on the
78th and 79th floors. Then, when the fuel tank exploded, six floors of the building went up in
flames.
There was also a human toll. Fourteen lives were lost in the crash; the number would
have been far greater, however, if the accident had occurred on a weekday rather than on a
Saturday. During working hours, there might have 25,000 people in the building.
66. The subject of this passage is
A. an unusual accident

C. a scheduled flight

B. a military attack

D. a visit to a famous building

67. When did this event occur?


A.In the middle of the 19th century.

C. In the middle of the last century.

B. In late 19th century.

D. In late 20th century.

68. Look at the word "struck" in paragraph 1. This word could be best replaced by .
A. flown

B. landed

C. constructed

D. hit

69. How many people, as implied in the passage, were in the plane?
A. 1

B.2

C.3

D. 4

70. It can be inferred from the passage that fire.


A.did not break out anywhere
B. only engulfed part of the building
C. spread throughout the whole building
D.burned down the complete building

Read the following text carefully, then circle either A, B, C or D to fit each space in the
text.
THE LONDON TO BRIGHTON CAR RUN
The first London to Brighton run took place on November 14 th 1896. It was organized to
celebrate the (71). of a law which made it easier for cars in Britain to be driven on the
roads. Before then, the law (72) .

a driver and an engineer in the car and a man

walking in front of the vehicle with a red flag (73) . of its approach.
Since then, this annual run has become one of the most popular events on the British
motoring calendar, (74) .crowds of over one million lining the route. Only the (75)
. oldest cars, constructed during the ten year between 1895 and 1905, are allowed to
(76) . in it. Lovingly polished by their drivers, who are dressed in the clothing of the
(77) . the cars leave. Hyde Park in London at 7.30 am and arrive, (78) ., in
Brighton some sorry three hours later.
The 60-mile run is not a race there's an official coffee stop on the (79) . and
the cars are restricted to an average speed of only 30 kph. The only (80) . for
finishing is a medal, which is awarded to everyone who (81) . Brighton before 4
pm. The run traditionally (82) . participants from all four (83) . of the
world, including Europe, Asia, Africa and Australia Since the youngest car is nearly a
hundred years old, some of them (84) .down of course. But for the owners of the
400-plus vehicles, it's sin being there that (85) .the greatest pleasure.
71. A. electing

B. passing

C. settling

D. appointing

72. A. forced

B. needed

C. obliged

D. required

73. A. announcing

B. warning

C. declaring

D. forecasting

74. A. with

B. having

C. including

D. along

75. A. quite

B. certainly

C. very

D. surely

76. A. involve

B. take part

C. get engaged

D. include

77. A. while

B. spell

C. phase

D. period

78. A. hopefully

B. intentionally

C. ambitiously

D. purposely

79. A. direction

B. way

C. path

D. process

80. A. earning

B. profit

C. reward

D. credit

81. A. gets

B. meets

C. attains

D. reaches

82. A. appeals

B. engages

C. attracts

D. catches

83. A. edges

B. tips

C. limits

D. corners

84. A. break

B. fall

C. run

D. pull

85. A. lets

B. fetches

C. brings

D. results

Part D. WRITING
1. Choose the phrase or clause A, B, C, or D that best completes each sentence
86. Various societies define. in many rather complex ways.
A. that is successful

B. what success is

C. that success is

D. what is success

87. Equipment failures or damage. can interrupt local service of electric power.

A. are caused by storm

B. they are caused by storm

C. which caused

D. caused by storm

88. Before electric. common, Europeans used candles as a source of artificial light.
A. lightning it becameB. the lightning became
C. becomes the lightning

D. lightning became

89. Art critics and historians alike claim that Van Gogh's art.from that of his
contemporaries.
A. is a considerable difference

B. the difference is considerable

C. is considerably different

D. was considerably and differently

90. Man-made satellites in space carry instruments.


A. that record where flooding is worst can
B. where is the worst flooding that can be recorded
C. where can they record the worst flooding
D. that can record where flooding is worst
91. This road, . is narrow and winding.
A. which the two villages are joined
B. which the two villages are joining
C. which joins the two villages
D. that joins the two villages
92. Most of the suggestions, ., were not very practical.
A. raised at the meeting
B. which were raised at the meeting
C. that were raised at the meeting
D. which they have raised at the meeting
93. It was not until recently .
A. did we realize how truthful he was
B. when we had realized that he was truthful
C. which we realized that he was truthful
D. that we realized how truthful he was
94. The dress didn't fit her, . where she had bought it.
A. so that she took it back to the shop
B. so she took it back to the shop
C. however, she took it back to the shop
D. since she took it back to the shop

95. He went with a group of people, .

A. few of which were correctly equipped for such a climb


B. few of whom were correctly equipped for such a climb
A. which few of them were correctly equipped for such a climb
C. few among them were correctly equipped for such a climb

II. Each of these sentences is followed by four suggested explanations but of one is
correct. Decide the correct explanation (A, B, C, or D).
96. Wait a moment! It's on the tip of my tongue.
A. Something is hurting my tongue.
A. I can't speak for a moment.
B. I shall remember it in a moment.
C. The tip of my tongue is sore.

97. Go and see what the children are up to.


A. I want to know if they need anything.
A. I want to know what mischievous things they are doing.
B. I want to know which tree they have climbed this time.
B. I'd like to see if they are upstairs.

98. My fingers are all thumbs.


A. I am extremely clumsy.
A. My fingers are all very thick.
B. I am trying to attract your attention.
B. My fingers are all very nice.

99. That will put the cat among the pigeons.


A. The cat will have to stay outside the house tonight.
A. Then we shall have as many cats as we have pigeons.
B. The cat will play with the pigeons.
B. That will cause a lot of trouble.

100. What's happened? You look as if you have been in the wars.
A. You look like an old soldier.
B. You are wearing a lot of medals.
C. You look as though something unpleasant has happened to you.
D. You look as though you have been fighting.

English objective test 4


Time allowed: 90 min.
Part A PHONETICS
I. Choose the word A, B, C, or D in each group that has the underlined part pronounced
differently from the rest.
1. A. bough

B. tough

C. rough

D. enough

2. A. drunkard

B. postcard

C. remark

D. discard

3. A. coward

B. tower

C. powerful

D. mow

4. A. alumni

B. identify

C. idiom

D. pi

5. A. chimpanzee

B. chaos

C. chip

D. fetch

II. Find the word with the stress on the first syllable in each line.
6. A. region

B. rely

C. relax

D. resource

7. A. behavior

B. broadcaster

C. beneficial

D. experiment

8. A. linguistics

B. logical

C. limitation

D. lemonade

9. A. experience

B. efficient

C. extensive

D. evidence

10. A. ancient

B. advance

C. account

D. appear

Part B. LEXICO - GRAMMAR


I. Circle the best option (A, B, C, or D) for each gap in the following sentences.
11. Having finished the report.
A. it was submitted to his boss by him
B. he submitted it to his boss
C. his boss was submitted it by him
D. he had submitted it to his boss

12. Oil if you pour it on water.


A. floated

B. has floated

C. will be floated

D. floats

13. Modern buildings should with the surrounding area.


A. blend

B. fit

C. match

D. join

14. The Government have an inquiry to investigate bribery in local elections.

A. set out

B. set to

C. set about

D. set up

15. The water workers' claim for a 10 percent pay rise has been under by the

Government.
A. application

B. consideration

C. inquiry

D. regard

14. Mr. James Smith retired early ill health.

A. on account of

B. on behalf of

C. in front of

D. ahead of

15. A man's pay usually from the number of hours he works in a week.

A. results

B. depends

C. starts

D. earns

16. Dentists will always try to save teeth rather than take them

A. off

B. over

C. out

D. down

C. mob

D. flock

17. What he told me was a of lies.

A. load

B. pack

18. We can only give you the number of refugees crossing the border at the moment.

A. suggestive

B. indefinite

C. approximate D. unclear

19. This ticket two persons to the show.

A. delivers

B. includes

C. admits

D. enters

20. Trains stop here to passengers only.

A. get off

B. pick up

C. get past

D. get on

21. It isn't quite that he will turn up at the meeting.

A. certain

B. exact

C. right

D. sure

22. Many difficulties have as a result of the changeover to the new system.

A. raised

B. been raised

C. arisen

D. experienced

23. The of the lake is covered with reeds and rushes.

A. beach

B. coast

C. shore

D. bank

24. I was not by his many arguments so finally we agreed to differ.

A. convinced

B. persuaded

C. concerned

D. assured

C. out with

D. through with

25. All the children have gone mumps.

A. along with

B. down with

26. After waiting an hour he realized that the bus was to come.

A. improbable

B. impossible

C. uncertain

D. unlikely

27. When Rosemary is concentrating she me of her grandmother.

A. reminds

B. recalls

C. revises

D. remembers

28. He refused to give up work, he had won a million dollars.

A. even though

B. despite

C. as though

D. however

II. Select the alternative that is not similar in meaning to the italicized word
31. wealthy
A. rich
32.

C. ingenious

D. well-to-do

B. puzzle

C. disturb

D. trouble

B. queer

C. unusual

D. average

upset
A. bother

33.

B. well-off

curious
A. odd

34.. position

A. job

B. post

C. location

D. employment

B. horror

C. dread

D. sensation

B. treaty

C. dispute

D. reason

B. happening

C. temptation

D. occurrence

B. remarkable

C. magnificent

D. extravagant

B. conquer

C. overcome

D. drag

B. endeavour

C. impetus

D. exertion

35. terror
A. fear
36. argument
A. debate
37.

event
A. incident

38.

splendid
A. wonderful

39. defeat
A. beat
40. effort
A. attempt

III. Decide the correct word form for each numbered gap.
When did the first toys come into existence? Did they represent an attempt by adults to
make children (41) .. , or did they arise from the various playful (42) .. of children
themselves? As everyone knows, the young frequently (43) .. the behaviour of their (44)
.. , and in their play, they often adopt objects (45) ..by adults for entirely different
purposes. These objects (46) .. and lead to games in which everyday articles often play
unusual and (47) .. roles.
It is rather (48) .. that for an explanation of the origin of toys, we cannot turn to
folk stories. However, no traditional talc (49) .. to the origin of toys exists, and so our
knowledge (50) .. to archeological study and From documents.
41. A. happy

B. happiness

C. happily

D. unhappy

42. A. acts

B. actions

C. activities

D. active

43. A. copying

B. copy

C. copies

D. copied

44. A. olders

B. elderly

C. elders

D. olds

45. A. using

B. used

C. to use

D. to be used

46. A. courage

B. are courageous

C. encourage

D. encouraging

47. A. expected

B. expecting

C. unexpected

D. unexpecting

48. A. surprise

B. surprised

C. surprising

D. surprisingly

49. A. relates

B. related

C. relation

D. relationship

50. A. restricts

B. restricted

C. is restricted

D. is restricting

IV. Choose the correct verb for each gap in the following passage.
Although women now have the freedom to (51) .. whether or not the have children.

it is generally a joint decision between the partners involved In fact, in cultures which (52)
.. strongly traditional, the decision to have a baby may well be (53) .. by society,
family and the man of the family rather than the mother. Furthermore, in most modem
societies, with (54) .. expectations from both men and women, there is an obvious need
for major decisions, such as starting a family. Thus it is unreasonable to suggest that women
are solely responsible for the decision to have a child, and therefore it is unreasonable that
they are responsible for (55) .. the child up.
51. A. select

B. elect

C. choose

D. know

52. A. stay

B. remain

C. keep

D. become

53. A. infected

B. affected

C. done

D. finished

54. A. raising

B. decreasing

C. arising

D. increasing

55. A. bringing

B. educating

C. raising

D. keeping

Part C. READING
I. Decide the best preposition to be used in each gap in the passage.
According to some scientists, high-risk sports can be valuable (56) certain types
of people. Such activities help them learn that being frighten doesn't mean that they have to
lose control. The recent fashion (57) jumping from bridges attached (58) a
length of elastic rope, known as "bungee jumping", has now been tried by millions of people
(59) the world, and interest (60) it is continuing to grow.
Before the special elastic rope is tightened (61) them, jumpers reach
speeds of nearly 160 kph. First-timers are usually too terrified to open their mouths, and when
they are finally lowered safely (62) the ground, they walk around with broad smiles
(63) their faces,. saying repeatedly how amazing it was. However, for some people, it
is only the embarrassment of refusing to jump (64) the last minute that finally
persuades them to conquer the fear of heights and push them off (65) space.
56. A. to

B. for

C. with

D. of

57. A. to

B. for

C. in

D. of

58. A. to

B. for

C. with

D. by

59. A. on

B. over

C. round

D. in

60. A. for

B. as

C. in

D. like

61. A. to

B. for

C. around

D. of

62. A. on

B. to

C. over

D. above

63. A. on

B. to

C. over

D. in

64. A. in

B. for

C. until

D. at

65. A. in

B. from

C. into

D. onto

II. Read the passage and circle the best answer for each question.
In the very distant geological past all animals were aquatic. The very first vertebrates, or
animals with backbones, of which we have any fossil record, lived in the water. These
vertebrates, the fish, were adapted to underwater living. Their streamlined bodies were
covered with scales to, reduce surface friction: they had muscular tails so that they could
swim swiftly in such a dense medium as water; and they were endowed with gills for
breathing underwater.
Descendants of fish-type ancestors crossed the seashore barrier and accommodated
themselves to life on land. As amphibians, they possessed limbs instead of fins and lungs
instead of gills. But they never became completely free of the bonds that tied them to the
water; even today_ many amphibians return to the water to lay their eggs.
Millions of years after the first clumsy amphibians crawled over the land newer types of
land dwellers appeared; these animals gave rise to the present-day reptiles and mammals.
They were more completely converted for land dwelling, with bodies and biological activities
far different fro; those of fish. With these special adaptations, mammals have been able to
colonize the woods and meadows, the deserts and high mountains, often far removed from the
sea.
66. Of the animals with backbones, the first to appear were
A. mammals

B. fish

C. amphibians

D. birds

67. Fish are suited to underwater life because of their


A. scales

B. gills

C. streamlines shape D. All of the ahoy

68. In converting to land life, animals acquired


A. eyes

B. lungs

C. tails

D. warm blood

69. The word "descendants" in line 7 is closest in meaning to


A. descenders

B. offspring

C. grandchildren

D. ancestors

70. The passage suggests that the first amphibians used their limbs to
A. swim

B. crawl

C. jump

D. run

71. An example of an amphibian's incomplete adaptation to land life is its


A. return to water to lay eggs

B. scales-covered skin

C. need to keep its skin wet

D. inability to breath air

72. Animals found desert living possible only


A. when they became amphibious
B. if they migrated to the sea periodically
C. after they could walk on two feet
D. when they were fully adapted to land

73. The seashore was a barrier for descendants of fish type creatures in that
A. the land once rose much higher above the sea
B. crossing it required bodily changes
C. every attempt to cross it ended in death
D. once they crossed there was no return

74. The word "They" in line 13 refers to


A. animals

B. amphibians

C. land dwellers

D. reptiles and mammals

75. The adaptation process described in the article was accomplished .


A. over millions of years

B. through biological changes

C. by the receding of the seas

D. both a and b

III. Read the text and then decide which word best fits each space. Circle the letter you
choose for each question.
MEDIA AND ADVERTISING
After more than fifty years of television, it might seem only obvious to conclude that it
is here to (76) .There have been many objections to it during, this time, of course, and
(77) a variety of grounds. Did it cause eye-strain? Was the (78) bombarding
us with radioactivity? Did the advertisements contain subliminal messages, persuading us to
buy more? Did children turn to violence through watching it, either because so (79)
programmes taught them how to shoot, rob, and kill, or because they had to do something to
counteract the hours they had spent glued to the tiny screen? Or did it simply create a vast
passive (80) drugged by glamorous serials and inane situation (81) ? On the
other hand did it increase anxiety by sensationalizing the news [or the news which was (82)
by suitable pictures] and filling our living rooms with war, famine and political
unrest? (83) in all, television proved to be the all-purpose scapegoat for the second
half of the century, blamed for everything, but above all, eagerly watched. For no (84)
how much we despised it, feared it, were bored by it, or felt that it took us away from the old
paradise of family conversation and hobbies such as collecting stamps, we never turned it off
We kept staring at the screen, aware that our own tiny (85) was in if we looked
carefully.
76. A. be

B. stay

C. exist

D. prolong

77. A. with

B. over

C. by

D. on

78. A. screen

B. danger

C. machine

D. reason

79. A. that

B. far

C. many

D. what

80. A. programme

B. personality

C. audience

D. tense

81. A. comedies

B. programmes

C. perhaps

D. consequently

82. A. taken

B. presented

C. capable

D. accompanied

83. A. Taken

B. All

C. Somewhat

D. Thus

84. A. one

B. matter

C. difference

D. reason

85. A. fault

B. reflection

C. situation

D. consciousness

Part D. WRITING
Choose the sentence A, B, C, or D which is closest in meaning to the sign.
86.
WARNING :
Street musicians are not allowed to perform in this station.
A. Musicians must not play here.
B. Musicians do not have to play here.
C. Musicians can leave instruments here.
D. Musicians must keep their instruments with them.

87.
Latecomers will not be admitted until the interval.
A. The play will start later than usual today.
B. Please be quiet because the play has started.
C. We never start late for any reason.
D. If you arrive late you cannot go straight in.

88.
The management does not accept responsibility for property
left in the dining room.
A. You cannot bring your luggage into the dining room.
B. You must look after your things yourself.
C. You should give your things to the manager.
D. You should lock things in your suitcase.

89.
Outdoors shoes must not be worn in the sports
hall.
A. Hand in your shoes at the sports hall.
B. Don't leave your shoes outside the sports hall.
C. Change your shoes before entering the sports hall.

D. Please wear shoes all the time in the sports hall.

90.
Payment must be made at time of booking
A. Allow plenty of time for booking.
B. Pay when you book.
C. Sign the book when you pay
D. It is too late to make a booking.

91.
Please lower volume from 11 p.m
A.

Switch off at 11.

B.Do

not make a noise at midnight.

C.

Reduce the noise after 11.

D.

Switch off before midnight.

92.
Visas will only be issued to holders of return air tickets
A.

Get a visa before booking your flight.

B.Book

a return ticket before you ask for a visa.

C.

You can't fly without a visa.

D.

Apply for a visa before you book your return flight.

93.
Public meeting to discuss proposed motorway
A.

Many people don't want a motorway.

B.Building

the motorway starts today.

C.

People are going to talk about building a motorway.

D.

Everyone travelling on the motorway should meet here.

94.
Do not lean out of the window
A.

Open the window carefully.

A.

This window cannot be opened.

B.Do
C.

not put anything against the window.

Do not put your head out of the window.

95.
Members are reminded that they are responsible for any damage caused by their guests.
A.

If your guests break anything, you'll have to pay.

A.

If guests are hurt, members have to look after them.

B.

Please don't let guests make too much noise.

B.Please

remember to look after guests very politely.

II. Choose one sentence A, B, C, or D that has different meaning to the on sentence(s).
96. I think no city in the world is more beautiful than Venice.
A. According to me, Venice is the most beautiful city in the world.
B. For me, Venice is more beautiful than any other city in the world.
C. I think all the cities in the world are as beautiful as Venice.
D. I think all the cities in the world are less beautiful than Venice.

97. Animals can't speak our language. They can't tell us when they are Link, or annoyed
A. If animals could speak, they would be able to tell us when the: unhappy or annoyed.
B. One day we could speak animals' language and know when they aa unhappy or annoyed.
A. We don't know when animals are unhappy or annoyed because we do:-,7 know their

language.
B. We don't understand animals because we can't speak their language.

98. While I strongly disapprove of your behaviour, I will help you this time.
A. Despite the fact that I strongly disapprove of your behaviour, I will help you this time.
B. I will help you this time because I strongly disapprove of your behaviour
A. I will help you this time in spite of my strong disapproval of behaviour.
C. I strongly disapprove of your behaviour, but I will help you this time

99. My sister and I had never been to Beijing before.


A. Neither my sister nor I had been to Beijing before.
B. My sister had never been to Beijing before, and neither had I.
C. I had never been to Beijing before, and my sister hadn't either.
D. Both my sister and I had been to Beijing before.

100. She was irresponsible at times, so she lost her job.


A. She lost her job because she was irresponsible sometimes.
B. She was not always responsible. Therefore, she was dismissed.
C. She got her job though she was irresponsible.
D. The reason why she lost her job was that she was not always responsible.

English objective test 5


Time allowed: 90 min.
Part A. PHONETICS
I. Choose the word in each group that has the underlined, italic part pronounced
differently from the rest.
1. A. overhead

B. teammate

C. beacon

D. lean

2. A. challenge

B. snatch

C. brochure

D. chocolate

3. A. arrive

B. steward

C. coward

D. award

4. A. ranger

B. sugar

C. singer

D. giant

5. A. jolly

B. liquefy

C. army

D. crystal

II. Choose a word in each line that has different stress pattern.
6. A. counselor

B. description

C. inspector

D. amendment

7. A. inexpensive

B. psychiatry

C. patriotic

D. scientific

8. A. priority

B. terrifying

C. identify

D. compulsory

9. A. earthenware

B. evidence

C. disappear

D. slavery

10. A. generous

B. humorous

C. continent

D. insomnia

Part B. LEXICO - GRAMMAR


I. Choose the correct answer for each sentence.
11. We are . him to arrive at any moment.
A. wishing

B. hoping

C. expecting

D. waiting

12. The house still remains in. after the hurricane.


A. good condition

B. a good condition

C. good conditions

D. the good condition

13. My uncle is in . of 60 engineers and workers.


A. management

B. leadership

C. direction

D. charge

14. If they aren't careful with their money, they'll get into .
A. loss

B. problem

C. debt

D. missing

15. When we arrived at the campsite, it was with rain


A. running

B. pouring

C. falling

D. dropping

16. He promised to mend the broken wheel soon without............

A. fail

B. failure

C. trouble

D. mistake

17. Many scientists are sure there is on other planets.

A. existence

B. creature

C. people

18. People who are unemployed can receive the

D. life

A. pension

B. dole

C. scholarship

D. allowance

19. Some workers earn more money by working

A. over-hours

B. over

C. overtime

D. extra time

20. Before he left school, his father told him to start thinking about choosing a

A. profession

B. career

C. work

D. occupation

21. You can't smoke in here. It is the law

A. anti

B. against

C. opposite

D. according

C. stained

D. puzzled

22. She her work by being careless.

A. threatened

B. spoiled

23. You need a special to go into this part of the area.

A. permit

B. permission

C. allowance

D. agreement

24. Write to me and tell me about your holiday in China.

A. every

B. much

C. some

D. all

25. The dentist told him to open his mouth

A. broad

B. much

C. greatly

D. wide

26. I walked away as calmly as I could they thought I was a thief.

A. in case

B. or else

C. to avoid

D. owing to

27. The stolen jewels were a lot of money.

A. valued

B. cost

C. worth

D. priced

C. hunting

D. detecting

28. The children loved the old castle.

A. exploring

B. discovering

29. being tired, I went for a picnic with my family.

A. That

B. Since

C. Although

D. Despite

30. Armstrong is not very fit. He doesn't any exercise.

A. practise

B. have

C. take

D. make

II. Choose the best modal verb for the gap in each of the following sentences.
31.. Patrick has got a yacht and a helicopter. He be extremely rich.
A. can

B. must

C. should

D. may

32. Why didn't you do it? You promised you , for certain.

A. could

B. must

C. would

D. might

33. They left only an hour ago. They hardly have arrived yet.

A. can

B. can't

C. shouldn't

D. may

34. I can't see my umbrella anywhere. I have left it in my office.

A. can

B. must

C. should

D. would

32. Have you heard about all the complaints? It have been a pleasant holiday for them.

A. can't

B. mustn't

C. shouldn't

D. mayn't

33. There's a chance that he'll arrive in time for supper, but he be a lot later.

A. would

B. must

C. should

D might

34. We waited for you for over an hour. You really have telephoned to say you were

not coming.
A. can

B. must

C. might

D. need

35. You didn't have to come all the way by bus. We have gone to the station in the

car to pick you up.


A. could

B. must

C. would

D. should

35. You have told that joke. Everybody has heard it too often.

A. can't

B. mustn't

C. shouldn't

D. needn't

40. The Mercedes overtook us at a very high speed. It have been doing more than 150
k.p.h.
A. could

B. must

C. should

D. might

III. Choose the correct word form to finish the passage.


A POPULAR WRITER
Emma Harte, in Barbara Taylor Bradford's novel, was a poor lonely girl who became the
(41) owner of an international chain of stores. Like the woman she writes about, Ms.
Bradford is beautiful and (42) She left school at sixteen and became a (43)
After twenty-three years of this work, she made the (44) to start writing novels. She is
now one of the most highly-paid (45) in the world Was Emma Harte's story based on
Ms. Bradford's own (46) successful life? "Fm afraid not," she said with "My life has
been quite different from Emma Harte's. She was to be born into a poor family. I came from a
middle-class home and I'm (49) married to a rich American film producer. The only
thing I share with my heroine is her (50) to work hard:"
41. A. wealth

B. wealthy

C. wealthier

D. wealthiest

42. A. ambitious

B. ambition

C. ambitiously

D. more ambitious

43. A. journal

B. journalist

C. journalism

D. journalizer

44. A. decide

B. decisive

C. decision

D. decisiveness

45. A. novel

B. novels

C. novelists

D. novel sellers

46. A. credible

B. credibly

C. incredible

D. incredibly

47. A. amuse

B. amusing

C. amusement

D. amusements

48. A. lucky

B. luckily

C. unlucky

D. unluckily

49. A. happy

B. happily

C. unhappy

D. unhappily

50. A. able

B. ability

C. inability

D. disable

IV. There is a mistake in the four underlined parts of each sentence. Find the mistakes
(A, B, C, or D).
51. Children enjoy telling and listening to ghosts stories, especially on Halloween night.
A

52. We had better to review this chapter carefully because we will have some questions
A

on it on our test tomorrow.


51.

The little boy's mother bought him a five-speeds racing bicycle for his birthday.
A

52.

Despite the time of the year, yesterday's temperature was enough heat to turn on the
A

air conditioning.
51. Danny spent such enjoyable vacation in Vietnam this summer that he plans to return
A

as soon as he saves enough money.


D
52. We wish today was sunny so that we could spend the day in the countryside
A

to communicate with nature.


D
53. If father would have a lot of money, he could buy us a modern computer.
A

54. Anybody who plans to attend the meeting ought to send a note for the
A

chairperson.

55. We thought our cameras were same, but his is different to the one that I bought.
A

56. No one in our office wants to drive to work any more because there are always
A

traffic jams in rush hour.


D
Part C. READING
I.

Read the two passages and circle the correct answer for each question.
The moon revolves once on its axis each time it orbits the Earth thus always presenting
the face to Earthbound observers. However, even to the unaided eye, this unchanging face

shows two contrasting types of landscape - dark, plain-like areas of low relief, and
brighter decidedly more rugged regions which cover about two-thirds of the surface.
Early astronomers mistakenly referred to the smooth dark areas as mania (or seas), giving
the name terrae (or lands) to the bright upland regions. The terms have persisted since,
even though the Moon's surface has long been known to he completely waterless.
61. The Moon's revolution is responsible for .....
A. the way it orbits the Sun
B. the way its own axis is referred to
C. the way Earthbound people can observe it
D. the way it is presented in history books
62. To us, the Moon's face.........

A. never changes

B. always changes

C. changes as we move our position

D. sometimes changes

63. On the Moon there are ..........

A. many kinds of landscapes

B. very few contrasts

C. light and dark areas

D. only low plains to be observe

64. One third of the face of the Moon we can see is composed of .........
A. very rough areas

B. low-lying areas

C. light areas

D. upland areas

65. One mistake early astronomers made was ..........


A. to confuse the words mania and terrae
B. to ignore the smooth dark areas
C. to consider some areas to be seas
D. to think that the upland regions were bright
There are three kinds of goals: short-term, medium-range and long-term goals. Shortrange goals are those that usually deal with current activities which we can apply on a
daily basis. Such goals can be achieved in a week or less, or two weeks, or possibly
months. It should be remembered that jug-as a building is no stronger than its foundation,
our long-term goals cans amount to very much without the achievement of solid shortterm goal- Upon completing our short-term goals, we should date the occasion then add
new short-term goals that will build on those that have be.:- completed
The intermediate goals build on the foundation of the short-range go They might deal
with just one term of school or the entire school year, or they could even extend for
several years. Any time you move a step at a time, you should never allow yourself to
become discouraged or overwhelm As you complete each step, you will enforce the belief

in your ability grow and succeed. And as your list of compassion dates grows, your
motivation and desire will increase.
Long-range goals may be related to our dreams of the future. They mi: cover five years or
more. Life is not a static thing. We should never allow a long-term goal to limit us or our
course of action.
66. Our long-term goals mean a lot
A. if we complete our short-range goals
B. if we cannot reach solid short-term goals
C. if we write down the dates
D. if we put forward some plans
67. New short-term goals are built upon ..........
A. two years

B. long-term goals

C. current activities

D. the goals that have been completed

68. When we complete each step of our goals,


A. we will win final success
B. we are overwhelmed
C. we should build up confidence of success
D. we have strong desire for setting new goals
69. Once our goals are drawn up, ...........
A. we should stick to them until we complete them
B. we may change our goals as we have new ideas and opportunities
C. we'd better wait for the exciting news of success
D. we have made great decisions
70. It is implied but not stated in the passage that ........
A. those who have long-term goals will succeed
B. writing down the dates may discourage you
C. the goal is only a guide for us to reach Our destination
D. everyone should have a goal
II. Read the text and then decide which option best fits each numbered space.
There is much more water than land (71)................. the surface of the earth. The seas and
oceans (72) ................nearly four-fifths of the whole world, and only one-fifth of
(73)................ land. If you travelled over the earth (74) ................ different directions, you
would have to spend (75)................more of your time (76)................on water than on roads or
railways. We sometimes forget that (77) ................every mile of land there (78)................four
miles of water.

There is much water (79)................ the surface of our earth that we (80) ................ to use two
words to describe. We use the word SEAS (81) ................ those parts of water surface which
(82) ............only a few hundreds of miles wide, the OCEANS to describe (83)................huge
areas of water (84) ................are thousands of miles wide (85) ................very deep.
71.

A. in

B. on

C. from

D. over

72.

A. covered

B. covering

C. cover

D. to cover

73.

A. them are

B. its

C. it is

D. it's

74.

A. in

B. for

C. to

D. by

75.

A. many

B. few

C. much

D. too

76.

A. moving

B. to move

C. move

D. moved

77.

A. on

B. in

C. within

D. for

78.

A. have

B. is

C. are

D. being

79.

A. on

B. in

C. under

D. across

80.

A. must

B. should

C. have

D. would

81.

A. describe

B. describes

C. to describe

D. describe:

82.

A. have

B. is

C. are

D. will be

83.

A. a

B. the

C. little

D. a little

84.

A. what

B. where

C. which

D. who

85.

A. then

B. and

C. but

D. also

Part D. WRITING
Choose the sentence nearest in meaning to the one in italics.
86. The new law will not be popular with the man in the street.
A. Passers-by will not like the new law.
B. Ordinary men will not like the new law.
C. Poor men will not like the new law.
D. Pedestrians will not like the new law.
87. The boy was too fat to run far.
A. The boy was very fat and couldn't run far.
B. The boy's fatness didn't stop him running far.
C. The boy had to run a lot because he was fat.
D. The boy became ill because he was too fat.
88. "I'll give you a lift home if you like."
A. He offered to give me a lift home.
B. I liked him to give me a lift home.
C. He could install a lift in my home.

D. If I liked, he would install a lift in my home.


89. The plane had scarcely taken off when it crashed.
A. The plane took off and soon afterwards it crashed.
B. The plane scarcely crashed after it took off.
C. When the plane was taking off, it crashed.
D. When the plane crashed, it had not taken off yet.
90. During the walk the two smallest children ceased to keep up with the main group.
A. During the walk the two smallest children could keep up with the main group.
B. During the walk the two smallest children were left behindd.
C. During the walk the two smallest children were ahead of the main group.
D. During the walk the two smallest children refused to keep up with the main group.
91. It is possible Wendy didn't hear what the homework was.
A. Wendy may not have heard what the homework was.
B. It is not possible for Wendy to hear what the homework was.
C. Wendy can't help hearing what the homework was.
D. Wendy can't have heard what the homework was.
92. If it hadn't been for my father's encouragement, I would never have become a chef.
A. If my father hadn't been courageous, I wouldn't have become a chef.
B. If my father had encouraged me, I would never have become a chef.
C. My father didn't encouraged me to become a chef.
D. It was my father who encouraged me to become a chef.
93. I think I should have cooked more food. There's nothing left now.
A. I shouldn't have cooked so much food.
B. I regret cooking too much food now.
C. I didn't cook much food and I think it is OK now.
D. I didn't cook much food and I think it was a mistake.
94. You can't have added any garlic or we'd be able to taste it.
A. It's possible you added garlic but I am not sure.
B. I am sure you didn't add garlic.
C. You should not have added garlic.
D. You needn't have added garlic.
95. You needn't have washed the sheets. The hotel staff do the cleaning.
A. It's good that you washed the sheets.
B. It wasn't necessary to wash the sheets, even though you did.
C. It was necessary to wash the sheets, but you didn't do it.

D. It was unnecessary to wash the sheets, and you didn't do it.


II. Choose the best question (A, B, C, or D) for the following answers.
96. A. What kinds of books are sold in this bookstore?
B. What kinds of books are lent to the students?
C. What kinds of books are you fond of reading?
D. Which books can you read easily?
- Books that are easy to read and exciting.
97. A. Who are your writers?
B. Who are your favorite authors?
C. Which writers do you like to talk about?
D. Which authors do you like to talk to?
- Oh, John Le Carre, Len Deighton writers like that.
98. A. Do you only read spy stories?
B. Don't you like spy stories?
C. Don't you like me to lend you some spy stories?
D. Do you avoid reading spy stones?
- No, not only spy stories. I do read other kinds of books, too.
99. A. And what are you writing at the moment?
B. And what are your writers doing at the moment?
C. And what are you reading at the moment?
D. And are you reading a spy story at the moment?
- At the moment? Well, I'm on the very last chapter of a book by J.
100. A. What is the writer's name?
B. What is the book's name?
C. What is the title of the book?
D. What are the names of the main characters?
- Harry Porter and the half-blood prince.

English objective test 6


time allowed: 90 min.
Part A PHONETICS
1. Choose the word in each group that has the underlined part pronounced differently
from the rest.
1.

A. kites

B. sketches

C. oranges

D. buzzes

2.

A. breathe

B. paths

C. wither

D. marathon

3.

A. supreme

B. complete

C. criteria

D. refund

4.

A. question

B. conquest

C. quest

D. picturesque

5.

A. determine

B. examine

C. discipline

D. undermine

II. Decide the word which is stressed differently from the others.
6.

A. emphasize

B. equipment

C. improvement D. distinguish

7.

A. technology

B. expenditure

C. irrespective

D. discovery

8.

A. iron

B. career

C. supreme

D. absorb

9.

A. universal

B. kilometer

C. cooperate

D. community

10.

A. economical

B. productivity

C. unpleasantly

D. unexpectedly

Part B. LEXICO - GRAMMAR


I.

Choose the correct answer to finish each of the sentences.

11. He was clearly.........to see her again.


A. delightful

B. delighted

C. cheerful

D. cheered

12. It was a very...........evening. Nothing really happened.


A. dull

B. fed up

C. bored

D. disinterested

13. I am not really........ this kind of music. I prefer music you can dance to.
A. in

B. for

C. into

D. with

14. It's ........ for people to get depressed if they are out of work.
A. normal

B. everyday

C. ordinary

D. typical

15. He deals with the........matters in the office, his boss deals with the important ones.
A. everyday

B. average

C. usual

D. standard

16. House prices ............. greatly from one area to the next.
A. contrast

B. vary

C. distinguish

D. differentiate

17. Eighty kilometers is the.........fifty miles.


A. equivalent of

B. equivalent from C. equal of

D. equal to

18. She is quite ....... and is certainly capable of doing a more demanding than the one she is
doing now.

A. wise

B. bright

C. practical

D. sensible

19. I read a(n) .......... of the accident in the newspaper.


A. material

B. effort

C. account

D. event

20. She quit her job because she was not .......... with her salary.
A. contented

B. delicate

C. serious

D. grateful

II. Choose the best word form to complete each sentence.


21. The ............ of the air has certainly brought great benefits to man.
A. conquer

B. conquest

C. conqueror

D. conquering

22. More ............ should be given to these people to continue their studies.
A. courage

B. encourage

C. encouraging

D. encouragement

23. It is essential that the Professor ........... warmly welcomed at the airport.
A. is

B. be

C. will be

D. would be

24. People who live in cities spend their lives in an atmosphere of ........... noise.
A. continuous

B. continual

C. continuing

D. continued

25. We walk in streets where the noise of traffic is almost


A. deaf

B. deafening

C. deafened

D. deafness

26. If trouble breaks out, they may be arrested and even ...........
A. prisoned

B. imprisoned

C. imprisonment D. imprisoning

27. Retirees now have all day in which to do the ........... things they always said they never
had time to do before.
A. numberless

B. numbering

C. numerous

D. numerical

28. Travelling in big cities is becoming more ........ every day.


A. troublesome

B. trouble

C. troubled

D. troubling

29. To be successful, an artist must show great ...........


A. origin

B. origins

C. original

D. originality

30. It's good news: the factory's ........... has increased considerably this year.
A. output

B. input

C. put-in

D. put-out

II. Choose the right option for each gap in the following sentences.
31. ............. you called I was at the conference.
A. At the time

B. By the time

C. In the time

D. For the time

32. Yes, we do eat out ......... , but not very often. It's so expensive nowadays.
A. in time

B. from time to time C. against time D. at one time

33. We didn't much like the new neighbours at first, but .......... we grew quite fond of them.
A. by the time

B. in time

C. at the same time D. against time

34. .............., his secretary wears the most extraordinary clothes.

A. With time

B. At one time

C. At times

D. In time

35. We're fighting ............. to finish the job for you before the end of the week.
A. in time

B. on time

C. by the time

D. against time

36. ............. the fire brigade found our mountain house, we had put out the fire ourselves with
the garden hosepipe.
A. With time

B. By the time

C. At the same time D. In time

37. They're most unpunctual people but because the boss was going, to be at the party they
had to be .......... for once.
A. in time

B. to time

C. with time

D. at the same time

38. You're quite right, of course, but ......... you needn't have been so rude about it
A. at one time

B. from time to time C. at the same time D. at times

39. We were allowed ............. to park our car outside our front .
A. by the time

B. in time

C. with time

D. at one time

40. It's wonderful! The trains here always leave, and nearly always arrive ...............
A. in time

B. on time

C. to time

D. with time

IV . There is a mistake in the four underlined parts of each sentence. Find the Mistakes
(A, B, C, or D).
41. Card Anderson discovered two atomic particles that he identified while studied cosmic
A

rays.
42. No one knows exactly how many species of animals lives on earth.
A

43. Assessment instruments in nursery schools they feature items and other materials different
A

from those on elementary school tests.


C

44. Michigan's rivers, inlets, and lakes attract tourists who derive pleasure from canoeing and
A

water-ski.
D
45. Analysts have translated clay tablets that demonstrate that the Babylonians were high
A

skilled in Arithmetic.
46. The visual nerves of the brain interprets wave-lengths of light as perceptions of color.

47. It is possible to have wealth but little income and having income but no wealth.
A

48. When a criminal case goes to trial, the defendant may election to have it heard either by a
A

jury or by a judge.
D
49. John Keynes used his knowledges of economics to help his colleagues and himself.
A

50. Government offices store and maintain such documents as certificates of birth, marrying,
A

and death.
D
Part C. READING
I. Read the following passage and choose the correct answer (A, B, C, or D) for each of
the questions.
Quite different from storm surges are the giant sea waves called tsunamis, which derive
their name from the Japanese expression for "high water in a harbor". These waves are
also referred to by the general public as tidal waves, although they have relatively little to
do with tides. Scientists often refer to them as seismic sea waves, far more appropriate in
that they do result from undersea seismic activity.
Tsunamis are caused when the sea bottom suddenly moves, during an underwater
earthquake or volcano, for example, and the water above the moving earth is suddenly
displaced. This sudden shift of water sets off a series of waves. These waves can travel
great distances at speeds close to 700 kilometers per hour. In the open ocean, tsunamis
have little noticeable amplitude, often no more than one or two meters. It is when they hit
the shallow water the coast that they increase in height, possibly up to 40 meters.
Tsunamis often occur in the Pacific because the Pacific is an area of heavy seismic
activity. Two areas of the Pacific well accustomed to the threat of tsunamis arc Japan and
Hawaii. Because the seismic activity that causes tsunamis in Japan often occurs on the
ocean bottom quite close to the islands, the tsunamis that hit Japan often comes with little
warning and can therefore prove disastrous. Most of the tsunamis that hit the Hawaiian
Islands, however, originate thousands of miles away near the coast of Alaska, so these
tsunamis have a much greater distance to travel and the inhabitants of Hawaii generally
have time for warning of their imminent arrival.

Tsunamis are certainly not limited to Japan and Hawaii. In 1755, Europe experienced a
calamitous tsunami, when movement along the fault lines near the Azores caused a
massive tsunami to sweep onto the Portuguese coast and flood the heavily populated area
around Lisbon. The greatest tsunami on record occurred on the other side of the world in
1883 when the Krakatoa volcano underwent a massive explosion, sending waves more
than 30 meters high onto nearby Indonesian islands; the tsunami from this volcano
actually traveled around the world and was witnessed as far away as the English Channel.
51. The paragraph preceding this passage most probably discusses
A. tidal waves

B. tides

C. storm surges

D. underwater earthquakes

52. According to the passage, all of the following are true about tidal waves EXCEPT that
A. they are the same as tsunamis
B. they are caused by sudden changes in high and low tides
C. this terminology is not used by the scientific community
D. they refer to the same phenomenon as seismic sea waves
53. The world "displaced" in the second paragraph is closest in meaning to..............
A. located

B. not pleased

C. filtered

D. moved

54. It can be inferred from the passage that tsunamis ............


A. cause severe damage in the middle of the ocean
B. generally reach heights greater than 40 meters
C. are far more dangerous on the coast than in the open ocean
D. are often identified by ships on the ocean
55. As used in the passage, water that is "shallow" isn't ..........
A. clear

B. deep

C. tidal

D. coastal

56. A main difference between tsunamis in Japan and in Hawaii is that tsunamis in Japan are
more likely to............. .
A. arrive without warning

B. be less of a problem

C. come from greater distances

D. originate in Alaska

57. The possessive "their" in the third paragraph refers to ......... .


A. the Hawaiian islands

B. thousands of miles

C. these tsunamis

D. the inhabitants of Hawaii

58. A "calamitous" tsunami in the last paragraph is one that is .................


A. expected

C. at fault

B. extremely calm D. disastrous

59. From the expression "on record" in the last paragraph, it can be inferred the tsunami that
accompanied the Krakatoa volcano ..............

A. occurred before efficient records were kept


B. was not as strong as the tsunami in Lisbon
C. was filmed as it was happening
D. might not be the greatest tsunami ever
60. The passage suggests that - the tsunami resulting from the Kr- volcano ..............
A. caused volcanic explosions in the English Channel
B. was far more destructive close to the source than far away
C. was unobserved outside of the Indonesian islands
D. resulted in little damage
II. Read the following text and choose the correct option for each of the
numbered gap.
THE HANGING GARDENS OF BABYLON
The hanging gardens of Babylon were considered to be one of the Seven Wonders of the
(61) ............World. They are believed to have been built by king Nebuchadnezzar in the sixth
(62) ............ BC as a present for his wife, Amytis.
The gardens were (63) ............ in layers - one on top of the other, much like a modern
multi-storey car (64) ............ although a lot more (65) ............ to look at. Each layer was a
large terrace (66)............ with tropical flowers, plants and !trees. The large (67) ............of
water which these plants required was (68) ............ from the river Euphrates nearby. It is said
that Nebuchadnezzar and his wife would sit in the (69) ............ of the gardens and (70)
............down on the city of Babylon below. The gardens' fame quickly (71) ............, and
travelers would come from far and wide to (72) ............ them. Even thousands of years ago,
people used to go (73)............The city of Babylon itself was also famous throughout the
whole (74)...........for its beautiful buildings, huge tiled walls and magnificent gatesmade of
brass.
Sadly, nothing (75)....................today of the beautiful hanging gardens, and the city of
Babylon lies in ruins in what is modern-day Iraq.
61.

A. antique

B. ancient

C. historical

D. traditional

62.

A. decade

B. period

C. era

D. century

63.

A. constructed

B. assembled

C. collected

D. invented

64.

A. park

B. stop

C. station

D. garage

65.

A. good-looking

B. attractive

C.handsome

D. adorable

66.

A. included

B. contained

C. filled

D. consisted

67.

A. total

B. sum

C. amount

D. number

68.

A. dragged

B. pulled

C. pushed

D. pumped

69.

A. shadow

B. shade

C. gloom

D. glow

70.

A. look

B. see

C. watch

D. observe

71.

A. distributed

B. spread

C.extended

D. moved

72.

A. approve

B. respect

C. admire

D. assess

73.

A. glimpsing

B. sightseeing

C. glancing

D. staring

74.

A. planet

B. globe

C. earth

D. world

75.

A remains

B. stays

C. waits

D. continue

III. Complete the following passage by choosing the right preposition.


FAILING A J0B INTERVIEW
When you first apply for a job, you might not succeed in getting it. always a good idea to ask
them to explain (76) ............. you what prevented from beating the other candidates. Don't
complain (77) ............. the situa, but ask them to advise you (78) ............. what you can do
better next Perhaps the interviewer disapproved (79) ............. or disagreed (80).............
something you said. Perhaps they just glanced (81) ............. your application saw something
that made it easy to choose (82) ............. you and ano-candidate. Don't regard it (83) .............
a failure, but recognize it (84) ............. chance to learn more. As long as you don't worry too
much (85) ............. it continue to believe in yourself, you'll eventually find the chance you
been waiting for. Then your family and friends will be able to congratu you on your success!
76.

A. for

B. to

C. about

D. with

77.

A. for

B. to

C. about

D. with

78.

A. against

B. on

C. about

D. with

79.

A. against

B. on

C. of

D. with

80.

A. against

B. on

C. of

D. with

81.

A. at

B. on

C. of

D. through

82.

A. from

B. of

C. between

D. among

83.

A. for

B. as

C. like

D. with

84.

A. for

B. as

C. like

D. with

85.

A. for

B. to

C. about

D. with

Part D: WRITING
I. Choose the sentence A, B, C, or D that is similar in meaning to the original sentence
given in italic.
86. She said it had nothing to do with me.
A. She told me that it was none of my business.
B. She told me that it had done nothing for me.

C. She said I did nothing about it.


D. She said she had done nothing for me.
87. His personal problems have an influence on his ability to do his job.
A. His personal problems enable him to do his job.
B. His ability influences his personal problems and his job.
C. His ability affects his personal problems in his job.
D. His personal problems affect his ability to do his job.
88.Only if you work hard now have you any chance of success.
A. Your chance of success affects your working hard.
B. Your chance of success depends on your working hard.
C. Working hard lessens your chance of success.
D. The harder you work, the more chances you have.
89. "Let's organize a club meeting on Saturday."
A. She offered to organize a club meeting on Saturday.
B. She suggested organizing a club meeting on Saturday.
C. We have to organize a club meeting on Saturday.
D. We are suggested to organize a club meeting on. Saturday.
90. If youdidn't cheat in the exam, you wouldn't be punished.
A. You were not punished as you didn't cheat in the exam.
B. You didn't cheat in the exam so as not to be punished.
C. Don't cheat in the exam and you won't be punished.
D. You never cheat in the exam, so you arc not punished.
91. "Shall I post the letter for you?"
A. He asked me if I should post the letter for him.
B. He offered to post the letter for me.
C. He wanted me to post the letter myself.
D. I wanted him to post the letter for me.
92. "I would suggest that we should try to get local support for the - motorway."
A. I suggested they should try to get local support for the new motorwa:.
B. I wanted to know why shouldn't we try to get local support for the -motorway.
C. My suggestion was that we try to get local support for the new moto
D. I suggested we should try getting local support for the new motorway_
93. "Waiter! This soup is too cold."
A. The customer called the waiter to try the cold soup.
B. The customer complained that the soup was too cold.

C. The customer called the waiter and said that the soup was too cold for
D. The waiter called the customer to give the cold soup.
94. If you re/Use to carry out my orders you will get the sack.
A. If you refuse to carry out my orders, you will be given a sack of clotheB. If you carry out my orders, you will be made redundant.
C. If you refuse to carry out my orders, you will be criticized.
D. If you don't carry out my orders you will be dismissed.
95. The decision is none of your business.
A. The decision is not covered in your business.
B. You are not busy with the decision.
C. The decision doesn't concern you.
D. Your business doesn't affect the decision.
II. Each of these sentences is followed by four suggested explanations but one is correct.
Decide the correct explanation (A, B, C, or D).
96. He was such a wet blanket at the party tonight!
A. He brought a blanket to the party.
B. He was wet through after the party.
C. He made people at the party wet through.
D. He spoiled other people's pleasure.
97. We are going to get into hot water when we arrive home.
A. We are going to have a nice hot bath.
B. We are going to have trouble.
C. We have to boil water when we arrive home.
D. The water will have become hot by the time we get home.
98. The Jacksons are terribly hard up.
A. They live a long way up the hill.
B. They are cruel people.
C. They are extremely poor.
D. They work very hard
99. I'll lay my cards on the table.
A. I'll have no secrets from you. cold for him.
B. I want to stop playing.
C. I'll be back in a moment or two.
D. Let's continue playing cards.
100. The boss is like a bear with a sore head today.

A. He badly needs a haircut.


B. He has a very bad headache.
C. He is in a bad mood.
D. He likes a teddy bear.

English objective test 7


Time allowed: 90 min.
Part A. PHONETICS
I. Choose the word in each group that has the underlined, italic part pronounced
differently from the rest.
1. A. leaning

B. reason

C. feature

D. pheasant

2. A. full

B. skull

C. pull

D. bull

3. A. myth

B. theme

C. thick

D. cloths

4. A. publish

B. stub

C. climb

D. bulb

5. A. insecticide

B. recount

C. campus

D. applicant

II. Find the word with the stress on the LAST syllable in each line.
6. A. happen

B. local

C. expect

D. wander

7. A. correspond

B. separate

C. conclusive

D. appliance

8. A. innovation

B. efficiency

C. misunderstand

D. identified

9: A. evidence

B. disappear

C. fluency

D. tropical

10.A. symbol

B. purpose

C. nuclear

D. exchange

Part B. LEXICO - GRAMMAR


I. Choose one option that has similar meaning to the word given.
11. region
A. religion
12.

B. change

C. ability

D. opinion

B. usual

C. informative

D. informal

B. fix

C. shut

D. open

B. mysterious

C. wealthy

D. worthless

close
A. lock

15.

D. nation

casual
A. baggy

14.

C. era

opportunity
A. chance

13.

B. area

precious
A. invaluable

II. Circle the best answer for each sentence.


16. The hall was very crowded with over fifty people __________into it.
A. pushed

B. packed

C. stuck

D. stuffed

17. The teachers at the school ________with flu one after the other.
A. went down

B. went off

C. went out

D. went under

18. To promote him so quickly you must have a very high ________of his ability.

A. view

B. opinion

C. idea

D. feeling

19. He was born during the war, which would ________him about 50 now.
A. give

B. make

C. age

D. calculate

20. The noise got ________ as the car disappeared into the distance.
A. smaller

B. fainter

C. weaker

D. slighter

21. had to be up early the next morning, so I ________ myself and left the party.
A. refused

B. thanked

C. excused

D. apologised

22. The ________ part of the week is always busy for me.
A. front

B. start

C. early

D. near

23. When you come tomorrow, why not ________ your brother with you?
A. fetch

B. take

C. bring

D. carry

24. We had to drive carefully because the road was icy in several ________ .
A. blocks

B. places

C. pieces

D. bits

25. The hotel receptionist said she would ________ what she could do about the dripping tap
immediately.
A. find

B. try

C. see

D. look

26. He ________ very quickly after a long illness.


A. covered

B. uncovered

C. discovered

D. recovered

27. As his car had broken down, she gave him a ________ to work.
A. drive

B. carry

C. lift

D. passage

28. It didn't take the inspector ________ to solve the murder.


A. time

B. much

C. lot

D. long

C. visage

D. light

29. His face has rather a sad ________ .


A. aspect

B. look

30. As it was so hot in the office he ________ his tie.


A. tightened

B. let out

C. slackened

D. left

III. Choose the correct form of the verbs for each blank.
When the North and the South finally (31) ________ down their arms in the end of the
American Civil War, they (32) ________ for over four years South, which (33) _______
several battles but (34) ________the war, (3) _______economically exhausted. It (36) ______
a tragedy that Abraham Lincoln. (37) _______ the North to victory and (38) _______ now
ready to be generous South, (39) _______ to make the peace. Five days after General Lee
(40)_______ Appmottox, Lincoln was assassinated
31.

A. laid

B. had laid

C. had been laying D. were playing

32.

A. fought

B. had fought

C. had been fighting D. were fighting

33.

A. won

B. had won

C. has won

D. was winning

34.

A. lost

B. has lost

C. had been losing D. was losing

35.

A. was

B. had been

C. has been

D. would be

36.

A. is

B. was

C. has been

D. would be

37.

A. led

B. had led

C. would lead

D. was leading

38.

A. is

B. was

C. has been

D. would be

39.

A. survived

B. surviving

C. hadn't survived D. didn't survive

40.

A. surrendered

B. surrendering

C. had surrendered D. was surrender

IV. Choose the correct preposition or adverb for each blank in the following sentences.
41. I can't make ............ who it is. He is too far-away.

A. out

B. in

C. for

D. up of

42. I am in charge when Mr. Smith is ............ . I took over from him on June 1st.
A. out

B. in

C. off

D. away

43. There are other harmful drugs ............. alcohol and nicotine.
A. except

B. from

C. beside

D. besides

44. She wasn't paying attention and crashed ............ a parked car.
A. with

B. down

C. into

D. over

C. for

D. after

45. This is the coldest winter ............ years.


A. since

B. in

46. He felt he was badly let ............ by his best friend.


A. down

B. off

C. out

D. away

47. They took ................his passport so that he wouldn't leave the country.
A. up

B. off

C. out

D. away_

48. The bank can make loan ............. individuals, organizations, governments, and
businesses.
A. with

B. for

C. out

D. to

49. I was besides myself with joy when my father said the car was ........... my
A. at

B. of

C. with

D. into

50. Spacemen have to practise living in weightlessness before they fly..........outer


space.
A. out

B. out of

C. to

D. into

Part C. READING
I. Circle the best answer for each space to complete the passage below.
Set in the red desert of central Australia is the mining town of Coober Pedy. At first sight, the
town looks ______(51) to many other such communities, but Coober Pedy is different. Sixty

per cent of its population of some 4,000 people live underground. There are today about 800
underground houses as well as shops, hotels and even churches in the town and the______(52)
hills. Once a site has been chosen, special tunnelling machines are ______(53) in create
passages and rooms in the sandstone. Rock pillars are left to ______(54) the roof, and doors
and windows are cut into the front. Houses are of all shapes and______ (55), the largest
having twenty rooms, and some even have their own swimming pool.
Living underground may (56) strange but in fact it has a ______(57) of advantages. In
summer, the temperature outside can______(58) an astonishing 47C, and in winter the nights
can be______(59) cold. However, inside the houses it remains a steady 25C all
year______(60). Many people say that living underground ______(61) them feel very secure.
There is no problem with noise from the neighbours and the houses are not ______(62) by the
fierce dust storms that regularly______(63) through the area. And of course, if your family
______(64) or lots of friends come to stay, you can ______(65) dig another room.
51. A. similar

B. like

C. same

D. alike

52. A. enclosing

B. close

C. near

D. surrounding

53. A. entered

B. brought

C. worked

D. placed

54. A. push

B. lift

C. rise

D. support

55. A. volumes

B. areas

C. sizes

D. numbers

56. A. consider

B. hear

C. suggest

D. sound

57. A. sum

B. plenty

C. number

D. total

58. A. achieve

B. reach

C. fulfill

D. hold

59. A. extremely

B. heavily

C. sharply

D. strongly

60. A. wide

B. round

C. across

D. along

61. A. makes

B. enables

C. allows

D. gets

62. A. spoiled

B. influenced

C. disturbed

D. affected

63. A. pour

B. sweep

C. flood

D. hurry

64. A. grows

B. rises

C. stretches

D. explodes

65. A. ever

B. regularly

C. always

D. only

II. Read the following passage and choose the best answer A, B, C or D for sentence
below.
WHAT SORT OF SHOPPER ARE YOU?
Love it or hate it, we all go shopping. But there are different types of shopper. Do you
know which kind you are? Abby Edwards asked around
A. Melanie, 22, dancer

I'm an addict - I can't spend enough money! When I have the cash there and then I don't get

miserable. I definitely have difficulty walking past sale signs. My boyfriend's exactly the
same, so we often drag each around the shops. We're an addicted couple!
B. Juliet, 28, personnel manager

I only shop for essentials I hate shopping and only go when I really have to When I do, I
know what I want and I won't settle for anything else. I find shopping tiring and there's always
something I'd rather be doing.
C. Lizzie, 41, secretary

I don't enjoy shopping in the least. I really dislike shopping for clothes can never find what I
want, or anyone to help me look for it. The shops too noisy, everything is disordered and I
find it an absolute nigh Fortunately, I rarely have to shop for clothes as most of my clothes are
given to me. As for food shopping, I go to the local supermarket once a week end get it over
and done with quickly!
D. Ann, 29, nursery worker
I love. shopping but only on certain days and never on a Saturday, as it's far too busy. I'd
say I'm a careful shopper ............ I always set off with a fair idea of what I want, and I never
snap things up immediately. I have to look in other shops, in case I can find a better bargain
It takes ages!
WHICH OF PEOPLE
compares prices before buying things?

66.______

finds it impossible to resist buying things?

67. ____

prefers spending her time on activities other than shopping?

68.______

feels unhappy when she cannot afford to go shopping?

69.______

does not need to buy clothes very often?

70.______

prefers going to the shops at quiet times?

71.______

is critical of the way shops are run?

72.______

goes shopping when it is really necessary?

73.______

never goes shopping at weekends?

74.______

enjoys shopping with other people?

75. _____

III. Choose the best preposition for each numbered gap.


Fish live almost everywhere. They are found in the near freezing waters of the Arctic and in
the steaming waters (76) ....... tropical jungles. They live in roaring streams and (77)..........
quiet underground rivers. Some fish make long journeys (78) .......... the ocean. Others spend
most of their life buried (79).......... sand on the ocean bottom. Most fish can't leave water; yet
some survive for months (80) .......... dried-up riverbeds.
Fish have enormous importance (81) .......... man. They provide food for millions of people.

Fishermen catch them for sport, and many people keep them (82)..........pets. Fish are also
important in the balance of nature. They eat plants and animals and (83) ......... turn, become
food (84) .......... plants and animals. Fish thus help keep (85)..........balance the total number of
plants and clothes as I animals on the earth.
76.

A. on

B. in

C. among

D. inside

77.

A. in

B. on

C. under

D. inside

78.

A. along

B. through

C. across

D. into

79.

A. on

B. in

C. under

D. among

80.

A. on

B. in

C. under

D. among

81.

A. for

B. to

C. with

D. toward

82.

A. as

B. like

C. for

D. with

83.

A. on

B. in

C. to

D. at

84.

A. to

B. of

C. for

D. among

85.

A. on

B. at

C. in

D. for

Part D. WRITING
I. Choose the best phrase or clause that fits each blank in the following sentences.
86. He looked such a fool with his hair dyed green that I just ........... laughing at him.
A. couldn't help

B. can't help

C. couldn't enjoy

D. didn't stop

87. He found your remarks offensive. You really .............. to him like that.
A. couldn't speak

B. couldn't have spoken

C. can't have spoken

D. oughtn't to have spoken

88. How on earth .............. lied to your best friends?


A. should you have

B. must you have

C. could you have

D. needn't you have

89. Jim's aunt bought him books on astronomy and football, .............. he ha lightest interest in.
A. either of whom

B. neither of which

C. neither of what

D. either of that

90. He tiptoed into the room .............. waken his mother.


A. so as not to

B. in order to

C. so as to

D.in order not to

91. Descriptive analysis of language merely reflects ........... used without concern for the
social prestige of these structures.
A. it how grammar structures and vocabulary is

B. how are grammar structures and vocabulary


C. how grammar structures and vocabulary are
D. it is how grammar structures and vocabulary are
92. Originally, the first European colleges consisted of groups of individuals ........... joined
their efforts to study sciences, medicine, and law.
-ng
-ng

A. who lived together and


B. whose life together and
C. and who lived together
D. whose living together and

.,g

93. Some language experts might say ............ to a person speaking a language one does not
understand and still determine whether the speaker is excited or exhausted, angry, or pleased.
A. that possible it is listen
B. what is possible to listen to it
C. that it is possible to listen
D. whether is it a possibility it is listen
94. This booklet tells you
A. which are travel agencies to avoid
B. which travel agencies to avoid
C. which travel agencies are to avoid
D. the travel agencies which to avoid
95. ............ when I realized I had left my wallet home.
A. Scarcely had I entered the supermarket
B. Scarcely I had entered the supermarket
C. No sooner had I entered the supermarket
D. No sooner I had entered the supermarket
II. Choose the sentence that has similar meaning to the original sentence(s).
96. Julia forgot to do her homework.
A. Julia did her homework but she didn't remember.
B. Julia forgot doing her homework.
C. Julia didn't do her homework as she didn't remember.
D. Somebody did her homework for Julia but she forgot it.
97. We can't deny that all of us made certain mistakes early on.
A. It can be denied that not all of us made mistakes.
B. It is true that nobody could avoid making mistakes.
C. We admit that we could avoid making certain mistakes when young

D. Everyone of us denies that we made certain mistakes early on.


98. "I don't mind at all if use my typewriter. Go ahead"
A. She allowed me to use her typewriter.
B. She said I could use her typewriter if she went away.
C. I was forbidden to use her typewriter, and I should go.
D. She told me to go ahead
99. "Don't tell anyone or you'll be sorry."
A. She said she was sorry as I didn't tell her.
B. She warned me not to tell anyone.
C. I would be sorry if she didn't tell me.
D. She said that I should apologize to her.
100. The agreement ended six-month negotiation. It was signed yesterday.
A. The agreement which ended six-month negotiation was signed yester
B. The agreement which was signed yesterday lasted six months.
C. The negotiation which lasted six months was signed yesterday.
D. The agreement which was signed yesterday ended six-month negotiation.

English objective test 8


Time allowed: 90 Min.
Part A. PHONETICS
I. End the word which is pronounced differently in the part underlined.
1.

A mountain

B. pronounce

C. county

D. poultry

2.

A. deceive

B. perceive

C. ceiling

D. leisure

3.

A. colonise

B. special

C. oversleep

D. aspects

4.

A. won

B. common

C. conceited

D. terror

5.

A. unfortunate

B. nun

C. unanimous

D. unlace

II. Define the word having a different stress pattern in each line.
6.

A. overweight

B. convenient

C. resistant

D. allowance

7.

A. criminal

B. accurate

C. diseases

D. resolute

8.

A. resources

B. televise

C. descendant

D. possession

9.

A. survival

B. scholarship

C. mechanic

D. imagine

10.

A. majority

B. discovery

C. benefactor

D. machinery

Part B. LEXICO - GRAMMAR


I. Choose the right option to complete the following English idioms.
11. as light as a ............
A. lamp

B. sun

C. wind

D. feather

B. earth

C. forests

D. mountains

B. daisy

C. lotus

D. carnation

B. a crab

C. a lobster

D. a shrimp

B. a lime

C. vinegar

D. grapefruit

B. rooster

C. judge

D. lawyer

B. precious

C. rare

D. expensive

B. white

C. pure

D. simple

12. as old as the ..............


A. hills
13. as fresh as a ...............
A. rose
14. as red as ................
A. blood
15. as sour as
A. a lemon
16. as sober as a ................
A. bird
17. as ........... as gold
A. good
18. as ........... as a sheet
A. clean
19. as .......... as a rock

A. strong

B. quiet

C. firm

D. dry

B. strong

C. stiff

D. tough

20. as ............ as nails


A. hard

II. Choose the best option to finish each of the following sentences.
21. No longer .......... to do all her housework with her hands because our family now
owns some new labour saving devices.
A. my mother has

B. has my mother

C. did my mother have D. does my mother have


22. Can you................ your papers with you when you come to see me, please?
A. bring

B. collect

C. take

D. get

23. He gave the listeners a vivid ............ of his journey through Peru.
A. news

B. communication C. account

D. tale

24. She opened the packet and emptied the .......... into a saucepan.
A. fullness

B. contents

C. insides

D. container

25. High interest rates............people from borrowing money.


A. decrease

B. decide

C. disgust

D. discourage

26. The house stood by itself in the middle of a field. It was completely ............
A. lonely

B. isolated

C. unaccompanied D. unsupported

27. The judge .............. the criminal to twenty years in prison.


A. condemned

B. punished

C. inflicted

D. sentenced

28. He was an ........... writer because he persuaded many people to see the truth of his ideas.
A. ordinary

B. influential

C. unlimited

D. accurate

29. Workers who do not obey the safety regulations will be ...... immediately.
A. refused

B. rejected

C. disapproved

D. sacked

30. He had to leave his family ............ when he went abroad to work.
A. at a loss

B. behind

C. out

D. at all costs

31. I went to talk to the manager, he told me he could only ........ me a few minutes.
A. provide

B. spare

C. hear

D. let

32. Can you ............... the BBC World Service on your new radio?
A. put on

B. take in

C. get at

D. pick up

33. A few hours after the injection the feeling of numbness in your arm will........
A. wear off

B. fade out

C. drop away

D. fall through

34. At first the children enjoyed the game but quite soon the novelty ............
A. went off

B. died out

C. died down

D. wore off

35. What you say is true, but you could have ........... it more tactfully.
A_ talked

B. phrased

C. observed

D. remarked Decide

III. Decide the correct word form to be used in each gap in the following sentences.
36. The situation was further complicated by John's ............. .
A. decision

B. undecidedness

C. indecision

D. indeciciveness

C. a handful

D. a handfulness

37. Duong's mother gave him ............. of sweets.


A. a hand

B. hands

38. In his anxiety to make himself ............, he 'spoke too loudly and too slowly.
A. understand

B. understood

C. understanding D. to understand

39. Now that she has got a job, Huong is .............. of her parents.
A. dependent

B. dependable C

independent

D. depending

40. It is quite ............. to ask him again. He'll never agree.


A. point

B. pointing

C. pointed

D. pointless

41. If you behave with this sort of ................ to your customers, I don't think you'll remain in
business long.
A. unpolite

B. impolite

C. impoliteness

D. unpoliteness

42. The speaker showed his ........... by constantly straightening his tie.
A. nerves

B. nervous

C. nervelessness

D. nervousness

43. Is there any difference between egoism and ............?


A. selfish

B. selfishness

C. selfless

D. selflessness

44. My son, now that you're the head of the family, you must take your .......... place at the
head of the table.
A. right

B. rightness

C. rightful

D. right-hand

45. I need a new secretary. I want someone who is charming, efficient and absolutely .........
A. trusting

B. trustful

C. trusted

D. trustworthy

IV. Choose the correct phrase for the gap in each of the sentences.
46. ............... the invention of the steam engine, most forms of transport were horse, drawn.
A. Akin to

B. Prior to

C. In addition to

D. With refer

47. This organization is completely ......... any political affiliation.


A. in case of

B. in memory of

C. instead of

D. independent of

48. Let us hope that.......... a nuclear war, the human race still survive.
A. in relation to

B. with reference to

C. within the realm of

D. in the event of

49. Of course I'll play the piano at the party but I'm a little .............. .
A. out of use

B. out of place

C. out of turn

D. out of practice

50. This is ............... the most difficult job I've ever tackled.
A. by far

B. by all means

C. by the way

D. by rights

Part C. READING
Read the following passage and choose the correct answer for each question.
Unlike buried treasure and some minerals, fossils are usually not valuable in themselves.
However, most of the fossils you find will be of greatest value to you. They will help you
learn much about the geologic history of the earth and its inhabitants.
If it were not for fossils, we would have no way of knowing about the animals that once
roamed the earth or that once crawled on the bottom of the ancient seas. In fact, we would not
even know for sure that there were seas in some regions if it were not for the many fossils of
sea animals that we find there.
You can look at some fossils and see for yourself the shape of the animals. Even if you collect
only bits of one kind of fossil, you may still be able to tell what it looked like. Making sense
out of fossil bits is a little like working a jigsaw puzzle. Sometimes you will lose pieces, but
you can still recognize the picture and even use your imagination to fill in the missing details.
The habits of ancient sea animals are not so easy to do Therefore, scientists study the structure
and the habits of similar living animals to learn what the ancient ones must have been like.
There are some things, however, that neither the study of fossils nor the study of living
animals can tell us. For example, no one knows what colour most ancient sea animals were.
Also, since the soft parts disappeared soon after death, many details of their internal structure
are unknown. So the pictures of ancient sea creatures (as well as dinosaurs!) are what
scientists think the animals looked like. Reconstructing the life of an ancient sea animal on the
basis of fossil evidence and the characteristics of similar living animals is a little like relying
on circumstantial evidence in court.
51. This passage is mainly about animals that.............
A. have very tough hides.

B. once lived on the earth.

C. inhabit the seas of today.

D. are becoming extinct.

52. With the help of fossils we can..............


A. reconstruct life of the past.
B. predict the future.
C. explain the changes of the seasons.
D. save some species of animals from extinction.
53. Many details of ancient animals are lacking because ...........
A. many species have not been found.
B. volcanic disasters have permanently buried many fossils.

C. many fossils lie in deep water and cannot be recovered.


D. soon after death the soft parts of the body decay.
54. To "determine" in the passage is closest in meaning to ..........
A. come across

B. study

C. find out

D. imitate

55. This passage implies that a scientist's evaluation of the past is based on ...........
A. laboratory experiments with live animals.
B. a few facts mixed with a lot of opinions.
C. predictions made in the past and present.
D. field studies of our society.
II. Read the passage and choose the best answer for each question.
THE CRUSADES WIDEN EUROPE'S HORIZONS
The cross-bearers who went on the long journey to the Middle East from France, England,
Germany and Italy knew little of the kind of people they were going to meet there. All they
knew was that they were going to try to take sacred territory from the hands of "the Infidels",
"unbelievers whose God. and Prophet were different from their own. After once mal; with the
East, the crusaders were surprised to find there a highly culture which was not only much
older than Europe's but in many ways quite superior.
Islam and Christianity had many things in common - most importantly, a belief in one God.
The Islamic codes of morality and hospitality deeply impressed the men from the West. At a
time when losses from pestilence called for every bit of medical skill available, doctors
accompanied the crusaders discovered that Moslem medicine was f of their own.
56. The best title for this selection would be .............
A. The Conquest of the Middle East.

B. Education of the Crusaders

C. Crusades of Europe.

D. Islamic Law and Religion.

57. The crusaders were surprised to discover that Moslem culture was ...............
A. less advanced than their own.

B. similar to their own.

C. more advanced than their own.

D. primitive in most respects.

58. The crusaders were called "cross-bearers" because they ..............


A. believed in God.

B. suffered so greatly.

C. were so rude.

D. were Christians.

59. We can conclude from the passage that wounded crusaders were ...........
A. left to die unaided.
B. helped by Moslem medical knowledge.
C. carried home for medical treatment.
D. cured by their accompanying doctors.

60. The word pestilence, as used in this passage, is closest in meaning to ................
A. infection.

B. wound.

C. epidemic disease.

D. insect poison.

III. Choose the best preposition or adverb for each gap in the following passage.
ACUPUNCTURE
Acupuncture is a Chinese medical technique which has been pi more than 4000 years. It
involves inserting long thin needles (61) ............... .particular spots in the skin, known as
acupuncture points, and rotating them. It is mainly used to relieve pain but it is also
sometimes used (62) ............. curing disease and improving general health.
Acupuncture is one of the great mysteries of medical science. There is little doubt that it can
be effective (63) ............. relieving pain. Western doctors have witnessed surgical operations
carried out (64) ............. Chinese patients who were anaesthetized only (65) ...........
acupuncture and yet showed no signs of pain. However, Western scientists have still not come
(66) ................. an adequate explanation as to how acupuncture actually works. At one time it
was believed that acupuncture was related in some way (67) ............ hypnosis, but this has
now been proved not to be true. Still, acupuncture is becoming increasingly popular in the
West, (68) ................ many American and European doctors now believing acupuncture may
have a role to play in medicine, although most argue that much more research needs to be
done first.
In 1972, acupuncture received some welcome publicity as a result of President Nixon's trip
(69) ............... China. Nixon became ill (70) ............ the trip and was rushed to hospital. He
later told reporters that acupuncture had greatly relieved his pain.
61. A. in

B. into

C. through

D. on

62. A. for

B. in

C. by

D. with

63. A. for

B. in

C. with

D. on

64. A. by

B. on

C. with

D. for

65. A. by

B. on

C. with

D. for

66. A. into

B. across

C. up with

D. on

67. A. of

B. into

C. with

D. to

68. A. of

B. for

C. with

D. as

39. A. in

B. to

C. into

D. through

40. A. on

B. during

C. in

D. through

IV. Choose A, B, C, or D for each gap to finish the following passage.


THE ORIGINS OF PHOTOGRAPHY
The world's first surviving photograph was taken in 1827 by a Frenchman called Niepce. Up
to that (71)............., it had been impossible to capture permanently a living image, (72) .........
in a painting or drawing. Niepce pointed his early camera at the window of his country home
and produced an image. It wasn't very (73) ............., and it took him eight hours in bright

sunlight , but the image still survives to this day.


Another Frenchman, Daguerre, heard about Niepce's work and (74) ............him. They became
(75)...........and worked together to create photographic process. This process was very
complicated and (76) .......... a great deal of skill. (77) ............ the difficulties, it became very
popular people around the world were taking daguerreotypes, as they were known In England,
William Henry Fox Talbot (78) ................ developed his own process at about the same time
that Daguerre and Niepce were working on theirs. His method (79) .......... more than one copy
to be made, (80) ............. the daguerreotype could not be reproduced.
This new technology created all kinds of opportunities for (81) ............. brave enough to
travel to remote locations and to go into dangerous situations. Photographers (82) ..........as
Roger Fenton of England and Mathew Brady of America took some of the first war
photographs.
By the 1880s, (83) ........... American George Eastman produced the first Kodak camera, the
world was ready for mass photography. The Kodak camera had a roll of film inside and was
(84) ............ easier to use than any previous camera. It was an instant success and soon people
were (85) .......... picture taken as if it were the most ordinary thing in the world.
71. A. point

B. period

C. occasion

D. mark

72. A. although

B. except

C. despite

D. otherwise

73. A. clean

B. shiny

C. smooth

D. clear

74. A. contacted

B. communicated

C. informed

D. greeted

75. A. companions

B. partners

C. rivals

D. opponents

76. A. claimed

B. demanded

C. asked

D. mention

77. A. In spite

B. Though

C. However

D. Despite

78. A. had

B. was

C. did

D. got

79. A. made

B. allowed

C. let

D. produced

80. A. yet

B. even

C. whereas

D. otherwise

81. A. those

B. some

C. they

D. ones

82. A. even

B. so

C. just

D. such

83. A. where

B. when

C. which

D. while

84. A. considerably

B. extremely

C. absolutely

D. completely

85. A. being

B. having

C. doing

D. making

Part D. WRITING
I. Choose A, B, C, or D to show the best way of making A. sentence from the words and
phrases given.

86. Remains/ ancient cities/ popular/ modern tourists.


A. The remains of ancient cities are popular to modern tourists.
B. The remains of ancient cities are popular with modern tourists.
C. The remains of ancient cities were popular to modern tourists.
D. The remains of ancient cities were popular with modern tourists.
87. Ruins/ Pompeii/ Italy/tell us/ much/ life/first century AD.
A. The ruins of Pompeii in Italy tell us much about life in the first century AD.
B. The ruins of Pompeii in Italy tell us about much life in the first century AD.
C. The ruins in Pompeii of Italy told us about much life in the first century AD.
D. The ruins in Pompeii of Italy told us much about life in the first century AD.
88. Inhabitants/ this city/ all/ kill/ one week/ volcanic disaster.
A. The inhabitants of this city all were killed after one week in volcanic disaster.
B. The inhabitants of this city all were killed after one week in a volcanic disaster.
C. The inhabitants of this city were all killed in one week in a volcanic disaster.
D. The inhabitants of this city were all killed within one week in a volcanic disaster.
89. Over/ past/ hundred years/ volcanic dust/ ash/ Pompeii/ dig away.
A. Over the past hundred years the volcanic dust and ash of Pompeii have been dug away.
B. Over the past hundred years the volcanic dust and ash of Pompeii were dug away.
C. Over the past hundred years the volcanic dust and ash of Pompeii have dug away.
D. Over the past hundred years the volcanic dust and ash of Pompeii have been digging
away.
90. And we/ see/ people/ Pompeii/ as/ they/ be/ day/ die.
A. And we saw the people of Pompeii as they were on the day
B. And we can see the people of Pompeii as they were the day the,
C. And we see the people of Pompeii as they were on the day they
D. And we saw the people of Pompeii as they were on the day the:
II. Choose A, B, C, or D to complete the following sentences.
91. Should I find your camera .............. .
A. I will bring it round to your house
B. I could bring it round to your house
C. I would have brought it round to your house
D. I brought it round to your house
92. Were we to buy a computer ............ .
A. I could have learnt something about the Internet
B. we would have sent more e-mails

C. we would be able to write e-mails to friends


D. I'll write e-mails to my friends
93. Had I known how useful mobile phones were ............. .
A. I had got one a long time ago
B. I would have got one a long time ago
C. I'd like to get one as soon as possible
D. I let you get one a long time ago
94. My mum doesn't visit websites, even ..........
A. though I know I'll use them in the future
B. if I find ones that I think might interest her.
C. so, she doesn't know the first thing about them
D. when she doesn't have to use them in her work
95. Technology will continue to develop
A. although we could find ourselves facing serious problems
B. despite we want it or not
C. unless we enable them to
D. whether we think that is a good thing for our society or not
III. the sentence A, B, C, or D which is closest in meaning to the sign.
96. Please inform the College Office of any change of address immediately
A. The College has moved to a new building.
B. If you move house, tell the College Office.
C. Tell the College Office your home address when you first arrive.
D. Ask at the College Office if you want to change your room.
97.

Standing room only

A. There is no more room.


B. There are no more seats.
C. There is room for one more person.
D. There is only one more room.
98.

Lasts up to one week in


fridge
Unsuitable for freezing

A. Don't keep this food longer than a week.


B. Don't put this food in the fridge.

C. This food will last longer in the freezer.


D. This food will last over a week if frozen.
99.

Please keep this doorway clear

A. Don't lock this door.


B. Please wait here until the door is opened.
C. Please don't stand here.
D. Please use the other door.
100.

This hospital has no emergency department

A. Accident patients are not admitted here.


B. The emergency department is closed.
C. This hospital only accepts accident patients.
D. Contact this hospital in an emergency.
English objective test 9
Time allowed: 90 min.
Part A: PHONETICS
I. Choose the word whose underlined part is pronounced differently from the rest in the
same line.
1. A. literature

B. apprehension

C. adventure

D. facility

2. A. recommend

B. recollect

C. reclaim

D. recognition

3. A. preferable

B. derivation

C. preparation

D. preliminary

4. A. envelopes

B. communicates

C. headaches

D. judges

5. A. hatched

B. learned (adj)

C. dedicated

D. needed

II. Choose the word whose prominent stress is different from the others in the same line.
6. A. computer

B. imagine

C. property

D. horizon

7. A. electric

B. luxury

C. architect

D. mineral

8. A. arithmetic

B. assassinate

C. agriculture

D. contributor

9. A. preservation

B. potentially

C. perversity

D. magnificent

10. A. trigonometry

B. dramatically

C. photochemical

D. documentary

Part B: LEXICO -GRAMMAR


I. Choose the answer among A, B, C or D which best fills the blank in following
sentences.
11. In the past, most people believed that the world's resources could never be used .......... .
A. up

B. all

C. whole

12. I do not think that purple shirt .......... with your yellow skirt.

D. at all

A. suits

B. fits

C. goes

D. wears

13. The main road through Salisbury was blocked for 2 hours today after an accident ..........
several vehicles.
A. containing

B. connecting

C. involving

D. including

14. The company directors asked the government to ......... in the dispute and prevent a strike.
A. intervene

B. interact

C. intercept

D. interpose

15. After the campaign a special medal was ............ to all combatants.
A. gained

B. awarded

C. earned

D. deserved

16. The usual reason for exemption from tax does not ................. in this case.
A. apply

B. impose

C. regard

D. concern

17. We don't sell foreign newspapers because there is no ........... for them.
A. request

B. claim

C. requirement

D. demand

18. In the legal profession, men ............. women by 5 to 1.


A. outnumber

B. supersede

C. overcome

D. outclass

19. must take this watch to be repaired as it ............. over 20 minutes a day.
A. increases

B. progresses

C. accelerates

D. gains

20. That was absolutely delicious. Can you give me the ..........?
A. formula

B. instruction

C. prescription

D. recipe

II. Choose the correct word form among A, B, C or D to fill each of the blanks to finish
the following passage.
Every weekend in Britain thousands of walkers head off into the countryside with the
necessary (21) .............. to enjoy their hiking. One thing they will not leave behind because it
is considered an absolute (22) ............. is their Ordnance Survey Map. These maps are
(23) ............. to the walkers as they are very (24) ............. with up-to-date details about every
part of the country. The Ordnance Survey (25) ............. in the eighteenth century and was (26)
............. intended to provide the army of the time with maps. Before this most maps were (27)
............. and unreliable. Better maps were thought to be important because the country felt
(28) ............. by invasion from abroad or (29) ............. at home. The organisation is still
wholly owned by the Government, but it was recently given (30) ............. fund status, which
allows it to reinvest any profits.
21. A. equip

B. equipment

C. equipped

D. equipments

22. A. necessity

B. necessary

C. necessitate

D. necessarily

23. A. valueless

B. valuable

C. value

D. unvaluable

24. A. informant

B. information

C. informative

D. inform

25. A. beginning

B. began

C. begin

D. begun

26. A. originate

B. origin

C. originally

D. originates

27. A. accurate

B. inaccurate

C. accuracy

D. unaccurate

28. A. threatening

B. threat

C. threatened

D. threaten

29. A. rebelling

B. rebellion

C. rebel

D. rebels

30. A. trader

B. trade

C. traders

D. trading

III. Choose the most suitable preposition among A, B, C or D for each gap to complete
the passage.
If you look at the sky (31) .......... a clear night, you will see that there are so many twinkling
lights that nobody is able to count them all. Most of these lights are stars. They are (32) ..........
big moving balls in the sky. The stars we see (33) .......... night are much farther away
(34) .......... our sun, which is also a star. Some of these stars are like our earth, they arc
(35) .......... More solid material. They are called planets. They shine just as a mirror does
when you hold it (36) .......... the sunlight. Instead (37) .......... giving (38) .......... the light of
their own, these planets reflect the light (39) .......... our sun. The other planets of the sun are
much closer (40) .......... our planet earth than the stars are. However, these other planets are
very far away. It would take a travelling more than 60,000 miles an hour about five months to
reach the nearest one.
31. A. on

B. at

C. in

D. from

32. A. as

B. like

C. above

D. under

33. A. on

B. at

C. beside

D. be yond

34. A. between

B. of

C. from

D. ne xt

35. A. from

B. by

C. of

D. in

36. A. to

B. against

C. opposite

D. across

37. A. along

B. from

C. with

D. of

38. A. up

B. back

C. out

D. in

39. A. from

B. of

C. with

D. by

40. A. to

B. away

C. from

D. within

IV. Choose the correct verb form among A, B, C or D for each numbered gap to
complete the passage.
I (41) ............. up while flight attendant was serving breakfast. The plane was, crossing the
Channel, and when I looked out of the window, I could see ships far below. I (42)............
home after a long time abroad, I had been away for over years. I didn't know what to expect. I
wondered whether things (43) ............ much.
The plane landed and the bus took us to the terminal building. After (44) ............
my luggage, I walked out of the baggage hall. I didn't know whether anyone there to meet me.

But when I appeared, I had a big surprise. half my family were there. It was wonderful.
(46)............ the old town," my sister told me. "Almost everything has changed since you last
(47) ............ it. They are building a new bridge at the moment. They (48) .......... it for the past
three years, but they (49) ............ it yet." I've been back for nearly a month now. And my sister
was right. The place very different, but strangely enough it still (50) ............ the same,
e the people that I love are here. I'm leaving again in a few weeks' time. but I know that even
if I'm away for twenty years again, this will always be my home.
41. A. waked

B. woke

C. awake

D. awaken

42. A. come

B. am coming

C. was coming

D. have been coming

43. A. had changed

B. had been changed

C. have changed

D. have been changed

44. A. collected

B. collecting

C. having collected

D. collect

45. A. is

B. are

C. were

D. would be

46. A. don't recognise

B. didn't recognise

C. won't recognise

D. wouldn't recognise

47. A. see

B. seen

C. have seen

48. A. build

B. build

C. have been building

49. A. haven't finished

D. saw
D. have built

B. hasn't finished

C. haven't been finishing


50. A. feel

D. hasn't been finished

B. feeling

C. felt

D. feels

V. In each line below, four words or phrases have been underlined. Choose the one word
or phrase (A, B, C or D) that would not be accepted in standard English.
51. Even though the girls have all ready visited St. Augustine, they want to return
A

to the Castillo de San Marcos.


52. The refugees are very upset because they have been deprived to their horn and their
families.
A

53. If it had not been for the computerized register tape from the grocery store, I never have
A

been able to figure on my expenditures.


D
54. Our new television came with a ninety-days warranty on all electrical, components.
A

55. It is difficult to get used to sleep in a tent after having a soft, comfortable to lie on.

56. Tim hopes Go skiing in the mountains this weekend if the weather permits
A

57. The political candidate talked as if he has already been elected to the presidency.
A

58. The teacher tried to make the classes enjoyable experiences for the students so they would
A

take a greater interest in the subject.


D
59. The streets are wet. There must be a heavy rain last night.
A

60. John said he will invited some of his friends to dinner so Mary had to buy some more food
A

and drink.
Part C. READING
I. Read the following passage and answer the questions that follow by choosing the best
option among A, B, C or D.
The forest from which Man takes his timber is the tallest and impressive plant community on
Earth. In terms of Man's brief, it appears permanent and unchanging, save for the seasonal
growth and fall of he leaves, but to the forester, it represents the climax of a long succession
of events.
No wooded landscape we see today has been forest for all time. Plants have minimum
requirements of temperature and moisture and, in ages past, virtually every part of Earth's
surface has, at some time, been either too dry or too cold for plants to survive.
However, as soon as climatic conditions change in favour of plant life, a fascinating sequence
of changes, called a primary succession, occurs first to colonize the barren land and the lichen
surviving on bare rock. Slowly, the acids produced by these organisms crack the rock's
surface, plants debris accumulate and mosses establish shallow root-holes. Ferns may follow
and, with short grasses and shrubs, gradually form a covering of plant life. Roots : Ace even
deeper into the developing soil and eventually large shrubs give way to the first trees. These
grow rapidly, cutting off sunlight from the smaller plants, and soon establish complete
domination - closing their ranks forming a climax community which may endure for
thousands of years. Yet even this community is not everlasting. Fire may destroy it outright
and settlers may cut it down to gain land for pasture or cultivations. If the land is men
abandoned, a secondary succession will take over, developing much faster on the more

hospitable soil. Shrubs and trees are among the early invaders, their seeds carried by the wind,
by birds and lodged in the coat of mammals.
For as long as it stands and strives, the forest is a vast machine storing energy and the many
elements essential for life.
61. Why do we tend to think the forest as permanent?
A. Because the trees are so tall.
B. Because it is renewed each season.
C. Because our lives are comparatively short.
D. Because it is an essential part of our lives.
62. What has sometimes caused plants to die out in the past?
A. Interference from foresters.
B. Variations in climate.
C. The absence of wooded land.
D. The introduction of new types of plants.
63. In a "primary succession", what makes it possible for mosses to take root?
A. The type of rock.

B. The amount of sunlight.

C. The amount of moisture

D. The effect of lichens.

64. What conditions are needed for shrubs to become established?


A. Ferns must take root.
B. The ground must be covered with grass.
C. More soil must accumulate.
D. Smaller plants must die out.
65. Why is a "secondary succession" quicker?
A. The ground is more suitable.
B. There's more space for new plants.
C. Birds and animals bring new seeds.
D. It is supported by the forest.
II. Choose the best option among A, B, C or D which best fills each of the blanks.
It is true that there have been great (66)...........in technology over the last forty years. For
examples, the uses of mobile phones and email communication are common these days.
(67) .........., machines could never do as good a job as a human, especially when it (68) ..........
to interpreting what people arc (69) ........... Of course, machines can translate plain
statements such as "Where is the bank?" but even simple statements are not all because
meaning depends on more than just (71) .......... For instance, the word "bank" has a
(72) .......... of different meanings in English. How does a translating machine know which
meaning to take?
How For ins In order to understand what people arc saying, you need to take into ac the

(73).......... between speakers and their situation. A machine cannot (74) .......... the difference
between the English expression "Look out" meaning "Be careful!" and "Look out" meaning
"Put your head out of the window". You need a human being to (75) .......... the situation.
(76) .......... with written language, it is difficult for a machine to know how to translate
(77).......... because we rarely translate every word (78) .......... the contrary, we try to take into
consideration how the idea would be (79).......... in the other language. This is hard to do
because every language has its own way of (80).......... and saying this.
66. A. advances

B. steps

C. progress

D. advantages

67. A.. Moreover

B. In addition

C. However

D. Besides

68. A. arrives

B. goes

C. comes

D. reaches

69. A_ talking,

B. saying

C. speaking

D. telling

70. A. obvious

B. direct

C. straight

D. straightforward

71. A. grammar

B. vocabulary

C. words

D. sentences

72. A. number

B. variety

C. plenty

D. couple

73. A. relations

B. relationship

C. attitude -

D. action

74. A. say

B. speak

C. talk

D. tell

75. A. interpret

B. understand

C. think

D. believe

76. A. However

B. In contrast

C. Similarly

D. In addition

77. A. exactly

B. righteously

C. rightfully

D. accurately

78. A. With

B. For

C. On

D. By

79. A. translated

B. expressed

C. spoken

D. said

80. A. doing

B. working

C. making

D. creating

Part D. WRITING
I. Choose the phrase or clause A, B, C or D that best completes each sentence.
81. I have never seen ......... before.
A. such good film

B. so good film

C. so good a film

D. such good a film

82. Only when I saw the photograph.........he was your brother.


A. that I realised

B. did I realise

C. that I realise

D. do I realise

83. ..........your help, I wouldn't have been successful.


A. Had not it been for

B. Were not it for

C. Had it not been for

D. Were it not for

84. It is a biological fact that children............. their parents.


A. take up

B. take to

C. take after

D. take out

85. Andrew suggested........... a class re-union.


A. to his ex-classmates that they should have
B. his ex-classmates to have
D. his ex-classmates that they should have
C. that his classmates had
86. I understand that the governor is considering a new proposal
A. what would eliminate unnecessary writing in government
A.who wants to cut down on the amount of writing in government
B. to cause that the amount of papers written in government offices reduced
C. that would eliminate unnecessary paperwork in government

87. Jane changed her major .from French to business,


A. with hopes to be able easier to locate employment
A.hoping she can easier get a job
B. hoping to find a job more easily
B. with the hope for being able to find better a job

88. The doctor insisted that his patient


A. that he not work too hard for three months
A.to take some vacations for three months
B. taking it easy inside of three months
B. take it easy for three months

89. Richard was asked to withdraw, .ft.om graduate school because


A. they believed he was not able to complete research
B. his ability to finish the research was not believed or trusted
C. it was decided that he was not capable to complete the research
D. he was deemed incapable of completing his research

90. The committee members resented


A. the president that he hid not tell them about the meeting
A.the president not to inform them of the meeting
B. the president's not informing them of the meeting
C. that the president had failed informing themselves that there was be a meeting

II. Choose the sentence among A, B, C or D that is closest in meaning original sentence.
91. Criteria like language or tribe can become the basis for disintegration.
A. Political disintegration can be based on criteria like language or tribe.
B. Language or tribe are criteria of political disintegration.
C. Political disintegration are basic to criteria like language or tribe.

D. Criteria of political disintegration are language or tribe.


92. Language group conflicts may persist beyond the situation which gave rise them.
A. Language group conflicts may persist beyond the situation from where they originated.
B. Language group conflicts may persist beyond the situation from which they originated.
C. Language group conflicts may persist beyond the situation which they originated.
D. Language group conflicts may persist beyond the situation they originated.
93. It is not certain that John will get the job.
A. John is not open to whether he will get the job or not.
B. It is not open to the question whether John will get the job or not.
C. It is not open to question whether John will get the job or not.
D. It is not open for John to get the job
94. Waiting for buses irritates me.
A.I have nerves waiting for buses.
B. Waiting for buses nerves me.
C. Waiting for buses gets into my nerves.
D.Waiting for buses gets on my nerves.

95. "Nothing will persuade me to sleep in that haunted house," she said
A.She denied sleeping in that haunted house.
B. She flatly refused to sleep in that haunted house.
C. She refused to sleep in that haunted house.
D.She denied having slept in that haunted house.

96. We regret to inform you that your application hasn't been successful.
A. Much to our regret, we have to inform you that your application hasn't been successful.
A. Much from our regret, we have to inform you that your application hasn't been successful.
C. Much of regret, we have to inform you that your application hasn successful.
D. Much with our regret, we have to inform you that your application been successful

97. The demand was so great that they had to reprint the book immediately.
A. So was the demand for the book that they had to reprint it immediate
B. Such was the demand for the book that they had to reprint it immediate
A. So great the demand was for the book that they had to reprint immediately.
B. Such great was the demand for the book that they had to reprint immediately.

98. You can't expect me to pay for the ticket.


A. There's no question of my paying for the tickets.
B. There's no question about my paying for the tickets.
C. I can't expect to pay for the ticket.

D. I can't expect you to pay for the ticket.

99. Some people .simply can't remember historical dates.


A. Some people find themselves hard to remember historical dates.
B. Some people find themselves hard remembering historical dates.
C. Some people find themselves incapable of remembering historical
D. Some people find themselves incapable in remembering historical

100. You won't find a more dedicated worker than Mrs. Jones anywhere.
A. Nowhere will you find a more dedicated worker than Mrs. Jones.
B. Nowhere will you find a more dedicated worker like Mrs. Jones.
C. Nowhere won't you find a more dedicated worker than Mrs. Jones.
D. Nowhere you will find a more dedicated worker than Mrs. Jones.

English objective test 10


Time allowed: 90 min.
Part A. PHONETICS
I. Choose the word whose underlined part is pronounced differently from the rest in the
same line.
1. A. procedure

B. soldier

C. education

D. credit

2. A. architecture

B. charity

C. christen

D. cholera

3. A. earflaps

B. attached

C. delicate

D. astrology

4. A. crabbed

B. aged (n)

C. crooked

D. raised

5. A. reality

B. gravity

C. abnormality

D. responsibility

II. Choose the word whose prominent stress is different from the others in the same line.
6. A. career

B. avoid

C. manage

D. advise

7. A. burglary

B. employment

C. courageous

D. dependent

8. A. computer

B. expression

C. instantly

D. appliance

9. A. elementary

B. continental

C. beneficial

D. dormitory

10. A. curriculum

B. enthusiast

C. subordinate

D. democratic

Part B. LEXICO - GRAMMAR


I. Choose the answer among A, B, C or D which best fills the blank in each of following
sentences
11. The coins are ............ to be over a thousand years old.
A. described

B. said

C. told

D. mentioned

12. A large number of students in this school ............ English fluently.


A. speak

B. speaks

C. is speaking

D. has spoken

13. Tony's boss doesn't want him to ............ a habit of using the office phone for personal calls.
A. make

B. do

C. have

D. increase

14. You can't enter this camp without ............ from the General.
A. a control

B. a demand

C. a permit

D. an allowance

15. Buy the new ............ of soap now on sale; it is softer than all others.

A. model

B. brand

C. mark

D. manufacture

16. The forecast predicted ............ weather with snow, sunshine, wind and thunder and that is

just what we have had.


A. differing

B. various

C. variable

D. fluctuating

17. That's a nice coat, and the colour ............ you well.

A. fits

B. matches

C. suits

D. couples

18. I can no longer afford the cost of ............ two cars, so I am selling one.

A. managing

B. running

C. controlling

D. operating

19. You should ............ a lawyer before you sign the document.
A. consult
20.

B. counsel

C. check

D. communicate

He could find a teaching........ in Vietnam without difficulty.


A. work

B. occupation

C. post

D. employment

II. Choose the correct option among A, B, C or D for each numbered gag to complete the
sentences.
21. At first I didn't recognize her because she ........... at least fifty pounds.
A. lost

B. might lose

C. had lost

D. would lose

22. Eva's got so.............to opera that it has become a constant companion of her life.
A. used to listening
B. used to listen
C. used in listening

D. use to listen

23. Had she wanted company, she............John's advice and called at her sister's
A. would take

B. would have taken

C. had taken

D. had had taken

24. By this time next year Peter ............ his master degree.
A. has taken
C. will have taken

B. will take
D. will be taking

25. The old lady scarcely cares for anything, ............


A. does she

B. doesn't she

C. is it

D. is she

26. The English ............. more tea than Americans do.


A. has drunk

B. drinks

C. drank

D. drink

27. I'll return Nancy's dictionary to her the next time I ............her.
A. will see

B. see

C. will have seen

D. have seen

28. In to or three months, he.......... an advance on his book and be able to put money down.
A. have

B. will have

C. have had

D. is having

29. To take off her boots or to put them on was an agony to her, but it .......... an agony for
years.
A. was

B. were

C. had been

D. has been

30. I passed the test. I .......... it without your help.


A. would not pass

B. wouldn't have passed

B. didn't pass

D. had not passed

III. Choose the correct word form among A, B, C or D for each numbered gap to complete
the passage.
Linda Devere Hardy, who died last week, was a famous woman. In the 1920s she was (31)
.......... to learn to fly, and became the first woman to become u fully (32).......... pilot. Her solo

(33)..........from London to Delhil was one of the greatest achievements of the early days of
aviation. Her (34) .......... to Angus Hardy took place after the end of the Second World War.
With his (35) .........., she wrote a number of (36).......... stories in the 1950s, which achieved
great (37).......... In 1962, her husband's tragic (38).......... in a car crash ended her (39)..........
career, and she spent the rest of her life on an (40).......... island off the coast of Scotland.
31. A. determined
B. determine
C. determining
D. determiner
32. A. qualifying

B. quality

C. qualified

D. qualify

33. A. flying

B. flew

C. flight

D. flown

34. A. marrying

B. marriage

C. marry

D. married

35. A. courage

B. encourage

C. courageous

D. encouragement

36. A. detective

B. detecting

C. detect

D. detectors

37. A. popularize

B. popularity

C. popular

D. popularization

38. A. die

B. dead

C. death

D. dying

39. A. literature

B. literary

C. literal

D. literacy

40. A. isolated

B. isolate

C. isolation

D. isolating

IV. Choose the best idiomatic expression among A, B, C or D to finish the following
sentences.
41. I am under the weather today. Under the weather means ..............
A. sick.

B. happy.

C. sad.

D. disappointed.

42. If your visits to a doctor are few and far between, it means
A. you get used to receiving medical treatment.
B. you do not get used to receiving medical treatment.
C. you like to visit a doctor.
D. you don't like to visit a doctor.

43. Everybody wished them to live happily for good. For good means ..............
A. in the end.

B. forever.

C. continuously.

D. temporatily

44. To strike while the iron is hot means ..............


A. to attack.

B. to decide.

C. to hurry.

D. to act a

45. She was racking her brains to understand me. Racking one's brains mea ..............
A. trying hard.

B. having a headache.

C. being eager.

D. being too nervous.

46. The skeleton gave the impression that it was about to leap out at n about to leap means
A. was on the point of leaping.

B. was concerned with leaping

C. was thinking of leaping.

D. was almost leaping.

47. You are preparing a meal and keeping an eye on the baby at the sari. Keeping an eye on
means ..............

A. looking at.

B. seeing.

C. watching.

D. seeing

48. All at once, I noticed that my wife seemed to be filled with alarm once means ..............
A. suddenly.

B. immediately.

C. instantly.

D. momentarily

49. To be in a good mood means ..............


A. to be good-looking.

B. to be happy.

C. to be hot-tempered.

D. to be quick-witted.

50. Carla always takes her time on the phone. She really enjoys calling friends and going on
and on about work. To take one's time means ..............
A. to save time.

B. not to hurry.

C. to consume time.

D. to waste time.

51. If someone asks you why you did something and you can't thin reason, you might respond
"Search me". Search me means .............. .
A. look for me.

B. I don't know.

C. watch out for me.

D. I don't care.

52. the firm is in the red, it means that ..............


A. it owes a lot of money.

B. it becomes successful.

C. it is doing well.

D. it is in need of working labour

53. The detective left no stone unturned in his search for the missing jewels. It means ..........
A. he checked everywhere.

B. he left all stones.

C. he turned all stones.

D. he didn't check carefully.

54. Since it is to be a surprise, don't let the cat out of the bag. Let the cat out of the bag
means ..........
A. let the bag away from the cat.

B. not reveal the secret.

C. keep the cat out of the bag.

D. give away the secret.

55. with butterflies in my stomach, I waited for my interview. With butterflies in one's
stomach means .............
A. nervously.

B. patiently.

C. happily.

D. courageously.

V. In each line below, four words or phrases have been underlined. Choose the one or
phrase (A, B, C or D) that would not be accepted in standard English.
56. decided to buy in the morning a new car, but in the afternoon he changed his mind.
A

57. Some of the plants in this store require very little care, but this one needs
A

much more sunlight than the others ones.


58. After George had returned to his house. he was reading a book.

59. We insist on you leaving the meeting before any further outbursts take place.
A

60. It has been a long time since we have talked to John, isn't it ?
A

61. Henry objects to our buying this house without the approval of our attorney, and John
A

does so.
D
62. The children were playing outdoors when it began rain very hard.
1

63. People respected George Washington because he was a honest man. and he turned
out
A

to be one of their talented leaders.


D
64. Food prices have raised so rapidly in the past few months that some families have
A

been forced to alter their eating habits.


D
65. The company has so little money that it can't hardly operate anymore
A

Part C. READING
I. Read the following passage and answer the questions that follow by el the best option
among A, B, C or D.
According to the best evidence gathered by space probes and astronomers, Mars is an
inhospitable planet; more similar to Earth's moon than to Earth itself It is frozen solid,
blanketed by the mere wisp of an atmosphere - a dry, stark, seemingly lifeless world.
They have begun to seriously investigate the possibility of transforming Mars into a
more Earth-like planet and animal life from Earth, and eventually establishing
permanent human colonies. "It was once thought to be so far in the future as to be
irrelevant," said Christopher McKay, a research scientist at the National Aeronautics
and Space Administration. "But now it is starting to look practical. We could do it in
four of five decades."

The idea of "terra-forming" Mars, as enthusiasts call it, has its roots in science fiction.
But as researchers develop a more profound understanding of how Earth's ecology
supports life, they have begun to see that it may be possible to create similar conditions
on Mars.
66. With which of the following is the passage primarily concerned?
A. The possibility of changing the Martian environment.
B. The challenge of interplanetary travel.
C. The advantage of establishing colonies on Mars.
D. The need to study the Martian ecology.
67. Which of the following does the author NOT give as a characteristic of the planet Mars?
A. It is cold and dry.
B. It is quite similar to Earth' moon.
C. It apparently has no life.
D. It has a rich atmosphere.
68. According to Christopher McKay, the possibility of transforming Mars ...........
A. could only occur in science fiction stories.
B. will take place in the next few years.
C. is completely impractical.
D. could be accomplished in 40 to 50 years.
69. As used in the passage, the term "terra-forming" refers to ..............
A. a process for adapting plants to live on Mars.
B. a means of transporting materials through space.
C. a method of building housing for colonists on Mars.
D. a system of creating Earth-like conditions on other planets.
70. According to the article, the basic knowledge needed to transform Mars conies
from...........
A. the science of astronomy.
B. a knowledge of Earth's ecology.
C. data from space probes.
D. science fiction stories.
II. Choose the best option among A, B, C or D which best fills each of the blanks.
PLACES
When I first arrived here to take up my new job, I stayed in a hotel, but I soon started looking
for some permanent (71) ............... The first flat (72) ............... over was in (73) ............... and
was obviously' extremely damp in winter. Quite apart from the fact that the only- (74)

............... was of a brick wall. Then I had a look at a small flat in a modern (75) ............... It
had a (76) ............... space and a garden, but the (77) ............... was far too high for me. I
didn't

want

to

(78)

...............

up

in a tiny place, so I answered an ad for house-sharing. The house was in a quiet (79)..............,
and as soon as I saw it I fell in love with it. There was a high overgrown (80) ...............around
the front garden, and (81) ............... to park cars in the drive. The room to (82) ...............
looked out (83) ............... the back garden, and had a big bay window. Although it meant
(84)............... the kitchen and living room, I did have my own bathroom, really just a shower
and washbasin (85)...............what must have once been a cupboard.
71. A. home

B. accommodation C. house

D. landlords

72. A. passed

B. viewed

C. came

D. looked

73. A. an attic

B. a basement

C. a cave

D. a beds

74. A. view

B. entrance

C. distance

D. bathroom

75. A. tower

B. department

C. block

D. square

76. A. living

B. breathing

C. working

D. parking

77. A. lift

B. rent

C. area

D. roof

78. A. end

B. live

C. shut

D. pay

79. A. surroundings

B. context

C. neighbourhood

D. premise

80. A. fence

B. bush

C. hedge

D. lawn

81. A. room

B. permission

C. areas

D. place

82. A. luckily

B. myself

C. pay

D. let

83. A. in

B. at

C. over

D. for

74. A. sharing

B. in

C. without

D. having

75. A. crammed

B. crowded

C. cluttered

D. poured

Part D. WRITING
I. Choose the phrase or clause A, B, C or D that best completes each sentence.
86. ..........did Jerome accept the job.
A. Only because it was interesting work
B. Because it was interesting work
C. Only because it was interested work
D. The work was interesting
87. .......... were slowly lowered to the ground for medical attention.
A. The victims who were screaming and who were burning
B. The screaming burn victims
C. The screamed burnt victims
D. The victims who were burning screamed
88. This car has many features including............

A. stereo, safety devices, air condition, and it saves gas


B. good music, safe devices, air conditioning, and gas
C. stereo, safety devices, air conditioned, and good gas
D. stereo, safety devices, air conditioning, and low gas mileage
89. The proposal was tabled........... that it would be helpful.
A. temporarily because there was not sufficient evidence
B. because for the time being there were not sufficient evidence
C. because at the present time there was not sufficient evidence
D. temporarily because there was not sufficient enough evidence
90. Adams was dismissed from his position ...........
A. because his financial records were improperly
B. because finance he kept poor records
C.. for keeping improper financial records
D. for keep financial records that were improper
II. Choose the sentence among A, B, C or D that is closest in meaning to the original
sentence.
91. The police think he was one of the robbers.
A. The police suspect him of taking part in the robbery.
B. The police suspect him to take part in the robbery.
C. The police suspect him of taking place in the robbery.
D. The police suspect him of playing part in the robbery.
92. I can very nearly remember the word.
A. The word is on the tip of my tongue.
B. The word is in the tip of my tongue.
C. The word is under the tip of my tongue.
D. The word is near the tip of my tongue.
93. A lot of things keep stopping me from working.
A. A lot of things keep getting on the way of my work.
B. A lot of things keep getting in the way of my work.
C. A lot of things keep to get in the way of my work.
D. A lot of things keep to get in the way of my work.
94. People say that he didn't do the work carefully.
A. The work is said to be done carelessly by him.
B. The work was said to have been done carelessly by him.
C. The work was said to be done carelessly by him.

D. The work is said to have been done carelessly by him.


95. It is certain that Paul knows the answer.
A. There is no doubt at all that Paul knows the answer.
B. There is no doubt at all about Paul knows the answer.
C. It is no doubt at all that Paul knows the answer.
D. It is no doubt at all about Paul knows the answer.
96. The rate of inflation has fallen steadily during recent months.
A. There has been a steady decline in the rate of inflation during recent
B. There is a steady decline in the rate of inflation during recent mom
C. There has been a steady decline of the rate of inflation during recent
D. There is a steady decline of the rate of inflation during recent months
97. We had better leave him a note because it is possible they will arrive laz
A. We had better leave him a note in case they may arrive late.
B. We had better leave him a note in case they will arrive late.
C. We had better leave him a note in case they are likely to arrive late.
D. We had better leave him a note in case they arrive late.
98. It might be better if that paragraph was omitted.
A. It might be better to leave that paragraph.
B. It might be better to leave out that paragraph.
C. It might be better to leave off that paragraph
D. It might be better to leave away that paragraph.
99. When he was forty, he finally got the job he wanted.
A. It is not until he was forty that he got the job he wanted.
B. It was not until he was forty when he got the job he wanted.
C. It was not until he was forty that he got the job he wanted.
D. It was not until he was forty when he had got the job he wanted.
100. This passport is valid until December 151h 2009.
A. The expiry date in this passport is December 15th 2009.
B. The expiry date at this passport is December 15th 2009.
C. This passport expires in December 15th 2009.
D. This passport expires on December 15th 2009.

English objective test 11


Time allowed: 90 min
Part A: PHONETICS
I. Choose the word whose underlined part is pronounced differently from the rest in the
same line.
1. A. drought

B. daughter

C. laughter

D. caught

2. A. applicant

B. abbey

C. abandon

D. happy

3. A brochure

B. champagne

C. chairman

D. chalet

4. A. distribute

B. triangle

C. contribute

D. tributary

5. A. houses

B. crises

C. oases

D. analyses

II. Choose the word whose prominent stress is different from the others in the same line.
6. A. gravity

B. professor

C. pyramid

D. colorful

7. A. industrial

B. exhibition

C. preparation

D. decoration

8. A. luxurious

B. energetic

C. adventurous

D. congratulate

9. A evaporate

B. temperature

C. impossible

D. experiment

10. A. brilliant

B. insect

C. lunar

D. proceed

Part B: LEXICO - GRAMMAR


I. Choose the answer among A, B, C or D which best fills the blank in each of the
following sentences.
11. If you deposit money in the bank, you get about 8% ............... .
A. increase

B. interest

C. rent

D. loan

12. Which of the following is not right?


A. a dozen eggs

B. a steak of meat

D. a bunch of bananas

C. a box of matches

13. ............. the committee, I'd like to thank you for your generous donation.
A. According to

B. On behalf of

C. In addition to

D. Thanks to

14. I saw a very good.............advertised in the paper yesterday.


A. job

B. work

C. occupation

D. career

15. She often wears a ring, a necklace and other .........


A. figures

B. documents

C. monuments

D. ornament

16. Which of the following things would help you see distant objects clearly?
A. spectacles

B. a microscope C. a telegraph

D. binocular:

17. The .......... in the north of Sweden is really beautiful.


A. scenery

B. nature

18. He was ............. with robbery.

C. view

D. scene

A. imprisoned

B. charged

C. arrested

D. accused

19. My uncle was ............. ill last summer but fortunately, he is now making a but steady
recovery.
A. critically

B. deeply

C. fatally

D. seriously

20. When a fire broke out in the Louvre, at least twenty .......... paintings destroyed, including
two by Picasso.
A. worthless
II.

B. priceless

C. valueless

D. meaningless.

Choose the best phrasal verb among A, B, C or D to finish the f sentences.


21. If business was bad, they had to .............. some of their staff.
A. put down

B. lay off

C. take on

D. hang up

22. The workers decided to .............. until their demands were met.
A. stand up

B. lie behind

C. sit in

D. sleep out

23. The union threatened to .............. the workers on strike.


A. call out

B. ask up

C. hear out

D. see down

24. On an impulse he .............. his job and went abroad.


A. gave in

B. put off

C. threw up

D. set down

25. The staff .............. in support of their pay claim.


A. fell off

B. walked out

C. kept back

D. turned off

26. If orders keep coming in like this, I will have to.............. more staff
A. give up

B. add in

C. gain on

D. take on

27. We got to the stop just as the coach was ...............


A. taking off

B. making out

28. Once again poor Colin has been


A. stood by

B. passed over

C. settling down

D. pulling away.

for promotion.
C. locked out

D. struck off

29. If my working conditions don't improve, I will ........... my notice.


A. Rive up

B. fire off

C. hand in

D. give out

30. We are looking for someone who can .........with the rest of our team.
A. get up

B. fit in

C. act up

D. work off

III. Choose the correct verb form among A, B, C or D for each numbered gap to
complete the passage.
Many changes (31) ............. to the world map this century. Boundaries (32) .............as a
result of world wars and foreign invasions, and many former colonies (33) ............. their
independence. In 1960, for instance, fifteen African countries (34) ............. independent,
some of which have changed their names. This tendency to change names when
countries became independent (35) ............. since then: Ceylon is now Sri Lanka and the

Trucial States (36) ............. the United Arab Emirates. More recently, in what
(37) ............. the Soviet Union, even cities (38) ............. by the change, the most notable
example (39) .............

Leningrad, which

in 1991 (40) ............. back to the pre-

Revolutionary name of St. Petersburg.


31.

A. have made

B. have been made

32.

A. have redrawn

B. has redrawn

C. have been redrawn

D. were redrawn

33.

A. have gained

B. gained

C. have been gained D. gain

34.

A. become

B. became

C. have become

35.

A. have continued B. has continued


C. continue

C. are made

D. make

D. has become

D. continues

36.

A. have become

B. becoming

C. to become

D. has become

37.

A. was

B. were

A. is

B. are

38.

A. have affected

B. have been affected

C. were affected

D. are affected

39.

A. is

B. are

C. is being

D. are being

40.

A. go

B. goes

C. went

D. has gone

IV. Choose the correct word form among A, B, C or D for each numbered gap to
complete the passage.
In spite of the fact that foreigners are always grumbling about English cooking, it's
(41) ............ whether their (42) ............ complaints are justifiable. It can be argued that this
stream of criticism tends to be based on rather (43) .............. experience and that the proof of
the pudding is in the eating. Owing to the fact that these people are (44).............to be short of
money, the seek out the more (45) ......... restaurants rather than (46) .......... establishments. If
your sole
experience of English cooking is (47) .............. to eating fish and chip, the (48) .......... of your
leaving the country with a (49).............impression is (50) ........... .
41.

A. debates

B. debate

C. debatable

D. debated

42.

A. continuous

B. continual

C. continue

D. continuation

43.

A. limited

B. limits

C. limit

D. limiting

44.

A. inclining

B. inclines

C. incline

D. inclined

45.

A. economic

B. economical

C. economy

D. economize

46.

A. renown

B. renowned

C. renowns

D. renowning

47.

A. confine

B. confined

C. confines

D. confining

48.

A. likelihood

B. likeliness

C. liking

D. likeness

49.

A. favourable

B. favours

C. favoured

D. favourably

50.

A. minimized

B. minimum

C. minimal

D. minimize

V. In each line below, four words or phrases have been underlined. Choose word or
phrase (A, B, C or D) that would not be accepted in standard
51. My cousin attends an university in the Midwest which specializes in astronomy.
A

52. It was him who came running into the classroom with the news.
A B

53. There's a new Oriental restaurant in town, isn't it?


A B

54. His father does not approve of him to go to the banquet without dressing formally.
A

55. Children enjoy telling and listening to ghosts stories, especially on Halloween night.
A

56. The Andersons has just had an enclosed brick patio built after fighting off the insects
A

for two months.


57. Paul did so well in his speech today that he should have rehearsed it many times
A

this past week.


D
58. Hal's new sports car costs much more than his friend Joel.
A

59. Max would rather to be fishing from this boat in the lake than sitting at his desk in the
office.
A

60. Sally must have called her sister last night, but she arrived home too late to call her.
A

61. If a crisis would occur. those unfamiliar with the procedures would not know how to
A

handle the situation.


62. Standing among so many strangers, the frightened child began to sob uncontrollable.
A

63. I do not believe that I have ever seen as many expensive cars than were in that shopping
A
B
C
D
center.

64. The novel is definitely a dense-packed narrative, but one which requires a
A

knowledge of cultural background or an excellent encyclopedia.


D
65. They played so good game of tennis last night that they surprised their audience.
A

Part C. READING
Read the following passage the choose the correct option among A, B, C or D to finish
the sentences or answer the questions below.
Unlike the eye the car has no lid; therefore noise penetrates without protection. Loud noises
instinctively signal danger to any organism with a hearing mechanism, including human
beings. In response, heartbeat and respiration accelerate. In fact, there is general increase in
functioning brought about by the flow of adrenaline released in response to fear.
Because noise is unavoidable in a complex, industrial society, we are constantly responding in
the same ways that we would respond to danger. Recently, researchers have concluded that
noise and our response may be much more than an annoyance. It may be a serious threat to
physical and psychological health and well-being, causig, damage not only to the ear and
brain but also to the heart and stomach. We have long known that hearing loss is America's
number one nonfatal health problem, but now we are learning that some of us with heart
disease and ulcers may be victims of noise as well.
66. What is the author's main point?
A. Hearing loss is America's number one nonfatal health problem.
B. Loud noises signal danger.

C. Noise may pose a serious threat to our physical and psychological healti
D. The ear is not like the eye.
67. According to the passage, people respond to loud noises in the same WI that they respond
to.......
A. danger

B. annoyance

C. damage

D. disease

68. It can be inferred from this passage that the.....................


A. respond to fear

C. increases functions

B. enjoys greater protection than the ear

D. is damaged by noise

69. Noise is...................


A. not a serious problem today.

B. an unavoidable problem in an industrial society.


C. America's number-one problem.

D. a complex problem.
70. What was the topic of the paragraph that preceded this passage?
A. fear

B. heart disease

C. ulcers

D. the eye

II. Choose the best option among A, B, C or D which best fills each of the blanks.
EDUCATION AND WORK
Have you ever asked yourself what you are working for? If you have ever had the time to
(71)......... this taboo question, or put it to others in moments of weakness or confidentiality,
you (72) .......well have heard some or all of the (73) .......It's the money of course, some say
with a smile, as if explaining something to a small child. Or it's the satisfaction of (74) .......
well done, the sense of achievement behind the clinching of (75).......important deal. I worked
as a bus conductor once, and I can't say I (76) ....... the same as I staggered along the swaying
gangway trying to (77)....... out tickets without falling over into someone's lap. It's the
company of other people perhaps, but if that is the (78) ....... , what about farmers? Is it the
conversation in the farmyard that keeps them captivated by the job? Work is power and sense
of status say those (79) .......have either attained these elusive goals, or feel aggrieved that
nobody has yet recognized their leadership (80) .......Or we can blame it all on someone else,
the family or the taxman. I suspect, and I say this under my (81)......., that most of us work
rather as Mr Micawber lived, hoping for something to (82) ....... up. We'll win the pools, and
tell the boss what we really think. We'll scrape together the (83) ....... and open that little shop
we always dreamed of, or go (84) .......the world, or spend more time in the garden. One day
we'll get that (85) .......we deserve, but until then at least we have something to do. And we are
so busy doing it that we won't have time to wonder why.
71.
72.
73.
74.
75.
76.
77.
78.
79.
80.
81.
82.
83.
84.
85.

A. consider
A. will
A. below
A. a work
A. a
A. enjoyed
A. make
A. case
A. people
A. qualities
A. oath
A. turn
A. resources
A. around
A. ambition

B. meditate
B. ought
B. rest
B. a job
B. a
B. wished
B. turn
B. one
B. must
B. status
B. suspicion
B. move
B. opportunities
B. over
B. promotion

C. propose
C. might
C. following
C. a task
C. the
C. felt
C. issue
C. question
C. who
C. property
C. breath
C. ease
C. rest
C. into
C. vocation

D. launch
D. would
D. latter
D. an effort
D. 0
D. hoped
D. give
D. former
D. to
D. requirements
D. pressure
D. end
D. money
D. to
D. station

Part D. WRITING
I. Choose the phrase or clause A, B, C or D that best completes each sentence.

86. The defendant refused to answer the prosecutor's questions.............


A. because he was afraid it would incriminate him
B. for fear that they will incriminate him
C. because he was afraid that his answers would incriminate him
D. fearing that he will be incriminated by it
87. .................. will Mr. Forbes be able to regain control of the company.
A. With hard work

B. In spite of his hard work

C. Only if he works hardly

D. Only with hard work

88. Mrs. Walker has returned................


A. a wallet back to its original owner
B. to its original owner the wallet
C. the wallet to its-originally- owner
D. the wallet to its original owner
89. The hospital owes................... for the construction of the new wing.
A. the government twenty million dollars
B. for the government twenty million dollars
C. to the government twenty million dollars
D. twenty millions of dollars to the government
90. ............. I admire you, I think you are silly.
A. Much as

B. Now that

C. As long as

D. In case

91. Having been asked to speak at the convention, .....................


A. some notes were prepared for Dr. Casagrande
B. Dr. Dr. Casagrande prepared some notes
C. the convention members were pleased to hear Dr. Casagrande
D. some notes were prepared by Dr. Casagrande
92. .................... so many people been out of work as today.
A. More than ever before

B. Never before have

C. In the past, there never have

D. Formerly, there never were

93. The artist was asked to show some paintings at the contest because .............
A. he painted very good
B. they believed he painted well
C. of their belief that he was an good artist
D. the judges had been told of his talents
94. I was the first person ............. that road accident.
A. to witness

B. witness

C. witnessing

D. witnessed

95. George would certainly have attended the proceedings ..............


A. if he didn't get a flat tire
B. if the flat tire hadn't happened
C. had he not had a flat tire
D. had the tire not flattened itself
II. Choose the sentence among A, B, C or D that is best made from the prompts.
96. W. Shakespeare / born / John Shakespeare / Mary Arden / 1564 / Stratford-upon-Avon.
A. W. Shakespeare was born to John Shakespeare and Mary Arden in 1564 in Stratfordupon-Avon.
B. W. Shakespeare was born by John Shakespeare and Mary Arden in 1564 in Stratfordupon-Avon.
C. W. Shakespeare was born of John Shakespeare and Mary Arden in 1564 of Stratfordupon-Avon.
D. W. Shakespeare was born with John Shakespeare and Mary Arden in 1564 in Stratfordupon-Avon.
97. Almost / _famous / pop singer / heard / this show.
A. Almost the famous pop singers will be heard in this show.
B. Almost famous pop singers will be heard in this show.
C. Almost all famous pop singers will be heard in this show.
D. Almost the most pop famous singers will be heard in this show.
98. English / become / effective / medium /international / communication .
A. English has become an international communication of effective medium
B. English has become an effective international communication of medium.
C. English has become an effective medium of international communication
D. English has an effective medium of become international communication

99. In Paris/ Hemingway meet/ many/ his old friends/ encourage/ his writ career.
A. In Paris, Hemingway encouraged many of his old friends who met him in his

writing career.
B. In Paris, Hemingway met many of his old friends, who encouraged him in his writing

career.
C. In Paris, Hemingway encouraged his writing career which many of old friends met.
A. In Paris, Hemingway encouraged his writing career to meet many of his old friends.

100. The two novel/ make/ Mark Twain famous/ "Tom Sawyer" and "Huckleberry Finn'/
inspire / the river environment.
A. It was "Tom Sawyer" and "Huckleberry Finn" which made Mark Twain famous by

inspiring the river environment.


A. It was the river environment and the two novels, "Torn Sawyer" and "Huckleberry

Finn", which inspired Mark Twain and made him famous.


B. It was the river environment which was inspired by the two novels that made Mark

Twain famous, "Tom Sawyer" and "Huckleberry Finn".


C. It is the two novels, "Tom Sawyer" and "Huckleberry Finn", inspired the river

environment that made Mark Twain famous.

ENGLISH OBJECTIVE TEST 12.


Time allowed: 90 min.
Part A PHONETICS
I. Choose the word whose underlined part is pronounced differently from the others in each
group.
1.
2.
3.
4.
5.

A. creative
A. mechanic
A. association
A. examiner
A. laugh

B. creature
B. chemist
B. digestion
B. determine
B. enough

C. creation
C. merchandise
C. indication
C. famine
C. plough

D. create
D. chorus
D. aviation
D. miner
D. cough

II. Choose the word whose second syllable is stressed in each line.
6. A. circumstance
B. awareness
C. institute
D. introduce
7. A. influential
B. advertiser
C. contaminate
D. numerator
8. A. extraordinary
B. mathematics
C. calculation
D. accuracy
9. A. inventory
B. economic
C. collaborator
D. editorial
10. A. artificial
B. managerial
C. systematize
D. diameter
Part B. LEXICO -GRAMMAR
I. Choose the answer among A, B, C or D which best fills the blank in each of following
sentences
11. It is a good idea to see your doctor regularly for .............
A. a revision

B. a control

C. an investigation D. a check-up

12. There is a fault at our television station. Please do not ........... your set.
A. change

B. adjust

C. repair

D. switch

13. He was in ......... of a large number of men.


A. management

B. leadership

C. charge

D. direction

14. When he retired from his job, the director ...... ... him with a clock.
A. offered

B. pleased

C. satisfied

D. presebted

15. When I talked to the manager, he told me he could ............. me a few munites.
A. provide
B. spare
C. hear
D. let
16. He opened the letter without ............. to read the address on the envelope.
A. worrying
B. caring
C. fearing
D. bothering
17. There was a big hole in the road which ............. the traffic.
A. held up
B. kept down
C. stood back
D. sent back
18. The explorers walked all the way along the river from its mouth to its ............. .
A. cause
B. well
C. source
D. outlet
19. He was afraid of losing his suitcase so he tied a ............. on it on which he written his
name.
A. badge
B. mark
C. label
D. notice
20. To our ............., his illness proved not to be as serious as we had feared.

A. anxiety

B. eyes

C. relief

D. judgement

II. Choose the most suitable preposition among A, B, C or D for each numbered gap to
complete the passage.
All living things depend a lot (21)....... plants. Plants provide us with food and shelter. Many
animals eat plants (22)........ grass and wheat. We, in turn kill them (23).......food. Green plants
also provide us with oxygen something that we breathe (24).......to live. Plants also provide us
with wood so that we can make them (25) .......materials for our house. Many animals like
birds, squirrels and insects live (26)....... trees. Even animals (27).......the sea need plants.
Seaweed gives (28).......oxygen so that there is oxygen in the water (29).......the fish and other
sea creatures. Seaweed is also food (30) ....... some of these creatures.
21. A. on

B. of

C. from

D. in

72. A. beyond

B. like

C. as

D. near

23. A. from

B. on

C. for

D. as

24. A. out

B. back

C. away

1). in

25. A. to

B. of

C. into

D. from

26. A. on

B. in

C. under

D. with

27. A. in

B. on

C. within

D. out of

28. A. into

B. out

C. in

D. off

29. A. along

B. with

C. for

D. on

30. A. about

B. with

C. from

D. to

III.. Choose the correct verb form among A, B, C or D for each numbered gap to
complete the passage.
This happened last year when one afternoon my husband and I (31)....... to
repaint our kitchen ceiling. It was about three o'clock and our two young children (32) .......in
their room quite happily. I had sent them there while we (33).......so that they (34) ............. in
our way. We (35) ....... for about twenty minutes when we suddenly (36).......that they (37)
....... talking and laughing. The silence was very worrying!
I (38) .......upstairs and opened the door of their room. They (39).......their wall which they
(40)....... painting blue, green and yellow. "We're painting too, Mummy," they said, smiling.
31.
32.
33.
34.
35.
36.
37.
38.
39.

A. decided
A. played
A. decorated
A. didn't get
A. were painting
A. had realised
A. had stopped
A. was rushing
A. admiring

B. had decided
B. had played
B. were decorating
B. don't get
B. was painting
B. realised
B. were stopping
B. rushing
B. were admiring

C. deciding
C. had been playing
C. was decorating
C. won't get
C. had been painting
C. had been realising
C. stopped
C. had rushed
C. admired

D. having
D. were playing
D. decorate
D. wouldn't get
D. painted
D. realise
D. stopping
D. rushed
D. had admired

40.

A. were just finishing


C. had just been finishing

B. was just finishing


D. had just finished

IV. Choose the correct word form among A, B, C or D for each numbered gap to
complete the passage.
Although recently there has been a small reduction in the number of people out of work in
Europe, (41) ....... is still the number one (42) ....... problem facing the member states of the
European Union. Moreover, (43) ....... of opportunity between men and women is still an
issue that (44) ....... have not come to grips with. In (45) ....... such as the law and engineering,
women are still noticeable by their absence. (46)

....... still discriminate against female

employees in a number of ways even if their (47) ....... . are the same as those of men. It would
be a pity if the (48) ....... of the EU on an (49) ....... level were marked by (50) ....... in the
important areas of social policy.
41.
42.
43.
44.
45.
46.
47.
48.
49.
50.

A. employment
A. socialist
A. unequality
A. politician
A. profess
A. Employees
A. qualified
A. achievement
A. economic
A. fail

B. unemployment
B. socialism
B. inequality
B. politicians
B. profession
B. Employers
B. disqualified
B. achievements
B. economical
B. failing

C. unemployed
C. social
C. equal
C. politics
C. professions
C. Employ
C. qualifications
C. achieve
C. economist
C. fails

D. unemployable
D. sociable
D. unequal
D. political
D. professional
D. Employment
D. qualities
D. achieves
D. economy
D. failures

V. In each line below, four words or phrases have been underlined. Choose the one word
or phrase (A, B, C or D) that would not be accepted in standard English.
51. I would rather that they do not travel during the bad weather, but they insist that
A
B
C
they must return home today.
A
52. That product that you bought at the lower price is the more inferior to the one that we
A
B
C
sell at a slightly higher price.
D
53. The general commanded the Officers' Club be off limits to the new recruits.
A
B
C
D
54. Louie got his sister read his class assignment, and then asked her to write the
A
B
report for him because he did not have enough time
C
D
55. It was suggested that Pedro studies the material more thoroughly before attempting to
A
B
C
D
pass the exam.
56. The man, of whom the red car is parked in front of our house, is a prominent
A
B
C
D
physician in this town.

57. The president mentioned the cabinet members that be was going to negotiate a
A
B
C
new treaty with the forerun minister.
D
58. Do you know how that the actors' strike will delay the beginning of the new
A
B
C
television season and cause the cancellation of many contracts?
D
59. We should have been informed Janis about the change in plans regarding our the
A
B
C
one weekend trip to the mountains.
D
60. When we arrived at the store to purchase the dishwasher physician in this town
A
B
advertise in the newspaper, we learned that all the dishwashers had been sold.
C
D
Part C. READING
I. Read the following passage and answer the questions that follow by choosing
the best option among A, B, C or D.
On the night of September 2nd, 1666 a fire broke out in a baker's shop near Fish Street Hill
in London. Before the flames were finally extinguished, nearly the entire city had been
reduced to ashes. Over thirteen thousand homes, fifty churches, and numerous public
buildings and hospitals were lost in the blaze. For all practical purposes, London was
destroyed.
The Great Fire was not seen as a total tragedy, however. The deplorable conditions of the
city had been attacked by physicians and humanitarians for years before the fire; thus, with
the opportunity clearly presented to create a shining new city; artists and craftsmen from all
over England hurried to submit their designs for the rebuilding of London.
Among those who submitted plans was Sir Christopher Wren, one of England's leading
architects and the Surveyor General of London. The task of rebuilding the city was given to
him. Wren realized that the Great Fire would not have been so damaging if the city had
been better laid out broader streets were needed to replace the crooked, narrow alleys
overhung with dilapidated the main thoroughfares of London would result in increased and
more effective transportation within the city.
Shortly after Wren began working on his first drafts for the rebuilding, King Charles I
issued a proclamation prohibiting the construction of any house or shop within the city
limits unveiled to the citizens of London, however, they were overwhelmingly rejected. The
most vocal leaders of the opposition were the landlords, who feared that such a drastic
widening of the streets would reduce the amount of land available for development.

Winter was approaching; consequently, it was necessary for the rebuilding to proceed at once.
Permission was, therefore, granted for the townspeople and landlords to commence
reconstruction of their houses and shops at the sites where they had been before the fire. Had
the need for immediate action not been so pressing, some kind of compromise could likely
have been reached. This was not to be, however, and the ideas that could have London one of
the world's most beautiful cities never came to pass.
61. The probable meaning of reduced to ashes is................
A. destroyed by fire

B. discolored by the fire and smoke

C. covered with ashes

D. made unbearably hot

62. Which of the following were probably among the deplorable condition that London
physicians and humanitarians had spoken out against?
A. Lack of adequate lighting.
B. The run-down condition of many houses and shops.
A. The narrowness and crookedness of many streets and alleys.
C. Both options A and B.

63. What was the result of the plans for the rebuilding of London?
A. Nothing they were never finished.
B. They were adopted in modified form.
C. They were followed and London was rebuilt at once.
D. Nothing London was rebuilt as it had been before the fire.

64. Which of the following describes the author 's probable attitude toward Christopher
Wren's plans?
A. He feels it would have been a mistake to rebuild London according to Wren's
designs.
B. He feels it was a mistake for London not to have been rebuilt according to Wren's
designs.
C. He feels that someone other than Wren should have been chosen to plan the
rebuilding.
D. He feels that Wren's ideas had much more influence than is commonly
acknowledged.
65. Which of the following would he an appropriate title for the passage?
A. The Life and Times of Sir Christopher Wren.
A. The Rebuilding of England.
B.

The Holocaust of Fish Street Hill.

C.

The London That Might Have Been.

II. .....Choose the best option among A, B, C or D which best fills each of the
blanks.
HEALTH AND THE BODY
Keeping fit and staying healthy have, not surprisingly, become a growth industry. Quite apart
(66) ....... the amount of money spent each year on doctors' (67) ....... and approved medical
treatment, huge sums are now spent on health foods and (68) ....... of various kinds, from
vitamin pills to mineral water, not to mention health clubs and keep-fit (69) ....... and videos.
We are more concerned than ever, it seems, (70) ....... the water we drink and the air we
breathe, and are smoking less, though not yet drinking less alcohol. This does not appear to
mean that (71) ....... and sneezes have been banished, or that we can all expect to live to a
hundred. To give a personal example, one of my friends, (72) ....... is a keep-fit fanatic, a nonsmoker and teetotaler, and who is very (73) ....... in what he eats, is at .present languishing in
bed with a wrist in (74) ....... and a badly sprained ankle. Part of his healthy (75) ....... is to
play squash every day after work, and that (76) .......

for the ankle. He also cycles

everywhere, and if you have ever tried to cycle through the rush-hour traffic with a sprained
ankle, you will understand (77) ....... he acquired the broken wrist. For (78) ....... , it seems,
is not just a matter of a good (79) ....... and plenty of exercise. Too much exercise can be
harmful, as many joggers have discovered. Eating the right food can easily become an
obsession, as can overworking, which you might have to do so as to be able to afford your
(80) ....... of the squash club, your mountain bike, your health food, and a few holidays in
peaceful and healthy places.
66. A. for

B. with

C. from

D. of

67. A. prescriptions

B. surgeries

C. hospitals

D. payments

68. A. medications

B. cures

C. drugs

D. remedies

69. A. books

B. television

C. advice

D. enthusiasts

70. A. than

B. about

C. for

D. hence

71. A. colds

B. coughs

C. flu

D. fevers

72. A. who

B. whom

C. whose

D. whoever

73. A. interested

B. varied

C. detailed

D. particular

74. A. crutches

B. plaster

C. treatment

D. danger

75. A. living

B. lifetime

C. lifestyle

D. liveliness

76. A. is

B. caters

C. depends

D. accounts

77. A. how

B. that

C. whenever

D. thus

78. A. fit

B. this

C. health

D. all

79. A. diet

B. eating

C. menu

D. recipe

80. A. share

B. visit

C. membership

D. subscription

Part D. READING
I. Choose the phrase or clause A, B, C or D that best completes each sentence.
81. After the funeral, the residents of the apartment building.............
A. sent faithfully flowers all weeks to the cemetery
B. sent to the cemetery each week flowers faithfully
C. sent flowers faithfully to the cemetery each week
D. sent each week faithfully to the cemetery flowers

82. . The committee has met and

........

A. they have reached a decision


B. it has formulated themselves some opinions
C. its decision was reached at
D. it has reached a decision

83. Having been served lunch, ..


A. the problem was discussed by the members of the committee
B. the committee members discussed the problem
C. it was discussed by the committee members the problem
D. a discussion of the problem was made by the members of the committee
84. Florida has not yet ratified the amendment, and
A.several other states hasn't either
B. neither has some of the others states
C. some other states also have not either

D. neither have several other states


85. The chairman requested that.........
A.the members studied more carefully the problem
B. the problem was more carefulnessly studied
C. with more carefulness the problem could be studied
D.the members study the problem more carefully

86. The professor said that...........


A. the students can hand in their reports on the Monday
B. the reports on Monday could be received from the students by him
C. the students could hand in their reports on Monday
D. the students could hand in on Monday their reports
87. This year will be difficult for this organization because.........
A. they have less money and volunteers than they had last year 3. it has less money

and fewer volunteers than it had last year


C. the last year it did not have as few and little volunteers and money
D. there are fewer money and volunteers that in the last year there were

88. The teachers have had some problems deciding.........


A. when to the students they shall return the final papers
B. when are they going to return to the students the final papers
C. when they should return the final papers to the students
D. the time when the final papers they should return for the students
89. She wanted to serve some coffee to her guests; however, .......
A. She hadn't many sugar
B. There was not a great amount of the sugar
C. She did not have much sugar
D. She was lacking in amount of the sugar
90. George would certainly have attended the proceedings............
A. if he didn't get a flat tire
B. if the flat tire hadn't happened
C. had he not had a flat tire
D. had the tire not flattened itself

II. Choose the sentence among A, B, C or D that is closest in meaning to original


sentence.
91. It was obvious that the old house was past its prime.
A. The old house had known better days.
B. The old house had known worse days.
C. The old house had good days.
D. The old house had had days.

92. The film didn't come up to my expectations.


A. The film was short of my expectations.
B. The film fell short of my expectations.
C. I was short of the film.
A. I fell short of the film.

93. " You don't appreciate me," she complained.


A. She complained that I took her for granted.
B. She complained that I had taken her for granted_
C. She said I don't appreciate her.

D. She said I hadn't appreciated her.

94. No sooner had they found her number than they called her.
A. They called her as soon as they found her number.
B. They found her number sooner or later.
C. They called her number sooner or later.
D. They found her number as soon as they called her.

95. Why didn't you switch off the computer before going home?
A. You should switch off the computer before going home.
B. You had switched off the computer before going home.
C. You ought to have switched off the computer before going home.
D. You must have switched off the computer before going home.

96. No matter how hard I tried I couldn't open the door.


A. Try as hard as I might, I couldn't open the door.
B.

I tried hard to open the door.

C.

It was difficult for me to open the door.

D. I could open the door with difficulty.

97. The flight lasts twenty four hours.


A. It is a twenty-four- hours flight.
B.

It is a twenty-four- hour flight.

C.

It is twenty-four- hour flight.

D. It is twenty-four- hours flight.

98. Someone has stolen her- bike.


A.

She has had her bike stolen.

B.

Her bike was stolen.

C.

She had her bike stolen.

D.

She has her bike stolen.

99. Having put the child to bed, I phoned the doctor.


A.

Before I put the child to bed, I phoned the doctor.

A. Having phoned the doctor, I put the child to bed.


B. Putting the child to bed, I phoned the doctor.
B.

After I had put the child to bed, I phoned the doctor.

100. They said, "Hurrah! We have won the contest!"


A. They said Hurrah that they had won the contest.
A. They said they had won the contest.
B. They exclaimed with joy that they had won the contest.

B.

They exclaimed with joy that they won the contest.

ENGLISH OBJECTIVE TEST 13


Time allowed: 90 min.
Part A. PHONETICS
I. Choose the word whose prominent stress is different from the others in the same line..
1. A. determination

B. accumulation

C. humiliation

D. acknowledgement

2. A. horizon

B. collection

C. petroleum

D. atmosphere

3. A. geometry

B. commentator

C. astonishment

D. conventional

4. A. circumvent

B. syllabus

C. sensitive

D. octopus

5. A. catalogue

B. championship

C. embarrass

D. character

II. Choose the word whose underlined part is pronounced differently from the others in
each group
6. A. explanation

B. adventure

C. acronym

D. addition

7. A. choir

B. chill

C. chief

D. charge

8. A. colony

B. company

C. colourful

D. covering

9. A. throughout

B. smooth

C. threaten

D. thunder

10. A. ranged

B. transformed

C. vaporised

D. attached

Part B. LEXICO -GRAMMAR


I.

Choose the answer among A, B, C or D which best fills the blank in each of
following sentences.
The house that we used to live in is in a very............. state.

11.

A. neglectful
12.

D. negligent

B. a few

C. little

D. a little

My brother always ............ at cards. No wonder he won every game.

A. cheated
14.

C. neglected

We wanted to give him ............ help, but he turned us down.

A. few
13.

B. negligible

B. lied

C. tricked

D. deceived

He was so tired that he wasn't capable ............ driving himself alone.

A. to

B. for

C. of

D, from

15. She went............ a bad cold just before Christmas.

A. down with

B. in for

.C. over

D. through

16. She used to ............ her living by delivering vegetables to local hotels.

A. earn .

B. gain

C. get

17. That sweater looks ............ small for a five-year-old.

D. win

A. bit

B. much

C. rather

D. even

18. since I moved house, I haven't had much ............ with those friends.

A. connection

B. contact

C. business

D. meeting

19. The ship ran ............ on the beach near the harbour.

A. afloat

B. adrift

C. aback

D. aground

20. I hope we will be able to avoid ............ anyone.


A. cheering

B. disappointing

C. satisfying

D. disagreeing

II. Choose the correct word form among A, B, C or D for each numbered gap to complete
the passage.
My wife and I (21) ....... in our house in the country for five years. We (22) ....... here
after our second child (23) ....... born. We (24) ....... in town for ten years and (25) ....... that
as soon as we could (26) ....... it, we (27) ....... . away from the smoke and the noise of the
city centre, which we finally (28) ....... in 1985. We have never regretted it. We (29) .......
reminded of the wisdom of our decision every morning when we draw our curtains (30) .......
open fields stretching before us.
21. A. lived

B. has lived

C. have been living D. live

22. A. move

B. moving

C. had moved

D. moved

23. A. were

B. was

C. is

D. are

24. A. had lived

B. had been living

C. lived

D. living

25. A. decided

B. decide

C. had decided

D. deciding

26. A. afford

B. to afford

C. affording

D. afforded

27. A. would move

B. will move

C. move

D. to move

28. A. do

B. done

C. did

D. had done

29. A. is

B. are

C. was

D. were

30. A. saw

B. see

C. for seeing

D. to see

III. Choose the correct word form among A, B, C or D for each numbered gap to
complete the passage.
Recently (31) ........... have become extremely worried about the world's population. The
problem is particularly acute in the (32) ........... countries. (33) ........... the natural resources
are becoming inadequate for the support of the increasing population. But the (34) ........... of
natural resources by artificial products is not always possible. Therefore, it is necessary to
conserve these (35) ........... resources for as long as possible. The (36) ........... of artificial
products is often important. This is (37) ........... so in every culture. For without (38) ........... it
would be impossible to increase food production even though there is a risk of (39) ........... by
their use. One is left with the (40) ........... problem of larger population requiring more and

more food.
31.

A. science

B. scientist

C. scientists

D. scientific

32.

A. developed

B. developing

C. development

D. develop

33.

A. Fortunate

B. Unfortunate

C. Fortunately

D. Unfortunately

34.

A. replace

B. replacement

C. placement

I). place

35.

A. invaluable

B. valualess

C. value

D. invalue

36.

A. utiliser

B. utilisation

C. utilising

D. utilise .

37.

A. particular

B. particularly

C. particularities

D. particularity

38.

A. fertilizers

B. fertile

C. fertilization

D. fertilize

39.

A. pollutant

B. pollute

C. pollution

D. pollute

40.

A. insoluble

B. unsoluble

C. unsolvable

D. unsolving

IV. In each line below, four words or phrases have been underlined, Choose the One
word or phrase (A, B, C or D) that would not be accepted in standard English.
41. The proposal has repealed after a thirty-minute discussion and a number of objections to
A
B
C
its failure to include our district.
D
42. This class. that is a prerequisite for microbiology, is so difficult that I
A
B
C
would rather drop it.
D
43. The doctor told Mr. Anderson that, because of his severe cramps, he should, lay in
A
B
C
bed for a few days.
D
44. Mr. Harris will be divided the biology class into two sections to prevent
A
B
C
overcrowding in his classroom.
D
45. The projector director stated he believed it was necessary to study the
A
B
C
proposals for several more months before__making a decision.
D
46. Although the danger that he might be injured. Boris bravely entered the
A
B
C
burning house in order to save the youngster.
D
47. Some Italian scholars stressed the study of grammar, rhetoric, learning about
A
B
C
D
history, and poetry.
D
48. When the tank car carried the toxic gas derailed, the firemen tried to isolate
A
B
C
the village from all traffic.

D
49. While the boys were ice skating, they slip on the thin ice and fell into the
A
B
C D
deep water.
50. If motorists do not observe the traffic regulations. they will be stopped, ticketed,
A
B
C
and have to pay a fine.
D
V. Choose the best option among A, B, C or D to fill each gap to finish the following
sentences.
51. The ............... collar workers received a rise, but the workers on the shop-floor ere told
they had to wait.
A blue

B. black

C. white

D. grey

52. We decided to celebrate by going out and painting the town


A. red
53.

C. black

D. pink

B. green

C. yellow

D. red

B. green

C. yellow

D. white

B. gold

C. yellow

blue

B.- orange

C. red

D. green

I'm fed-up with this job. I feed completely ........off.


A. browned

53.

B. blue

When the visitors from Japan arrived, the company gave them the carpet treatment.
A. blue

59.

D. orange

Tina never comes here now. We only see her once in a.........moon.
A. white

58.

C. yellow

When Bill saw my new car he was........with envy.


A. blue

57.

B. pink

The company fell deeper and deeper into the........and then went bankrupt.
A. black

56.

D. brown

You can talk to him until you're..... in the face, but he still won't understand.
A. white

55.

C. gold

Tony can't be trusted yet with too much responsibility, he's still .........
A. green

54.

B. purple

B. blued

C. blacked

D. yellowed

Julie's letter was unexpected. It arrived completely out of the.........


A. pink

B. green

C. blue

D. red

Part C READING
I.

Read the following, passage and answer the questions that follow b choosing
the best option among A, B, C or D.

Polyester is now being used for bottles. ICI, the chemicals and plastics company, believes that
it is now beginning to break the grip of glass on the bottle business and thus take advantages
of this huge market.

All the plastics manufacturers have been experiencing hard times as their traditional products
have been doing badly world-wide for the last few years. Between 1982 and 1984 the Plastics
Division of ICI lost a hundred and twenty million dollars, and they felt that the most hopeful
new mark,- was in packaging, bottles and cans.
Since 1982 it has opened three new factories producing "Melinar" - the raw material from
which high quality polyester bottles are made.
The plyester bottle was born in the 1970s, when soft drinks companies like Coca Cola started
selling their drinks in giant two-litre containers. Because of the build-up of the pressure of gas
in large containers, glass was unsuitable. Nor was PV S, the plastic which has been used for
bottles since the 1960s, suitable for drinks with gas in them. A new plastic had to be made.
Glass is still cheaper for the smaller bottles, and will continue to be so unless plastic becomes
much cheaper, but plastic does well for the large sizes. Polyester bottles are virtually
unbreakable. The manufacturers claim they are also lighter, less noisy when being handled,
and can be re-used. Shop-keepers and other business people are unlikely to object to a change
from glass to polyester, since these bottles mean few breakages, which are costly and timeconsuming. The public, though, have been more difficult to persuade. ICI's commercial
department is developing, different bottles with interesting shapes, and try to make them
visually more attractive to the public.
The next step could he to develop a plastic which could replace tins for food. The problem
here is the high temperatures necessary for cooking the food in the container.
61. Plastics of various kinds have been used for making bottles
A. since 1982.
B. since the 1970s but only for large bottles.
C. since the 1960s but not for liquids with gas in them.
D. since companies like Coca Cola first tried them.

62. Why is ICI's Plastics Division interested in polyester tar bottlers?


A. The other things they make are not selling well.
B. Glass manufacturers cannot make enough new bottles.
C. They have factories which could be adapted to make it.

D. The price of oil keeps changing.


63. Why aren't all bottles now made of polyester ?
A. The price of oil and plastic has risen.
B. It is not suitable for containing gassy drinks.
C. The public like traditional glass bottles.
D. Shop-keepers dislike re-usable bottles.
64. Manufacturers think polyester bottles are better than glass ones because they

A. are more expensive.


B. are more suited to small sizes.
C. are more exciting to look at.
C. do not break easily.

65. Plastic containers for holding food in the same way as cans
A. have been used for many years.
A. are possible only for hot food.
B. are an idea that interests the plastics companies.
B. are the first things being made in the new factories.

II. Choose the best option among A, B, C or D which best fills each of the blanks.
CRIME - REVERSING THE TREND
Crime, as we are all aware, has been a growing problem all over the world in the last thirty
years.

But

we

are

not

(66)

.........

against

crime.

Much

is

being

done-and more can be done to reverse the trend. You can play a part in it. The fist step
towards preventing crime is understanding its (67) ......... Most crime is against property, not
(68) ......... and most is not carried out by professionals; nor is it carefully planned. Property
crimes (69) ......... on the easy opportunity. They are often (70) ......... by adolescents and
young men, the majority of whom stop offending as they grow older - the (71) ......... age for
offending is fifteen. Also, and not surprisingly, the (72) ......... of being a victim of crime (73)
......... greatly depending on where you live.
This (74) ......... by criminals on the easy opportunity is the (75) ......... to much crime
prevention. Motor cars, for example, are a sitting target for t criminal. Surveys have shown
that approximately one in five drivers do not always (76) ......... to secure their cars by locking
all the doors and shutting all the windows, and in 30 per cent of domestic burglaries the
burglar simply walks in without having to use (77) ......... if opportunities like these did not
exist, criminals would have a much harder time. The chances are that many crimes would not
be committed, which would release more police time for (78) ......... serious crime.
Of course, the primary responsibility for (79) ......... with crime rests with the police and the
courts, but, if you care about improving the (80) ......... of life for yourself and your
community, there are many ways you can help reverse the trend.
66.

A. unprepared

B. hopeless

C. powerless

D. weak

67.

A. nature

B. type

C. reason

D. notorious

68.

A. the victim

B. the public

C. residents

D. citizens

69.

A. members

B. thrive

C. develop

D. happen

70.

A. performed

B. started

C. committed

D. done

71.

A. peak

B. major

C. maximum

D. top

72.

A. percentage

B. seriousness

C. rate

D. risk

73.

A. varies

B adapts

C. transformed

D. adjusts

74.

A. awareness

B. seizing

C. reliance

D. taking

75.

A. answer

B. method

C. way

D. key

76.

A. trouble

B. bother

C. care

D. ensure

77.

A. force

B. threat

C. tools

D. tricks

78.

A. removing

B. facing

C. tackling

D. dealing

79.

A. containing

B. destroying

C. fighting

D. coping

80.

A. quality

B. peacefulness

C. enjoyment

D. way

Part D: WRITING
I. Choose the phrase or clause A, B, C or D that best completes each sentence.
81. ......... , he began to make friends more easily.
A. Having entered school in the new city, it was found that
A. After entering, the new school
B. When he had been entering the new school
C. Upon entering into the new school

82. It is very difficult to stop the cultivation of marijuana because........


A. it grows very carelessly
B. of it's growth without attention
C. it grows well with little care
D. it doesn't care much to grow

83. The ,fact that space exploration has increased dramatically in the past thirty years .........
A. is an evidence of us wanting to know more of our solar system
B. indicates that we are very eager to learn all we can about our solar system
C. how we want to learn more about the solar system
D. is pointing to evidence of our intention to know a lot more about what is called our

solar system
84. Many of the international problems we are now facing............
A. linguistic incompetencies
B. are the result of misunderstandings
C. are because of not understanding themselves
D. lacks of the intelligent capabilities of understanding each other

85. Mr. Roberts is a noted chemist............

A. as well as an effective teacher


B.

and too a very effective teacher


C. but he teaches very good in addition
D. however he teaches very good also

86. Public television stations are different from commercial stations............


A.because they receive money differently and different types of shows
B. for money and program types
C. in the areas of funding and programming
A. because the former receives money and has programs differently from the latter

87. Manufacturers often sacrifice quality...........


A. for a larger profit margin
B. in place of to earn more money
C. to gain more quantities of money
D. and instead earn a bigger amount of profit

88. Automobile production in the United States............


A. have taken slumps and rises in recent years
B. has been rather erratic recently
C. has been erratically lately
D. are going up and down all the time

89. A major problem in the construction of new buildings..............


A. is that windows have been eliminated while air conditioning systems have not been
perfected
B. is they have eliminated windows and still don't have good air conditioning
C. is because windows are eliminated but air conditioners don't work good

D. is dependent on the fact that while they have eliminated windows. are not capable to
produce efficient air conditioning systems
90. While attempting to reach his home before the storm, .........
A. the bicycle of John broke down
B. it happened that John's bike broke down
C. the storm caught John
D. John had an accident on his bicycle
II. Choose the sentence among A, B, C or D that is closest in meaning to the original
sentence.
91. Don't wait any longer than ten minutes before you remove the crucible from the flame.

A. If possible, the crucible should remain in the flame longer than ten minutes.
B. Ten minutes is as long as you should wait before removing the crucible from the flame.
C. Removing the crucible from the flame for ten minutes is what you should do.
D. You should wait ten minutes to put the crucible in the flame.
92. It is not whether you win or lose, but how you play the game that is important.
A.

Winning is more important than losing, when you play the game.
A. Winning, is less important than playing the game well.
B. If you know how to play the game, you will win it.
C. Playing the game well and winning it is important.
93. It is necessary to have a doctor's prescription in order to buy most medicines in the United
States.
A. In the United States, medicine must be bought with prescription instead of money.
B. In most of the states, doctors give prescriptions for medicine.
C. Most medicine cannot be bought without a prescription in the United States.
D. In the United States, most doctors give prescriptions for medicine.
93. It is necessary to have o doctor's prescription in order to buy most medicines in the
United States.
A. In the United States, medicine must be bought with prescription instead of money.
B. In most of the states, doctors give prescriptions for medicine.
C. Most medicine cannot be bought without a prescription in the United States.
D. In the United States, most doctors give prescriptions for medicine.
94. No one except the graduate assistant understood the experiments.
A. All of the graduate assistants understood the experiments.
B. The experiments were not understood by any of them.
C. Only the graduate assistant understood the experiments.
D. All but one of the graduate assistants understood the experiments.
95. It is unlikely that the results of the elections will be made public before tomorrow
morning.
A. Tomorrow morning is probably the earliest that anyone will know the result of the

selection.
B. Before tomorrow morning we will probably know the result of the elections.

C. The results of the elections will most likely be made known before tomorrow morning.
A. We will probably not be told the results of the elections tomorrow morning.

96. As a conductor of heat and electricity, aluminum exceeds all metals except silver, copper,
and gold.
A. With the exception of aluminum, silver, copper, and gold are better thai--, any other

metal as conductors of heat and electricity.


B. Aluminum is a better conductor of heat and electricity than silver, copper and gold.
C. Silver, copper, and gold are better conductors of heat and electricity than aluminum is.
D. Silver, copper, and gold are exceeded only by aluminum as conductors a heat and

electricity.
97. Unless the trend reverses, low-priced pocket calculators will have replace.: the slide rules
completely within the next few years.
A. Slide rules will have been replaced by low-priced pocket calculators se-, if the trend

continues.
B. More people will be using slide rules than pocket calculators even the they are more

expensive unless the trend reverses.


C. Because they are low-priced, pocket calculators will replace slide rules the next few

years.
D. The trend is for slide rules to be used in spite of the low

prices of p calculators.
98. It is not illegal in some cultures to be married to more than one woman the same time, but
the monogamous relationship is the most common.
A. in some cultures it is legal to be married to more than one woman at once, but it is
more common to be married to just one woman.
B. It is against the law to be married to more than one woman at a time, although it is
common in some cultures.
C. The monogamous relationship, which is more common in most cultures, is illegal in
some cultures.
D. Being married to more than one woman at the same time is both common and legal in
some cultures.
99. Airlines routinely promise seats to 10 or 20 per cent more passengers than can he
accommodated because some people who confirm their reservations fail to keep
them.
A. About 10 to 20 per cent of the passengers who book flights do not travel because there

are no seats.
B. The people who confirm their reservations do not have seats because the airlines
promise them to more passengers than they can serve.
C. About 10 to 20 per cent of the people who want to fly can be accommodated because
of extra seats.
D. The airlines reserve seats at 110 to 120 per cent of capacity because of people who
make reservations but don't show up.
10. Turn off all the switches before leaving the room
A. All the switches must be turned off and you must leave the room.
B. All the switches must be turned off before you leave the room.
C. All the switches must be turned off before leaving the room.
D. You must leave the room as soon as you turn off all the switches.
ENGLISH OBJECTIVE TEST 14
Time allowed: 90 min.
Part A. PHONETICS
I. Choose the word in each group that has the underlined, italic part pronounced
differently from the rest
1. A. indecent

B. deficient

C. deform

D. indefinite

2. A. elephant

B. electricity

C. elevator

D. elegant

3. A. telegraph

B. telegram

C. telegraphy

D. telescope

4. A. champagne

B. champion

C. parachute

D. moustache

5. A. hesitant

B. president

C. Resident

D. lactose

II. Find the word having the main stress on the SECOND syllable in each line.
6. A. endurance

B. primary

C. corporate

D. corridor

7. A. separate

B. fancy

C. device

D. gaudy

8. A. executive

B. recognition

C. expectation

D. aspiration

9.A. documentary

B. significant

C. disappointing

D. applicable

10. A. furious

B. commercial

C. pharmacist

D. vacuum

Part B. LEXICO GRAMMAR


I. Circle the best answer for each sentence.
11..., we missed our plane.
A. The train is late

B. The train was late

C. To be late

D. The train being late

12.Although he claimed to have left his job voluntarily, he was actually ..


misconduct.
A. resigned

B. released

C. dismissed

13. They were..for smuggling jewelry into the country.

D. dispelled

A. judged

B. arrested

C. accused

D. warned

14. At the end of the winter, the price of warm clothes in the shops usually ..
A. drops

B. lowers

C. sinks

D. reduces

15. The dampness of the walls caused the paint to ... off.
A. crumble

B. splinter

C. flake

D. scale

16. I decided to buy this air-conditioner because I had confidence in the ..


name.
A. mark

B. maker

C. commodity

D. brand

17. Last night, Harry had a .. party with his female friends before getting
married.
A. hen

B. giraffe

C. stag

D. crocodile

18. I don't know him very well. He's just married


A. a friend

B. an acquaintance C. an associate

D. a fellow

19. The country's wealth comes chiefly from its many ..


A. herds of cattle

B. packs of cattle

C. crowds of cattle

D. schools of cattle

20. The factory dumped the nuclear wastes .


A. in secrecy

B. in secret

C. in a secret

D. in the secret

21. Its .. to decide where to go for the weekend.


A. on you

B. up for you

C. up to you

D. up with you

22. I will finish the job for the best of my


A. capacity

B. ability

C. knowledge

D. talent

23. There has been a great in his studying since the beginning of the year.
A. escalation

B. increase

C. practice

D. improvement

24. You must move your car at once . I shall have to give you a fine.
A. therefore

B. consequently

C. whether

D. otherwise

25. My teacher went . a bad cold just before the summer holiday.
A. out with

B. in for

C. down with

D. through with

26. Schools examinations help to find out how much knowledge students have
A. fetched

B. gained

C. attached

D. caught

27. What you wear for the interview is always important because it creates the first ..
A. appearance

B. performance

C. expression

D. impression

28. The more you relax and be yourself, the more . you are to succeed.
A. easy

B. possible

29. He robbed the bank and was sentenced

C. probable

D. likely

.. five years imprisonment.

A. with

B. for

C. to

D. in

30. It is important that she to tell him the truth and apologize him.
A. went

B. goes

C. go

D. will go

III. Choose the correct form of the verb for each blank.
When I opened the door I saw a man on his knees. He (31) ....... to our conversation and I
wondered how much he (32) ....... When I asked him what he (33) ....... he answered that he
(34) ....... a 50p piece outside the door and was looking for it. I didn't see any sign of the
money but I found a small notebook and a pencil which he had probably dropped when the
door (35) ....... suddenly. So he (36) ....... notes of our conversation! The notes were in a
foreign language, so I turned to the stranger and asked him to translate. But he (37) ....... my
hat over my eyes and (38) ....... off down the corridor. By the time I (39) ....... from the shock
he (40) ....... round the corner.
31 . A. had clearly been listening
C. clearly listened

B. had clearly listened


D. had been clearly listening

32.

A. was hearing

B. heard

C. had heard

D. had been hearing

33.

A. did

B. had done

C. was doing

D. has been doing

34.

A. had dropped

B. dropped

C. was dropping

D. had been dropping

35.

A. had opened

B. was opened

C. opened

D. was being opened

36.

A. had taken

B. had been taking

C. was taking

D. took

37.

A. pulled

B. had pulled

C. was pulling

D. was pulled

38.

A. ran

B. run

C. had run

D. was running

39.

A. am recovered

B. have recovered

C. recovered

D. would recover.

A. disappeared

B. would disappear

C. has disappeared

D. had disappeared

40.

II. Choose the correct form of the word to complete the following passage.
These days it is impossible to open a newspaper without reading about damage we are doing
to the environment. The earth is being (41) the future looks bad. What can each of us
do?
We cannot clear up our (42) rivers and seas overnight. Nor can we stop the (43)
of plants and animals. But we can stop adding to the problem while (44)
search for answers, and laws are passed in nature's (45) it may not be easy to change
our lifestyle (46) , but some steps are easy to take: cut down the amount of (47)

you do, or use as little plastic as possible. it is also easy to save energy, which also
reduces (48) bills. We must all make a personal (49) to work for the future of
our planet if we wanted to (50) a better world for our grandchildren!
A. threatening

B. threatened

C. threateningly

D. threat

A. pollution

B. pollutant

C. polluted

D. polluting

A. disappearance

B. appearance

C. appears

D. disappears

A. scientific

B. scientist

C. scientism

D. scientists

A. defended

B. defense

C. defending

D. defensive

A. complete

B. completion

C. completely

D. completive

A. driving

B. driven

C. drives

D. driver

A. housing

B. housecraft

C. household

D. housekeeping

A. decisive

B. decisiveness

C. decision

D. decisional

A. insurable

B. insure

C. insurance

D. insurer

Part C. READING
I. Read the following two passages and choose the correct answer for each question.
Most people picture sharks as huge, powerful, frightening predators, ready at any moment to
use their sharp teeth to attack unwary swimmers without provocation. There are numerous
fallacies, however, in this conception of sharks.
First, there are about 350 species of sharks, and not all of them are large. They range in size
from the dwarf shark, which can be only 6 inches (0.5 feet) long and can be held in the palm
of the hand, to the whale shark, which can be more than 55 feet long.
A second fallacy concerns the number and type of teeth, which can vary tremendously among
the different species of shark. A shark can have from one to seven sets of teeth at the same
time, and some types of shark can have several hundred teeth in each jaw. It is true that the
fierce and predatory species do possess extremely sharp and brutal teeth used to rip the prey
apart; many other types of shark, however, have teeth more adapted to grabbing and holding
than to cutting and slashing.
Finally, not all sharks are predatory animals ready to strike out at humans, on the least whim.
In fact, only 12 of the 350 species of shark have been known to attack humans, and a shark
needs to be provoked in order to attack. The types of shark that have the worst record with
humans are the tiger shark, the bull shark, and the great white shark. However, for most
species of shark, even some of the largest types, there are no known instances of attacks on
humans.

51. The author's main purpose in the passage is to ..


A.

categorize the different kinds of sharks throughout the world

B.

warn humans of the dangers posed by sharks

C.

describe the characteristics of shark teeth

D.

clear up misconceptions about sharks


52. The word "unwary" in line 2 is closest in meaning to ..
A. strong

B. combative

C. careless

D. fearful

C. bull shark

D. tiger shark

53. The longest shark is probably the ..


A. whale shark

B. great white shark

54. Which of following is NOT true about a shark's teeth?


A.

All sharks have teeth.

B.

A shark can have six rows of teeth.

C.

A shark can have hundreds of teeth.

A.

All sharks have extremely sharp teeth.

55. A "jaw" in line 9 is.............


A. a part of the shark's tail

B. a part of stomach

C. a backbone

D. a bone in the mouth

Jan Dibbets may someday have his work revered as much as his Dutch predecessors: Verneer,
Van Gogh, and Rembrandt. At a time when trends in art are toward abstraction and
minimalism, Dibbets' work integrates these two disparate trends into one remarkable whole.
In one series of composite works, he arranged sections of architectural photographs
into rounded patterns. Then, with pen and ink and watercolours, he connected the segments
together into 360-degree circular forms blending the abstract with the real and the imagined.
The imaginary images resemble the views of ceiling in gothic, baroque, and neo-classical
buildings as they might be seen from the ground floor.
56. As used in line 2, what is the meaning of the word "time"?
A. season

B. period

C. century

D. interval

57. The word "disparate" in line 3 is closest in meaning to .........


A. similar

B. unspoken

C. different

D. unknown

58. The word "neo-classical", as used in the passage, refers to .. .


A. trends in art

B. artistic patterns

C. museum decorations

D. architectural styles

59.As described in the passage. Dibbets used all of the following elements except .
A. pen and ink

B. watercolours

C. architectural photographs

D. still-life drawings

60.What does-this passage mainly discuss?


A. the life of Jan Dibbets
B. new forms of art created by Dibbets
C. an exhibit of abstract art
D. an art that follows the style of Verneer, Van Gogh, and Rembrandt
II. Circle the best answer for each space to complete the passage below.
The day Gerald arrived at the Aimara Hotel, he fell down the stairs. The manager called an
(61) . , but fortunately Gerald's leg was only badly (62). and not broken.
The doctor recommended swimming a further, but gave Gerald a an (64) . for some
tablets in case his legs become painful. The next day, Gerald sunbathed by the pool, and then
took a deep (65) . and dived into the water. There was not very much water in the
pool, and he (66) . one of his arms when he hit the bottom. This time, he complained
to the manager, who sent a special meal to Gerald's room. Later that night, he was suffering
from a (67) . back, the injuries to his arm and leg, and also a terrible stomachache.
He had a high (68) . And fell terrible. Luckily he had the tablets the doctor had
given him. to (69) . the pain. As he reached for them, he fell out of the bed and
broke his (70) He spent the rest of his holiday in bed.
61.

A. stretcher

B. prescription

C. ambulance

D. emergency

62.

A. sick

B. hurt

C. bruised

D. infected

63.

A. medicine

B. cure

C. drugs

D. treatment

64.

A. recipe

B. paper

C. order

D. prescription

65.

A. end

B. breath

C. mouthful

D. water

66.

A. hurt

B. ached

C. injury

D. sore

67.

A. sunny

B. sunburnt

C. sunshine

D. grilled

68.

A. fever

B. degree

C. temperature

D. pain

69.

A. hold

B. check

C. increase

D. reduce

70.

A. waist

B. lips

C. throat

D. wrist

III. Choose the correct preposition to complete the following passage.


THE PSYCHOLOGY OF ACCIDENTS
Most people are (71) ....... the impression that doing something by mistakeis quite different
from doing something (72) ....... purpose. In fact, according to some psychologists, many
accidents do not, the whole, really happen by chance. here may be good reasons (73) .......
actions which seem to be accidental. For example, someone who fails (74) ....... arrive on time
(75) ....... a meeting at work may be worried about his job or her job, or be (76) .......
difficulties at home. (77) ....... other words, there are often

good reasons for behaviour

which seems at first to be accidental. Of course, some people are involved (78) ....... more
accidents than others. These people are called "accident prone". In general they either suffer
(79) ....... stress, or could have a physical illness (80) .......knowing about it.
71. A. in

B. under

C. below

D. of

72. A. to

B. by

C. on

D. with

73.A. to

B. for

C. towards

D. on

74. A. during

B. with

C. to

D. about

75. A. to

B. for

C. at

D. in

76. A. at

B. under

C. with

D. in

77. A. On

B. In

C. By

D. With

78. A. with

B. into

C. in

D. by

79.A. from

B. by

C. off

D. out

80.A. with

B. beside

C. without

D. around

Part D. WRITING
I. Choose the best answer that shows the best way of building sentences from the given cues.
81. Once / young shepherd boy / who / tend / sheep / dark forest.
A. Once a young shepherd boy who was tending the sheep near a dark forest.
A. Once there was a young shepherd boy who tended his sheep near a very dark forest.
B. Once a young shepherd boy who tended sheep in a very dark forest.
C. Once there was a shepherd boy who was going to tend the sheep near a dark forest.

82. feel / lonely / day / he / think / plan / which / get / excitement.


A. Feeling lonely all day makes him think a plan which get excitement.
A. Feeling lonely all day and he thought a plan which gets some excitement.
B. He felt lonely all day and he thought a plan which gets him some excitement.

D. He felt lonely all day so he thought a plan which would get him some excitement..
83. He / rush down / towards /village/ call / "Wolf! Wolf! Wo1f!"
A. He rushed down towards the village and called out "Wolf! Wolf! Wolf!'
A. He was rushing down towards the village and calling "Wolf! Wolf! Wolf!"
B. He had rushed down towards the village and called "Wolf! Wolf! Wolf!"
C. He rushed down and ran towards the village to call "Wolf! Wolf! Wolf!"

84. Hear / villagers / come out / meet / help.


A. Hearing the villagers, he came out to meet and help them.
B. Hearing about this, the villagers came out to meet to help him.
C. Hearing this, the villagers came out to meet and help him.

D. Hearing about this boy the villagers came and met and helped him.

85. This / please / boy / so / much /few days / he / try / same trick.
A. This pleased the boy so much and a few days he tried the same trick.
A. This pleased the boy such much that a few days he tried the same trick.
B. This was pleasing to the boy so much that a few days he tried the same trick.
C. This pleased the boy so much that a few days later, he tried the same trick.

86. Like /previous / villagers / come / help / again.


A. Like the previous villagers came to help again.
A. Like the previous time, the villagers came to help him again.
B. Like him, the previous villagers came to help him again.
B. Liking him, the previous villagers came and helped again.

87. Shortly / after wolf/ actually;' come /forest.


A. Shortly after the wolf, the shepherd actually came into the forest.
B. Shortly after this, a wolf actually came out of the forest.
C. Shortly after this a wolf actually came from the forest.
D. Shortly after this the wolf actually came into the forests.

88. boy/cry "Wolf! Wolf! Wolf!" / help / louder / before.


A.The boy cried "Wolf! Wolf! Wolf!" and helped, louder than before.
B. The boy cried "Wolf! Wolf! Wolf!" and couldn't help shouting louder _than before.
C. The boy cried out "Wolf! Wolf! Wolf!" for help, even louder than before. D. D. The

boy was crying out "Wolf! Wolf! Wolf!" when the other came to help, even louder than
before.
89. But / lime / no one / come / think / boy / deceive / again.
A.But for this time, no one came because they think the boy is deceiving again.
B. But for this time, no one was coming as thinking the boy was deceiving again.
A. But this time no one came to help him thinking that this boy was, deceiving them

again.
C. But this time no one came to help him, because they thought this boy deceiving

them again.
90. wolf / make / good meal / boy's flock / sheep.
A. So the wolf made a good meal off the boy's flock of sheep!
B. The wolf made good meal off the boy's flock of sheep!

C. So the wolf made good meal of the boy's flock of sheep!


D. The wolf made good meal for the boy's flock of sheep!
II. Choose the sentence that has similar meaning to the original sentence.

91. It was quite natural for them to feel hungry by, the evening.
A. It was hard to understand why they felt hungry by the evening.
B. They were not supposed to feel hungry by the evening.
C. We were sorry when they got hungry by the evening.
D. It wasn't surprising they got hungry by the evening.

92. I wish she was going to the party.


A. I hope she'll come to the party.
B. I'm sorry she didn't come to the party.
C. I'm sorry she isn't going to the party.
D. I think that she would come to the party.

93.He asked her if she would meet him that evening.


A. He said to her: "Would you meet me that evening?"
B. He said to her: "Will you meet me this evening?"

C. He said to her: "Are you going to meet me this evening?" fore.


C. He said to her: "Why don't we meet this evening?"
94. When the power failed we went to a restaurant for dinner.
A. We were having dinner in a restaurant when the power failed.
B. The power failed just as we were going out for dinner.
C. There was going to be a power cut, so we went out for dinner.
C. We had dinner at a restaurant because of a power cut.

95.His wife said that it was quite unusual for him to read in bed.
A. His wife said that he was so lazy that he did not read in bed.
B. His wife said that he almost never read in bed.
C. She said that her husband had never read in bed.
D. She said that her husband reads in bed quite often.
96. A bird in the hand is worth two in the bush.
A. One bird in the hand is more important than two other birds.
B. It is better to keep something that you already have than to risk losing it by trying to

get much more.


C. To achieve two things at the same time with one action.
D. There are still many birds in the bush.

97. Many husbands often don't appreciate their wives; and vice versa.
A. Many husbands don't love their wives and their wives don't love them

either.
B. Many husbands are selfish and so are their wives.

C. Many husbands often take their wives for granted but their wives don't.
D. Many husbands and wives often take their spouses for granted.

98. Many people nowadays find it increasingly difficult to exist on the money they earn.
A. Many people are getting less and less money from work, so they don't _

enough to live.
B. The prices are getting higher and people can't live with the money they ea:- C.

Many people nowadays find it difficult to make both ends meet.


D. Many people nowadays do not live on salary.
99. "We are moving to Bristol next week but we promise to keep in touch with you They said
to us.
A. They said that they were moving to Bristol next week and promise to stay contact

with us.
A. They said they were moving to Bristol the following week and would phone or

write letters to us regularly.


B. They said they were moving to Bristol the following week but would come back

to us soon.
C. They said they were moving to Bristol the following week to week touch with us.

100. "I do wish you'd stop biting your nails, Brian. It really annoys me!" Brians mother said
to him.
A. Brian's mother told him to stop to bite his nails because it really annoyed

her.

B. Brian's mother didn't like him to stop biting his nails because it really annoys

her.
C. Brian's mother didn't like him to bite his nails because it got on her
D. Brian's mother didn't like him to bite his nails because it was annoyed.

ENGLISH OBJECTIVE TEST 15


Time allowed: 90 min.
Part A. PHONETICS
I. Choose the Word in each group that has the underlined, italic part pronounced
differently from the rest.
1. A. come

B. love

C. orange

D. company

2. A. dread

B. bead

C. thread

D. threaten

3. A. spear

B. bearing

C. rearing

D. gear

C. weigh

D. neighbourhood

4. A. freight B. weird

5. A. abroad B. hand

C. astronomy

D. acquainted

II. Find the word with the stress on the SECOND syllable in each line.
6. A. committee B. referee
C. elegant
D. refugee
7. A. energy

B. specific

C. dishwasher D. misbehave

8. A. alcohol

B. chemical

C. effective

D. president

9. A. competition

B. television

C. repetition

D. adjustment

10. A. accuracy B. delicacy

C. deliberate

D. literacy

Part B. LEXICO - GRAMMAR


I. Circle the best answer which best fits in the blank in each of the following
sentence.
11. I'd rather you . in here. There's a room for smokers out there.
A. not smoke B. don't smoke
C. didn't smoke D. won't smoke
12. Tony is so that his friends tend to tell him all their problems.
A. confidential B. optimistic

C. sympathetic

D. critical

13. During the starvation, we used to keep a .. of dried food in the factory.
A. substitute

B. reserve

C. preserve

D. conserve

14. We . our newly-married friends a happy life.


A. hoped

B. prayed

C. wished

D. cherished

15. You should take these tablets in order to your pain.


A. increase
16.

B. dislike

C. mind

D. object

My brother usually ....... at cards. it's no surprise that he wins every game.
A. misleads

18.

C. avoid D. decrease

Do you ...... my turning the television on now?

A. disapprove
17.

B. relieve

B. lies

C. deceives

D. cheats

He was too sure of himself to pay ........... to the warnings of the man from Sulphur
Creek.
A. notice

B. attention

C. respect

D. recognition

19. His brother is going to . a minor operation.


A. recover

B. suffer

C. undergo

D. withstand

20. She is totally . in her detective novels.


A. absorbing

B. absorbent

C. absorbed

D. absorptive

21. Before the product is put on the market, the company must whether it complies
with safety standards.
A. ask

B. calculate

C. ascertain

D. argue

22. A university education is of course important but it is essential that all employees undergo

a period of intensive
A. preparation B. concentration

C. learning

D. training

23. Surgeons use a to perform an operation.


A. knife

B. surgery knife

C. sword

D. scalpel.

24. Now that they have read it in they believe me.


A. black and white B. white and black C. red and blue D. blue and red
25. You can't a thing that woman says!
A. believe

B. rely

C. count

D. imagine

26. This job requires a good for figures.


A. head

B. understanding C. brain

D. faculty

27. As I won't he able to attend the meeting, I'd like you to sign
A. on my place B. on my behalf

C. on my name

D. on my account

28. The amount of tax you pay is to your income.


A. based

B. assessed

C. measured

D. proportionate

29. There' no in applying for the job unless you have right qualifications
A. point

B. reason

C. use

D. worth

30. The boss to his secretary using the office phone for personal calls.
A. disapproves B. criticizes

C. disagrees

D. objects

II. Choose the right word to complete the following English idioms.
31. It's as silent as in here!
A. Sunday

B. the grave

C. death

D. rice

32. I couldn't believe that they were brothers. They were as different as ..
A. Mars from Jupiter

B. milk from honey

C. chalk from cheese

D. dogs and cats

33 No matter what happens, Colin always seems to remain as cool as


A. an iceberg

B. a cucumber

C. cold feet

D. an ice-cream

C. a mountain

D. an elephant

34. David is as strong as


A. a gorilla

B. a horse

35. Fido is as gentle as , especially with children.


A. a pony

B. snowflakes

C. a lamb

D. an angel

III. Choose the correct form of the verb for each blank.
We (36) to a new house in the country last week. And for the last few days, we
(37) to organize ourselves in .our new home. Yesterday, my three brothers and
I (38) painting the downstairs rooms. Unfortunately, while I (39)
the paint, one of my sisters (40) The door. Nobody (41) her that we

(42) in the room, so instead of (43) the walls, we spent all morning
(44) the paint off the floor. But worse things (45) since then. This morning
when I (46) through the, water (47) ceiling next to my bed. We (48)
today so far repairing the roof. It's not all bad news, though. The school in the
village nearby (49) two years ago, and my parents (50) another
school for us yet.
36.

A. moved

B. had moved

C. had been moving

D. would move

A. tried

B. have been trying

C. have tried

D. are trying

A. had been starting

B. had started

C. started

D. was starting

39.

A. mixed

B. had been mixing C. had mixed

D. was mixing

40.

A. opened

B. had opened

C. would open

D. was opening

41.

A. told

B. had told

C. was telling

D. has told

42.

A. had been

B. were

C. would have been D. would be

43.

A. paint

B. painting

C. having painted D. being painted

44.

A. cleaning

B. having cleaned C. are cleaning

45.

A. have happened

B. are happening

C. has happened

D. happened

A. was waking up

B. woken up

C. woke up

D. was woken up

A. dripped

B. had dripped

C. had been dripping

D. was dripping

A. have spent

B. spent

C. have been spending

D. spend

A. would be closed

B. had closed

C. closed

D. was closing

A. haven't found

B. haven't been finding

C. didn't find

D. hadn't found

37.
38.

46.
47.
48.
49.
50.

D. to clean

IV. -choose the correct word form for each gap to complete the passage. WHO
DOES THE HOUSEWORK?
it's fairly common belief in Britain that men these days do more housework than they
did in previous (51) But is it really so? A recent survey has made some interesting

discoveries. (52) When men do help out, they enjoy shopping and cooking but most
are (53) to do the washing. A quarter of men think that women are better (54)
to looking after the after the home than men and 19% admit to making no (55) to
housework. The average man says that he does (56) of the housework, while the
average woman says she does three-quarters of it, so someone isnt telling the (57) It
was impossible to find any men who shared housework (58) with their partners.
Perhaps it is not so (59) therefore, that there are some (60) who want a new
law forcing men to do their share.
51.

A. generation

B. generalization C. generates

D. generality

52.

A. Apparent

B. Apparently

C. Apparentially

D. Unapparent

53.

A. willingly

B. unwillingly

C. unwilling

D. willing

54.

A. suitable

B. unsuitable

C. unsuited

D. suited

55.

A. contribution B. contributor

C. contributive

D. contributory

56.

A. more

B. most

C. much

D. many

57.

A. truly

B. truthfulness

C. truth

D. trueness

58.

A. equality.

B. unequally

C. inequality

D. equally

59.

A. surprising

B. surprised

C. surprise

D. surprisingly

60.

A. policies

B. politicians

C. politicizations

D. politician

V.

Decide the underlined word or phrase (marked A, B, C, D) in each


sentence that needs correcting.

61. The major part of the temple had been converted towards Lenin's room.
A

62. In a few seconds a computer can solve problems that might take a man years
A

to solve solo.
D
63. In the beginning radios cost much more than they do today because they
A

turned out slowly and expensively by hand.


C

64. Because advertising encourages us to buying and producing more things, it is sometimes
A

called the park plug of the business world.


D

65. Computers are wonderful proof of how clever people are in working out ways of
A

helping themselves.
D
66. A pharmacists ethical standards have to be high because he is entrusted to the
A

storage and distribution of dangerous drugs.


D
67. There are warm tropical regions all over the globe, but only the Indians of the
A

South America rain forests have adopted the hobby of sleeping in the air.
C

68. The painting was so beautiful that I stood there admired it for a long time.
A

69. One of the features of London is the number of big stores, most of them are to be
A

found in or near the West End.


D
70. All of my students are longing for having a nice summer holiday
A

at the seaside.
D
Part C. READING
I. Circle the best answer for each space to complete the passage below.
Childhood is the time when there are (71) responsibilities to make life
difficult. If a child has good parents, he is (72) , looked after and loved,
whatever he may do. It is improbable that he will ever again in his life be given so much
without having to do anything in return. In (73) life is always presenting new things to
the child-things that have lost their interest for older people because they are too well-known.
A child finds pleasure Playing in the rain, or in the snow. His first visit to the seaside is a
marvelous adventure. But a child has his (74) He is not so free to do as he wishes as
he thinks older people are: he is continually told not to Do things, or being punished (75)
.......... what he has done wrong. His lifes therefore not perfectly happy.

When a young man starts to (76) his own living, he becomes free from the
discipline of school and parents; but at the same time he is forced to accept responsibilities.
He can no longer expect (77) to pay for his food, his clothes and his room, but he has to work
if he wants to live comfortably. If he spends most time playing about in the way he used to as
a child, he will be hungry. And if he breaks the laws of society as he used to break the
laws of his parents, he may go to prison. If, however, he works hard, keeps out of trouble and
has good health, he can have the greatest happiness seeing himself make steady (78)
in his job and of building up for himself his own position in society.
Old age has always been thought of as the worst age to be, but it is
not necessary for the old to be unhappy. With old age (79)

wisdom

and the ability to help others with advice wisely given. The old can have
the joy of seeing their children making progress in life; they can watch
their grandchildren growing up around them; and perhaps best of all,
they can, if their life has been a useful one, feel the happiness of having
come (80) the battle of life safely and of having reached a time
when they can lie back and rest, leaving others to continue the fight.
71.

A. little

B. a few

C. a little

D. few

72.

A. eaten

B. fed

C. feeded

D. fetched

73.

A. short

B. term

C. addition

D. reality

74.

A. injury

B. difficulty

C. problem

D. pain

75.

A. because

B. by

C. at

D. for

76.

A. make

B. have

C. create

D. earn

77.

A. other

B. another

C. others

D. someone

78.

A. progress

B. achievement

C. improvements D. accomplishments

79.

A. coming

B. come

C. came

D. has come

80.

A. out

B. across

C. through

D. back

II. Read the passage and choose the correct answer A. B, C or D for each question.
Getting good results in your studies comes from developing good study habits. The best
place to study is a comfortable room with good lighting. The best chair for studying
should be one which you would be comfortable in, but not so comfortable that you may
fall asleep in it after a while! Before you study, look for a quite environment with no
distractions. If you find your home too noisy for studying, try the library or community
centre instead. When you study, set realistic targets for yourself to achieve. For example,
do not aim to study five chapters of Geography in one sitting when you know that you

take more than an hour to read one chapter. Furthermore, remember not to push yourself
too hard. When you have studied for a period of time, reward yourself by taking a short
break. You could perhaps take a short walk or listen to some music for a while.
If you begin to feel sleepy when studying, do not force yourself to go on as you may lose
your power of concentration. Instead, you may want to take a short nap to refresh
yourself. Studying may also be strenuous on the eyes. When you study, it may be a good
idea to lift your eyes away from your books occasionally. Looking at objects at a distance
or at greenery can help soothe tired eyes.
81 . The first thing to be considered in developing good study habits is.........................
A. your ability to study well
B. the best chair to sit for studying
C. environment for studying
D. your attitude towards good study at home

82. Which of the following does not improve your good study habits?
A. a quite with enough lighting
B. an extremely comfortable chair
C. a proper chair and a right-sized table
D. a public or college library

83. When studying, you'd better...........................


A. learn as Much as you want in one sitting
B. learn either sitting at the table or standing
C. take a short break after every chapter
D. set goals that are reachable for you

84. Taking a short nap is necessary when


A. you feel sleepy and cannot concentrate on studying
B. your eyes become red after studying
C. you cannot lift your eyes away from your books
D. you want to go on after getting your concentration

85. The most appropriate title for the passage may be.......................
A. How to use your study time efficiently
B. How to build up good study habits
C. How to get relaxed when studying
D. How to avoid sleepiness when studying

Part D. WRITING

I. Choose the best answer that fits each blank in the following sentences.
86. Hardly.......................someone rang the doorbell.
A. had I gone to bed then

B. had I gone to bed when

C. had I gone to bed than

D. had I gone to bed and

87. Only after seeing Hamlet on the stage.........


A. then I understood it

B. did I understand

C. when I understood it

D. did I understand it

88. It was sunny but he still brought with him a raincoat. - How funny! He
A.

can't have brought it with him

B.

needn't have brought it with him

C.

shouldn't have brought it with him

D.

didn't have to bring it with him.

89. Despite Ann's illness..........


A.

she insisted on going to work.

B.

but she insisted on going to work.

C.

however, she insisted on going to work.

D.

when she insisted on going to work.

90. The wallet I lost last week was found by a workman..........


A. who was digging a hole in the street outside the house.
B. from whom was digging a hole in the street outside the house
C. whom was digging a hole in the street outside the house.

91. D. whose digging a hole in the street outside the house.


....... the workers will stop working.
A. If the salary is doubled
B. Unless the salary isn't doubled
C. If not the salary is doubled
D. Unless the salary is doubled

92. The water..........


A, was too cold to swim in it.
B. was so cold that we can't swim.
C; was so cold that we couldn't swim in it.
D. was not enough cold to swim. 93. ........ all night.
A. I am not used to studying

C. I used to studying

B. I am not used to study

D. I use to study

94. ......... its accuracy, laser is very useful in medicine.


A. In view of

B. As a result of

C. Thanks to

D. Despite

95. Once almost untreatable, ..........


A. birthmarks can now be easily removed by laser operation
B. but now birthmarks can now be easily removed by laser operation
C. people can remove birthmarks by using laser treatment
D. but now people can remove birthmarks by using laser treatment

II.

Choose the sentence that has similar meaning to the original sentence.

96. Most people would be pleased if they got a windfall.


A. Most people are happy to get an amount of money unexpectedly.
B.

Most people are pleased if they have a chance to experience a windy storm.

C.

Most people like windy days in autumn.

D. Most people are happy to receive a wind in autumn.

97. My cousin is extremely tight-fisted.


A.

He is very strong.

A.

He doesn't want to spend his money.

B.

He is very thin.

C.

He likes wearing tight clothes.

98. The concert went like a bomb.


A.

The concert was a disaster.

B.

There was a bomb in the concert.

C.

The concert was very violent.

D.

The concert was extremely successful.

99. I think that Dorothy is making a mountain out of a molehill. A: Dorothy is


trying to gain success in her job.
B. Dorothy is creating as mountain from a hill.
C. Dorothy is making an unimportant matter seem important.
D. Dorothy is very careful.

100. He was born with a silver spoon in his mouth.


A. He was very sick when he was born.
B. His parents put a silver spoon in his mouth when he was born.
C. His parents were very rich.
D. His parents were jewelers.

ENGLISH OBJECTIVE TEST 16

Time allowed: 90 min.


Part A. PHONETICS
I. Choose the word with the underlined part pronounced differently from the others in each
1. A. hasty

B. tasty

nasty

D. waste

2. A. myth

B. with

C. both

D. tenth

3. A. statue

B. due

C. rescue

D. Tuesday

4. A. bought

B. fought

C. sought

D. drought

5. A. humid

B. fume

C. stupid

D. tunnel

II. Choose the word with the position the stressed syllable different from that of the
others in each line.
6. A. architect

B. festival

C. hesitate

D. hospitable

7. A. regularly

B. humidity

C. ability

D.

8. A. refusal

B. conductor

C. introduce

D. conrusing

9. A. indifferent

B. individual

C. gymnasium

D. facility

10. A. traveling

B. measuring

C. telegraph

D. exhaustion

fidelity

Part B. LEXICO GRAMMAR


I. Choose the correct form of the verbs among A, B, C or D to complete each of the
following sentences.
J: Hello, I (11) . you for ages.
M: No, it must be about two years since we last met. What (12) . with
yourself'?
J: I (13) .......... all sorts of things. Life (14) ........ very busy lately. 1(15) ..........

new job at the sailing centre.


M: Really! Doing what?
J:

Well. I (16) .............. beginner's courses to school groups this term but when
those (17) . I will be teaching more advanced groups on holiday courses.

M: That sounds great. What about your old job at the tourist office then?
J:

Well, I (18) ........... there for ten years and I quite (19) the job but I (20)
. to do something different. I have always enjoyed sailing in my spare
time so I thought it would be nice to do it for a job.

M: So how long (21) ........... at the sailing centre?


J:

For about two months now. I am sure I (22) ........

the

right

thing. It's

really an

enjoyable job. And what about you?


M: Well I am still at the Rembrandt Hotel. I've been working, there for fifteen years now.

But I (23) ........ a promotion so that's good. They (24).me the job of head
receptionist.
J:

Oh, that's good news.

M: Yes, it has made me feel much better about work. I mean, 1 (25) .
different jobs at the hotel but I've never had a job with this much responsibility before.
11. A. haven't seen

B. didn't see

C. don't see

12. A. are you doing

D. haven't been seeing

B. do you do

C. have you done

D. have you been doing

13. A. am doing

B. do

C. have done

D. have been doing

14. A. has been

B. is

C. was

D. has been being

15. A. started

B. have started

C. am starting

D. start

16. A. taught

B. have taught

C. am teaching

D. have been teaching

17. A. finished .

B. have finished

C. finish

D. am finishing

18. A. have worked

B. have been working

C. worked
19. A. have enjoyed
C. enjoyed

D. had worked
B. have been enjoying
D. had enjoyed

20. A. want

B. wanted

C. have wanted

D. am wanting,

21. A. were you

B. are you

C. have you been

D. had you been

22. A. did

B. do

C. am doing

D. have done

23. A. have

B. have had

C. am having

D. had

24. A. give

B. am giving

C. have given

D. have been giving

25. A. do

B. am doing

C. have done

D. have been doing

II. Choose the most suitable word form among A, B, C or D to complete the passage.
There is no doubt that a common language used throughout the world would do much to
bring countries (26) .. to each other. Though it is becoming (27) easy to move
from place to place, our (28) .. to communicate with one another gives rise to (29)
.. (30) .. and makes real contact between people of different nationalities (31)
.. . Many attempts have been made to overcome this problem and they have all failed.
The fear of foreign influence and (32) .. rules out the universal (33) .. of
anyone of the (34) .. major languages. (35) .. of this difficulty, many linguists
have constructed artificial languages which could have no possible political overtones. They
have argued that a language of this sort would perform much the same service as Latin did in
Middle Ages.
26. A. closely

B. closed

C. closer

D. close

27. A. increasely

B. increasingly

C. increased

D. increasing

28. A. ability

B. disability

C. inability

D. unability

29. A. numerous

B. numberous

C. numbered

D. numeral

30. A. understanding

B. understand

C. understandingly D. misunderstanding

31. A. impossible

B. possible

C. impossibility

D. possibilities

32. A. dominating

B. domination

C. dominate

D. dominations

33. A. acception

B. accepting

C. acceptance

D. acceptation

34. A. existence

B. existed

C. existent

D. existing

35. A. Aware

B. Awaring

C. Unaware

D. Unawaring

III. Choose the preposition among A, B, C or D which best fits each space in the
following passage.
Vincent Van Gogh. who was born in Holland in 1853, is one of the world's most famous
painters. Although his talent was unrecognized (36) .. his life, it was much
appreciated (37) .. his death.
After he had failed (38) .. every career he had attempted, Van Gogh first turned (39)
.. art to express his strong religious feelings. (40) .. he had decided to become
a painter in (41) .. 1880, he started to paint peasants and miners. During the next few
years, which are known as his 'Dutch period', he produced paintings (42) .. rather dark
greenish-brown colours. In 1886, when he went to Paris to visit his brother Theo, he was
immediately attracted (43) .. the Impressionist work he saw there. He decided to stay
in Paris and continued his paintings there. He was encouraged (44) . Picasso to use
more colours (45) his pictures and his subsequent paintings were bright and
immensely colourful.
36.

A. in

B. throughout

C. with

D. through

37.

A. with

B. before

C. after

D. from

38.

A. in

B. down

C. from

D. at

39.

A. to

B. for

C. into

D. with

40.

A. After

B. Before

C. During

D. Next

41.

A. over

B. about

C. after

D. before

42.

A. with

B. by

C. for

D. in

43.

A. with

B. for

C. to

D. toward

44.

A. from

B. of

C. at

D. by

45.

A. with

B. on

C. in

D. to

IV. Choose the correct option which could best complete each of the following Sentences.

46. How many times did the clock


A. hit .

B. beat

C. knock

D. strike

47. I know for ...... that he was away from home last night.
A. true

B. certain

C. right

D. exact

C. received

D. inherited

48. Sue....... great fortune from her parents.


A. offered

B. had

49. Your personality is important ...... the success of your work.


A. for

B. in

C. to the beach.

D. on

50. It was . everyone went to the beach


A. such hot a weather so

B. so hot weather that

C. such hot weather that

D. such a hot weather that

51. Grace advised us to withdraw . .


A. so as to get not involved B. so as not to get involved
C. as not to get involved

D. as to get not involved

52. The label failed to tell what ..


A. in the can was

B. was in the can

C. was the contents

D. the contents of the can was

53. He never had much property, and when his house was broken into by thieves, he had
. still.
A. less

B. little

C. a little

D. few

54. The machines often broke down because the staff were not . careful.
A. sufficient

B. enough

C. sufficiently

D. too

55. We had to do . to reach Boston.


A. five at least more miles.

B. at least five more miles

C. five miles at least more

D. more miles at least five

56.My car . 25 miles to the litre.


A. makes

B. does

C. needs

D. goes

57.Clearing the weeds from the churchyard was a much harder . than the children had
imagined it would be.
A. deed

B. task

C. service

D. work

C. serious

D. severe

58. David has a . accident.


A. heavy

B. strict

59. You are welcome . any books in the library.


A. to

B. with

C. for

D. by

C. started

D. exploded

60. Several shouts were in the night.


A. opened

B. fired

V. Identify the underlined word or phrase A, B, C or D that would not be accepted in


standard English.
61. The second and the third lessons were equally difficult.
A

62.Elephants can consume a large number of food.


A

63. It occurred to him that he didnt eat anything since the night before.
A

64. They had to queue outside the theatre for getting a taxi.
A

65. One of the majority causes of tides is the gravitational attraction of the moon.
A

66.He has just been operated and I think he will get well soon.
A

67. Her friend is a really young handsome man with curly hair and blue eyes .
A

68.He often goes to work by bus but today he came by his friend's car.
A

69.Is it truly that many yeas ago he had his portrait painted by an unknown artist?
A

70.The Planning Department wanted to change the traffic system so that they
A

conducted a survey.
D
Part C. READING
I. Choose the Word which best fits each numbered space in the following passage.
No (71) to Britain should go home without (72) Some time in Stratford-uponAvon.

(73)

in

the

heart

of

England,

it

is

town

rich

both

in history and in culture.


Stratford is a busy market town with a population of twenty-three thousand, and (74)
many trees and buildings (75) since medieval times. It is surrounded by some of the
prettiest countryside in England, and is an ideal base for those (76) to visit such places
of interest as Warwick Castle, or the beautiful modern cathedral in Coventry.
Without (77) , Stratford is (78) known as the town where the
playwright William Shakespeare was born and died. 1-lere you can visit his birthplace and
other buildings (79) with his family. These houses are all splendid examples of Tudor
architecture, and there arc many other historic buildings well worth (80) in the town.
71.A. visitor

B. people

C. one

D. body

72.A. staying

B. spending

C. wasting

D. doing

73. Situating

B. To situate

C. Laying

D. Situated

74.A. holds

B. consists

C. contains

D. composes

75.A. unchanging

B. unchangeable

C. unchanged

D. changeable

76.A. wish

B. wishing

C. like

D. liking

77. A. saying

B. suspicion

C. word

D. doubt

78.A. well-known

B. best

C. more

D. most

79.A. associated

B. connecting

C. associating

D. linking

80.A. visit

B. going

C. visiting

D. watching

II. Read the following passage and choose the correct answer for each question,
With the onset of the winter season, man's natural enemies, the common cold and the flu,
arrive with full force. It seems that the fluctuations in temperature and weather are guarantees
that coughs and sneezes will spread infecting germs among family and friends.
More than 100 different types of bacteria can cause a cold, and doctors sometimes use
antibiotics to treat bacterial colds. However, there are an equal number of viruses that can
cause influenza, and modern science offers no drugs capable of curing viral infections. In
most cases, the best advice is the usual prescription: get plenty of rest, drink a lot fluids, and
be prepared to suffer for three to ten days.
Some home cures help to relieve the symptoms of colds and flu. Mother's chicken soup, rich
in fats and oils, helps to revitalize a tired body and to soothe a sore throat. Garlic, containing
the active ingredient Alicia, has long been used to fight off the effects of bacteria and viruses.
Hot toddies consisting of small amounts of liquor mixed with honey, sugar, and lemon juice
can relieve soreness and draw out cold germs. Finally, recent evidence suggests that large
doses of vitamin C not only boost the immune system before a cold arrives, but also relieve

cold symptoms after they have set in.


81. What is the main idea of this passage?
A. Man's natural enemies.

B. The nature of colds and flu.

C. Fluctuations in temperature.

D. The onset of winter.

82. It can be inferred, from the passage that germs are spread______
A. through the air.

B. only in winter weather.

C. when the temperature is high.

D. with great force.

83. The word "infecting" in the passage can best be replaced by______
A. escaping

B. unavoidable

C. contaminating D. debilitating

84. It can be inferred from the passage that antibiotics_______


A.

have been on the market for a short time.

B.

may kill beneficial organisms

C.

are not yet effective on bacterial colds.

C.

cannot be used to treat influenza.

85. According to the passage, most cases of the flu


A. are intensified by drinking too many fluids.
A. last several days.
B.

are not caused by a lack of rest.

B.

result in excessive suffering.

Part D. WRITING
I. Choose one sentence among A, B, C or D which is closest in meaning to the original
sentence.
86. I am fed up with his behaviour.
A. I have enough of his behaviour.
A. I've had enough of his behaviour.
B. I've had enough for his behaviour.
B. I've had his behaviour enough.

87. Why don't you telephone me later to discuss this?


A. Why do not you give me a call later to discuss this?
A. Why don't you give me a call later to discuss this?
B. Why don't you give me a call later to discuss about this?
B. Why not giving a call later to discuss this?

88. Although I warned them not to climb that tree, they did.
A. Despite of my warning, they climbed that tree.

A. Despite my warning, they climbed that tree.


B. Despite my warning, they did climbed that tree.
C. Although my warning, they climbed that tree.

89. The person who encouraged me the most at school was my French teacher.
A. The person who gave me most of encouragement at school was my teacher.
B. The person who gave me most encouragement at school was my teacher.
C. The person who gave me the most encouragement at school was my teacher.
D. The person who gave me encouragement the most at school was my teaches.

90. I don't really think that he will accept our invitation..


A. It is unlike for him to accept our invitation.
B. It is unlikely that he accepts our invitation.
C. He will be unlikely to accept our invitation.
D. He is unlikely to accept our invitation.

91. People heard him make an interesting remark.


A. What an interesting remark did he make!
B. He was heard to make an interesting remark.
C. He was heard make an interesting remark.
D. He was heard making an interesting remark.

92. He is an outstanding leader and everyone knows him.


A. He is known an outstanding leader.
B. So. outstanding is he a leader that everyone knows him.
C. He is such an outstanding leader that everyone knows him.
D. Everyone knows him for outstanding leading.

93. The store was opened by Mark and his brother 10 years ago.
A. It was 10 years ago when the store was opened by Mark and his brother.
B. They were Mark and his brother who opened the store 10 years ago.
C. It was 10 years ago that the store was opened by Mark and his brother.
D. Mark and his brother were the only one who opened the store 10 years ago.

94. People say that he did it for his own sake.


A. He is said to do it for his own sake.
B. lie is said that he did it for his own sake.
C. He is said to have done it for his own sake.
D. It is said to do it for his own sake.

95. Could you help me to post these letters?


A. Would you be kind to post these letters for me?
B. Would you be so kind as to post these letters for me?

C. Do you mind to post these letters for me?


D. Would you mind posting me these letters?

II. Choose one sentence among A, B, C or I) that is hest written based on the words
provided.
96. You / so kind / complete / return / enclosed form / delay?
A. Would you be so kind as to complete and return the enclosed form without delay?
B. Would you be kind to complete and return the enclosed form without delay?
A. Would you be so kind so as to complete and return the enclosed form without

delay?
C. Would you so kindly complete and return the enclosed form without delay?

97. I /gratefud /you / send / latest catalogue / price lists.


A. I am grateful if you send me the latest catalogue of price lists.
A. I would be grateful to you to send me the latest catalogue of price lists.
B. I would be grateful if you could send me the latest catalogue of price lists.
C. I would be grateful if you could send to me the latest catalogue of price lists.

98. That hat/ terrible / you! / look / idiot!


A. That hat is terrible with you! You look like an idiot!
B. That hat is terrible to you! You look like an idiot!
C. That hat is terrible for you! You look like an idiot!
D. That hat is terrible on you! You look like an idiot!

99. She / look / pale when /arrive. /She /look / see a ghost.
A.

She looked so pale when she arrived. She looked like she had seen a ghost.

A. She looked so pale when she arrived. She looked as if she had seen a ghost.
A. She looked so pale when she arrived. She looked as she had seen a ghost.
B.

She looked so pale when she arrived. She looked as if she saw a ghost. 100.

100. I write / complain /play /perform / your theatre last week.


A. I am writing to complain the play that was performed at your theatre last week.
B. I am writing to complain about the play performing at your theatre last week.
C. I am writing to complain about the play, which was performed at your theatre last

week.
A. I am writing to complain about the play that was performed at your _______

last week.
ENGLISH OBJECTIVE TEST 17

theatre

Time allowed: 90 min.


Part A. PHONETICS
I. Choose the word with the underlined part pronounced differently from the others in
each line.
1. A. great

B. bread

C. break

D. steak

2. A. fan

B. bad

C. wash

D. catch

3. A. chair

B. cheap

C. chorus

D. champion

4. A. tomb

B. home

C. comb

D. only

5. A. hire

B. horror

C. heir

D. hair

II. Choose the word with the position of the stressed syllable different from that of the
others in each line.
6. A. organism

B. independent

C. presentation

D. indication

7. A. mystery

B. flourishing

C. eternal

D. calendar

8. A. astonish

B. forgetful

C. industrious

D. militant

9. A. reputation

B. contribution

C. unemployment

D. favourable

10.A. hostage

B. damage

C. reply

D. physics

Part B. LEXICO - GRAMMAR


I. Choose the correct form of the word A, B, C or D to complete each of the following
sentences.
11. You can't export goods without an export .
A. permitted

B. permission

C. permit

D. permitting

12. You risk .. your money when you put it into that business.
A. to lose

B. losing

C. loosing

D. to loose

13. Frank told everyone that he worked for a large company, but the company is .
A. non-existed

B. non-existent

C. unexisting

D. inexistent

14. He .. me not to go out alone at night.


A. cautiouses

B. cautioused

C. cautions

D. cautious

15. The .. was so exciting and absorbing that she could not put the book down.
A. thrill

B. thrilling

C. thrillness

D. thriller

16. The doctor gave a very .. report on my health.


A. favour

B. favourite

C. favourable

D. favoured

17.He turned on the record-player, producing a crash of .. sound.


A. electric

B. electrical

C. electrified

D.

18.They have carried out a .. into the causes of brain damage

A. research

B. searching

C. search

D. searchment

19.This home provides .. care for the elderly.


A. residence

B. resident

C. residential

D. residental

C. requesting

D. require

20.Single rooms are available on ..


A. request

B. requirement

II. Choose one preposition among A, B, C or D which best fits the space in each of the
following sentences.
21. I was delighted .. the present you gave me.
A. of

B. about

C. with

D. at

22. I can't understand people who are cruel .. animals.


A. with

B. to

C. of

D. for

23.We enjoyed our holiday, but we were a bit disappointed ...... the hotel.
A. at

B. in

C. with

D. by

24. I was surprised .. the way he behaved. It was completely out of character.
A. at

B. with

C. of

D. for

25.The people next door are furious .. us for making so much noise last night
A. to

B. at

C. about

D. with

26. He has just dropped his watch from the third floor and it has fallen .. the flower
bed below.
A. down

B. off

C. on

D. to

27. "Is Thailand .. the west of Vietnam?" "Yes, it is."


A. on

B. to

C. at

D. in

28. I am very impressed her English. It's very good.


A. with

B. of

C. for

D. to

29. I am sorry the smell of paint in this room. I've just decorated it
A. about

B. for

C. with

D. by

30. He didn't trust me. He was suspicious my intentions.


A. by

B. to

C. of

D. for

III. Choose the answer (A, B, C or D) which best fits the space in each of the following
sentences.
31. I remember the letter a few days before going on holiday.
A. to receive

B. to have received

32. She had to pass all her exams or


A. instead

B. else

C. received

D. receiving

she would have no holiday.


C. therefore

33. I can't tell Peter and Bill apart. They are twins.

D. though

A. similar

B. alike

C. resemblance

D. identical

34. People believe there is a between the two crimes.


A. join

B. chain

C. link

D. connector

35. If you can't find what you want in this chapter, look it up in the
A. reference

B. index

C. catalogue

D. directory

36. Mr. Duncan does not know the lawn mower after they had finished using it.
A. where did they put

B. where they did put

C. where they put

D. where to put

37. The teacher suggested that her students experiences with ESP.
A. write a composition on their
B. to write composition about the
C. wrote some compositions of his or her
D. had written any compositions for his
38. Of the two new teachers, one is experienced and .
A. the others are not

B. another is inexperienced

C. the other is not

D. other lacks experience

39. Transitory pains are those that come go quickly, rarely return, and signify
nothing.
A. however

B. instead

C. not

D. an

40 Wilbur Wright flew his plane France in 1909.


A. on

B. upon

C. until

D. over

41. There was a bad accident last night. One driver was killed, the other driver was badly
injured, and both cars were badly
A. damaged

B. destroyed

C. broken down

D. smashed

42. The World Wide Web is a large part of the Internet; it is a system of electronic documents
to one another.
A. related

B. linked

C. joined

D. bound

43. If you carefully, you can see how he does the card trick.
A. see

B. watch

C. glance

D. witness

44. A program where a host talks to famous people about their lives and careers is a/an

A. live show

B. interview

C. talk show

D. quiz show

45. He's a His first play was performed in a very small theatre.
A. writer

B. playwright

C. composer

D. novelist

IV. Choose the form of the verb which best fits each space in the following passage.

In mankind's millions of years on earth, many types of energy resources (46) However, in
the last ninety years, petroleum (47) by far the most important. Accounting for over
50 percent of all energy consumed, it (48) so essential that, without petroleum, we
(49) with economic disaster.
Of course, the earth's supply of petroleum (50) forever, and in fact, it (51)
sooner than most people (52) At the current rate of consumption, the world's proven oil
reserves (53) within forty to fifty years.
Obviously, measures (54) decades ago to reduce our dependence on oil.
Amazingly, such measures (55) today.
46. A. were utilized B. have been utilized
C. have utilized

D. utilized

47.A. became

B. had become

C. has been becoming

D. has become

48. A. is

B. has been

C. is being

D. had been

49. A. would be faced

B. would face

C. will be faced

D. will face

50. A. does not last

B. will not last

C. will not have lasted D. not lasts

51. A. will disappear

B. has disappeared

C. will be disappearing
52. A. have believed

D. disappears
B. will believe

C. believed

53. A. may exhaust

D. believe

B. may be exhausted

C. may be exhausting D. may have exhausted


54. A. should've taken

B. should take

C. should've been taken

D. should be taken

55. A. are still discussed B. are still being discussed


C. are still discussing D. have still been discussed
V. Decide which of the underlined part is not acceptable in standard English by
circling the letter A, B, C or D.
56. We all are slowly destroying the earth. The seas and rivers are too dirty to
A

57. swim in. There is so much smoke in the air that it is healthy to live in many
A

58. of the world cities. In one well-known city, for example, poisonous gases of
A

C D

59. cars pollute the air so much that traffic policemen have to wear oxygen mask.
A

60. We have cut down so many trees that there now are vast areas of wasteland
A

61. all over the world. As a result farmers in parts in Africa cannot grow enough
A

62. to cat. In certain countries in Asia, there is so little rice. Moreover, we do not
A

63. take enough care of the countryside. Wild animals are quickly disappeared.
A

64. For instance, tigers are rare in India now although we have killed too many for
A

65. them to survive. However, it isn't enough simply to talk about the problems.
A

We must act now before it is too late to do anything about it. Join us now. Save the earth. This
is too important to ignore.
Part C. READING
I. Read the passage below and decide the word that best fits each space.
The food we eat seems to have profound (66) . on our health. Although
science has made enormous (67) . in making food more fit to eat. It has at the same
time, made many foods unfit to eat. Some research has (68) . that perhaps eighty
percent of all human illnesses are related to diet and forty percent of cancer is related to the
diet as well, (69) . cancer of the colon. People of different cultures are more Drone
to (70) . Certain illnesses because of the characteristic foods they consume.
That food is (71) to illness is not a new discovery. In 1945, government
researchers realized that nitrates and nitrites (commonly used to (72)

colour

in meats) as well as other food additives caused cancer. (73) . these carcinogenic
additives (74) . in our food, and it becomes more difficult all the time to know
which ingredients on the (75) . labels of processed food are helpful or harmful.
The additives that we cat are not all so direct. Farmers often give penicillin to cattle and
poultry, and because of this, penicillin has been (76) . in the milk of (77) . cows.
Sometimes similar drugs are administered to animals not for medicinal purposes, but for
financial reasons. The farmers are simply trying to (78) . the animals in order to (79)
. a higher price on the market. Although the Food and Drug Administration (FDA)
has tried (80) . to control then procedures, the practices continue.
66. A. affect
67. A. steps
68. A. stated

B. effect
B. things
B. said

C. effects
C. advance
C. expressed

D. affects
D. technique
D. shown

69. A. specially
70. A. contract
71. A. connecting
72. A. conserve
73. A. Yet
74. A. remain
75. A. package
76. A. seen
77. A. treating
78. A. fattening
79. A. obtain
80. A. repeatly

B. especially
B. meet
B. associated
B. protect
B. However
B. stay
B. packaging
B. found
B. treatable
B. fatten
B. profit
B. repeatingly

C. generally
C. infect
C. attached
C. preserve
C. But
C. exist
C. packaged
C. tasted
C. treated
C. fat
C. matte
C. repeatedly

D. especial
D. affect
D. related
D. keep
D. Although
D. contain
D. packet
D. smelled
D. untreated
D. fatter
D. earn
D. repetition

II. Read the following passage and then choose the answer for each of the following
questions.
Napoleon Bonaparte's ambition to control all the area around Mediterranean Sea led him
and his French soldiers to Egypt. After losing a naval battle, they were forced to remain
there for three years. In 1799, while constructing a fort, a soldier discovered a piece of
stele (a stone pillar bearing an inscription) known as the Rosetta stone, in
commemoration of the town near the fort. This famous stone, which would eventually
lead to the deciphering of ancient Egyptian hieroglyphics dating to 3100 B.C, was written
in three languages: hieroglyphics (picture writing), demotic (a shorthand version of
Egyptian hieroglyphics), and Greek. Scientists discovered that the characters, unlike
those in English, could be written from right to left and in other directions as well. The
direction in which they were read depended on how the characters were arranged. Living
elements (animals, people and body parts) were often the first symbols, and the direction
that they faced indicated the direction for reading them. Twenty three years after the
discovery of the Rosetta stone, Jean Francois Champollion, a French philologist, fluent in
several languages, was able to decipher the first word Ptolemy the name of an Egyptian
ruler. This name was written inside an oval called a "cartouche". Further investigation
revealed that cartouches contained names of important people of that period.
Champollion painstakingly continued his search and was able to increase his growing list
of known phonetic signs. He and an Englishman, Thomas Young, worked independently
of each other to unravel the deeply hidden mysteries of this strange language. Young
believed that sound values could be assigned to the symbols, while Champollion insisted
that the pictures represented words.
81. All of the following languages were written on the Rosetta stone EXCEPT
A. French

B. demotic

C. Greek

D. hieroglyphics

82. Napoleon's soldiers were in Egypt in 1799 because they were............

A. celebrating a naval victory.


B. looking for the Rosetta stone.
C. waiting to continue their campaign.
D. trying to decipher the hieroglyphics.

83. The person responsible Jar deciphering the first word was...........
A. Champollion

B. Young

C. Ptolemy

D. Napoleon

84. Why was the piece of newly discovered stele called the Rosetta stone?
A. It was shaped like a rosette.
A. It was to honour Napoleon's friend Rosetta.
B. The town near the fort was called Rosetta.
C. The fort was called Rosetta.

85. What is the best title Jar the passage?


A. Napoleon's Great Discovery.
B. Deciphering the Hieroglyphics of the Rosetta stone.
C. Thomas Young's Great Contribution.
D. The Importance of Cartouches.

Part D. WRITING
I. Choose the sentence A, B, C or D which is closest in meaning to the original sentence.
86. You'll be better off staying the night.
A. It will be better to stay the night.
B. It will be good of you to stay the night.
C. It will be better for you to stay the night.
D. You will be better if you stay the night.

87. Bill had had the wireless repaired.


A. Bill had arranged for his wireless to be repaired.
B. Bill had repaired his wireless.
C. Bill had arranged to repair his wireless.

D. Bill had had someone to repair his wireless.


88. According to his arrangement, a pension will be provided everyone over sixty.
A. He has arranged for everyone over sixty to be provided a pension.
A. He has arranged to provide everyone over sixty a pension.
B. He has arranged for everyone over sixty to be provided with a pension.
B. He has arranged to provide for everyone with a pension.

89. After many years of hard work, he retired.

A. After working hard many years, he retired.


B. Having worked hard many years, he retired.
C. After he had many years of hard working, he retired.
D. After he had worked hard for many years, he retired.

90. With the money he has saved, he can buy a house.


A. His saving can help him buy a house.
B. His savings can help him buy a house.
C. He can buy a house with his saving.
D. He can buy a house due to his saving.

91. The suitcase was extremely, heavy but he managed to it easily.


A. Despite the heavy weight of the suitcase, he managed to lift it easily.
B. Although the suitcase was heavy but he managed to lift it easily.
C. The suitcase was not too heavy to lift.
D. He had no difficulty in lifting the suitcase.

92. "I never told anyone about your scheme," he said.


A. He denied not telling anyone about my scheme.
B. He denied telling anyone about my scheme.
C. He said not to tell anyone about my scheme.
D. He told to me he never told anyone about my scheme.

93. We missed the bus because we overslept.


A. We missed the bus as a consequence of oversleeping.
B. We overslept and as result we missed the bus.
C. The reason of missing the bus is we overslept.
D. If we didn't oversleep, we wouldn't miss the bus.
94. Without your sound advice, 1 would never have made such a good investment.
A. If you hadn't given to me sound advice. I would never have made such a _

good

investment.
B. If it hadn't been for your sound advice, I would never have made such a good

investment. investment.
A. I have made such a good investment thanks for your sound advice.
C. Thank you for your sound advice, without it I would never have made such a good

investment.
95. I don't think he's likely to phone me tonight.
A. He isn't thought to phone me tonight.
A. It will be unlikely that he phones me tonight.

B. I doubt if he will phone me tonight.


B. No doubt he will phone me tonight.

II. Choose one sentence (A, B, C or D) that is best written based on the words and
phrases provided.
96. We should/dictating machine / rather I secretary.
A. We should have dictating machine rather than a secretary.
A. We should have a dictating machine rather than to have a secretary.
B. We should have dictating machine rather than secretary.
C. We should have a dictating machine rather than a secretary.

97. stand /plane / doctor / good-bye / everybody / airport.


A. Standing besides the plane, the doctor said good-bye to everybody at the

airport.
B. Standing by the plane, the doctor said good-bye with everybody at the airport.
A. To stand by the plane, the doctor said good-bye to everybody at the airport.
B. Standing by the plane, the doctor said good-bye to everybody at the airport.

98. It / age / 14 / Jack London / leave .school / sailor.


A. It was at the age of 14 when Jack London left school and became a sailor.
B. It is at the age of 14 that Jack London left school and became a sailor.
A. It was at the age of 14 that Jack London left the school to become a sailor.
B. It was at the age of 14 that Jack London left school and became a

99. accident / my (fault / so / pay / damage.


A. The accident was my fault so I had to pay the damage.
B. The accident was my fault so I had to pay for the damage.
C. The accident was my fault and so I had paid for damage.
D. The accident was my fault and so I had to pay for damage.

100. some / leader / seventy / reach / peak / career.


A. Some leaders are seventy when they reach the peak of their career.
B. Some of the leaders ate seventy when they reach the peak of career.
C. Some of leaders are seventy when they reach peak of their career.
D. Some of the leaders are seventy when they reach the peak of their career.

ENGLISH OBJECTIVE TEST 18


Time allowed: 90 min.
Part A. PHONETICS
I. Circle the word whose underlined part is pronounced differently from the others in

each line.
1.

A. fear

B. near

C. pear

D. rear

2.

A. cloak

B. broad

C. soak

D. throat

3.

A. book

B. foot

C. brook

D. booth

4.

A. mechanical

B. mechanism

C. camping

D. ceramic

5.

A. stomach

B. starch

C. chimney

D. cheat

II.

Choose the word with the position of the stressed syllable different from that of
the others in each line.

6.

A. capitalist

B. diagnostic

C. satellite

D. flowery

7.

A. recapture

B. immature

C. domestic

D. recover

8.

A. commentator B. practicable

C. preferable

D. prehistory

9.

A. principal

B. recommend

C. recollect

D. unemployed

B. secretary

C. speculation

D. optimism

10. A. sanitary

Part B. LEXICO - GRAMMAR


I. Choose the form of the verb (A, B, C or D) which best fits each space in the passage.
Linda had recently moved to Atlanta from Ponders, the small town where she grew up. (11)
how difficult it (12) to find employment, she (13) in her hometown.
She loved the big city, though, and she felt that had she stayed in Ponders, she (14)
into a rut that she (15) from. The only problem was that Linda needed to find a job
soon, otherwise, she wouldn't be able to pay next month's rent, and she would have to go
back to ..................................Ponders. The trouble was she was shy about (16) . for
work.
One day. Linda (17) around downtown, (18) that, with a bit of luck,
she might find something. She saw a pleasant - looking florist's shop.
Maybe they (19) If so, she might get a job. If not, she (20) anything by
going and asking. Without even thinking further, she walked in.
11. A. Had she known B. She had known C. Did she know
12. A. was

B. would be

13. A. would stay

B. would have stayed C. stayed

D. She knew

C. would have been D. had been


D. had stayed

14. A. would be fallen B. would fall

C. would have fallen D. fell

15. A. would never escape

B. never would escape

C. had never escaped


16. A. to ask

B. asking

17. A. was going to wander

D. never escaped
C. having asked
B. had wandered

D. being asked

C. was wandering
18. feeling

D. wandered
B. felt

C. was feeling

D. had felt

19. A. would be hiring B. would hire

C. will be hiring

D. will hire

20. A. would have lost B. would not lose

C. will have lost

D. will not lose

II. The underlined parts in each line below are marked A, B. C, D. identify the one
underlined word or phrase that would not be acceptable in standard English.
21. Washington, D.C. lies between Virginia and Maryland by the Potomac River.
A

C D

22. It's about 220 miles south in New York City. The most pleasant and easiest
A B

23.way to get there is by long-distant bus, or by the fast (125-mile-an-hour)


A

24. train costing. a little more than the bus journey and a little less than to fly.
A

25.I would travel far more than 200 miles to watch Washington. It's not a
A

26. city that has grown up accidentally like most big cities have done. It was
A

27.carefully planned as the nation's capital by a French, Pierre L' Enfant. The
A

28.city was named by George Washington, the much-loved. much-admired,


A

29. much-respected first President of the United State. in 1791 he himself


A

30. arranged to buy the land at which it stands.


A

III. Choose the best word (A, B, C or D) to complete each of the following sentences.
31. The change in timetable will . many students having to catch an earlier bus.
A. result

B. lead

C. mean

D. cause

32. . you aren't suggesting that what he did was in any way defensible?
A. Certainly

B. Probably

C. Undoubtedly

D. Surely

33. No one was able to think . the noise was going on.
A. during

B. while

C. since

D. on account of

34. These figures give you some idea of the cost of . your car for one year.
A. controlling

B. handling

C. managing

D. maintaining

35. As the managing director can't go to the reception, I am representing the company
.
A. on his account

B. on his behalf

C. from his point of view

D. for his part

36. His illness made him . of concentration.


A. incompetent

B. unable

C. incapable

D. powerless

37. He spoke clearly and and we could understand every word he said.
A. distinguished B. distinguishable C. distinctly

D. distinctively

38. Mr. Johnson has been asked to the next meeting of the club committee.
A. manage

B. take

C. chair

D. lead

39. She did not staying at home as she had some sewing she wanted to do.
A. mind

B. object

C. matter

D. care

40. When he graduated from the university he got his


A. diploma

B. certificate

C. degree

D. bachelor

41. Mary was so disappointed that she into tears.


A. fell

B. broke

C. burst

D. rushed

42. After his illness, he was advised to . gardening as a hobby.


A. take up

B. occupy

C. turn into

D. enter

43. She is the most .. manageress we have ever had.


A. good

B. working

C. active

D. efficient

44. They are . to be married next year.


A. planned

B. prepared

C. intended

D. engaged

45. Everyone . Alan was invited to the party.


A. excepting

B. apart

C. but

D. though

IV. Choose the preposition which .best fits the space in each of the following
sentences.
46. I could make .. the expression on his face.
A. up

B. of

C. out

D. for

47. The piano went .. the door easily, but the sideboard didn't.
A. past

B. by

48. We have already fallen ..

C. across

D. through

our competitors in the production of advanced

computer hardware.
A. off

B. behind

C. down

D. into

49. Ellen had the operation this morning and it all went .. quite well.

A. off

B. by

C. through

D. on

50. The acid gives .. a characteristically pungent odour.


A. out

B. in

C. off

D. up

51. He got .. four years of war without suffering so much as a scratch..


A. across

B. through

C, over

D. out of

52. "Have you found the title for the book?" "No, not yet." "When will it Come
..?
A. up

B. off

C. to

D. out

53. A water shortage had struck the area and the wild life was dying .. alarmingly in
the intense heat.
A. from

B. out

C. off

D. of

54. Many households in the neighbourhood of the University add to their income by taking
.. students.
A. up

B. in

C. from

D. after

55. The Minister of Commerce set .. an emergency committee today to ensure the
maintenance of bread supplies
A. up

B. off

C. in

D. down

Part C. READING
I. Choose the correct word form (A, B, C or D) which best fits each space in the
following passage.
Between 1820 and 1860 the United States underwent three (56) ..... changes: waves of
immigrants flooded the port cities; the Anglo Saxon population greatly increased, and cities
grew with a rapid tempo. Proud (57) ......... of the old (58)....... ............. felt that those changes,
taken as a whole, constituted a threat to American society. The massive (59) ....... was
larger and more rapid than that of the past. No legal (60) ....... inhibited the stream of
those (61) ................. a new life in the Atlantic. About 5,000,000 immigrants came to America
between 1820 and 1860, half of them (62) ...... after 1850.
A small nation of 10,000,000 in 1820 was (63) ...... into a giant of 31,000,000 in 1860.
Many of the (64).......brought with them a way of life which (65) ........... radically from that of
America.
56.

A. signifies

B. signifying C. significance

D. significant

57.

A. defendents

B. defendants C. defenders

D. defendinLis

58.

A. publicity

B. publicant

D. republic

59.

A. immigration

B. immigrant C. immigrating

D. migrating

60.

A. restricts

B. restrictions C. restrictations

D. restricters

C. republican

61.

A. seekers

B. seek

C. seeking

D. seeked

62.

A. arrived

B. arriving

C. arrive

D. arrival

63.

A. transforming

B. transformedC. transformant

D. transformable

64.

A. newcomers

B. comers

C. rccentcomers

D. newlycomers

65.

A. differing

B. differed

C. differ

D. differs

IL Circle the letter A, B, C or D next to the word which best fits each space in tile
following passage.
A recent experiment has (66) ....... that the British postal service is (67) ....... than it was
150 years ago. A national newspaper sent a letter to Bristol, which is in the south-west of
England. They (68) .......the letter in London on Tuesday the 4 th , April, (69)....... a first class
stamp,.........................................................................which means that it should arrive the next day.

On the same day, they (70) ....... a and carriage, and gave another letter to the driver. Then they
sat back to (71) ....... what would happen.
The letter which was .(72) ......................................... by horse and carriage arrived after three days.
It was (73) .................................................................. for the horse to (74)

Bristol in a single

day, so both the horse and driver (75)....... overnight in a hotel not in the same room, we hoped!

However, the letter which (76) ............................ by Royal Mail didn't arrive until six days
later. Naturally, the Post Office was very embarrassed, and said that the vast majority of first
(77)....... letters would arrive the next day; this one was just an (78)....... mistake, but so many
people have had the same experience with the (79) ....... that you begin to wonder if we would
be better off (80) ....... all the post by horse again.
66. A. shown

B. said

C. spoken

D. given

67. A. efficient

B. effect

C. serviced

D. used

68. A. sent

B. posted

C. transported

D. carried

69. A. used

B. stuck

C. using

D. putting

66. A. hired

B. rented

C. had

D. used

67. A. watch

B. wait

C. see

D. know

68. A. sent

B. brought

C. loaded

D. taken

69. A. possible

B. capable

C. impossible

D. incapable

70. A. reach

B. arrive

C. come

D. go

71. A. sat

B. lived

C. stayed

D. remained

72. A. carried

B. went

C. brought

D. sent

66. A. price

B. kind

C. class

D. type

73. A. unforgettable

B. unknown

C. unheard

D. unfortunate

C. sending

D. letter

74. A. postage

- B. post

75. A. posting

B. sending

C. transforming

D. transporting

III. Read the following passage and then choose one answer for each of the following
sentences.
Many people believe that Americans love their cars almost more than anything else. It
may be true. From the time youngsters become fourteen years old or even earlier, they are
likely to start dreaming of having their own car. Many young people work after school
during their last year at high school in order to save money to buy a car. Learning to drive
and obtaining a driver's license may be one of the most exciting times of a young person's
life.
People who do not like to go to a doctor when they are ill will take their cars to a
mechanic at the first sign of a problem. Those who do not like to work around the home on
Saturday may devote most of that day to washing and waxing their cars.
Many men and women in the United States need to have cars. People need cars to go to
work in. Housewives need cars to go shopping or to take the children to school or for other
activities. That is why many families have two or even three cars.
In most states young people learn to drive in high school, where driver training is one of
the most popular courses. At the end of the course the student must take a driving test to
obtain a license. For many that piece of paper is an important symbol that they arc now adults.
81. What do many people believe?
A. Americans like their cars best.
B. Americans like their cars more than anything else.
C. Most Americans have more cars than anything else.
D. Only some Americans love their cars almost more than anything else.

82. What is one of the most exciting times of a young person's life?
A. The time when they have a car.
B. The time when they have learned to drive and obtained a driver's licence.
C. The time when they have learned how to drive a car.
D. The time when they get a driver's license.

83. What do many people devote most of their Saturday to?


A. Cleaning and polishing their cars.

B. Planting trees.

C. Repairing the roof.

D. Painting their houses.

84. Where do.some boys and girls learn to drive?


A. In church

B. In a club

C. In high school

D. In college

85. A driver's licence is important to many students because.............


A. they need to show it to their teachers.

B. they need to show it to their parents.


C. it shows that they are adults.
D. It shows that they are school leavers.

Part D. WRITING
Choose one sentence among A, B, C or D which is closest in meaning to the original
sentence.
86. If you go to the party with us, I will pick you up.
A. If you are ready to go to the party with us, I will pick you up.
B. If you are willing to go to the party with us, I will pick you up.
C. I will probably pick you up if you go to the party with us.
D. Please go to the party with us and I will pick you up.
87. It's believed that a general election will be held in Cambodia.
A. A general election is believed that it will be held in Cambodia.
B. A general election is believed to hold in Cambodia.
C. A general election is believed to he held in Cambodia.
D. A general election is believed it will he held in Cambodia.
88. It wasn't believed until the 15th century that the earth revolves around the sun.
A. That wasn't believed before the 15th century that the earth revolves

around the sun.


A. That the earth revolves around the sun wasn't believed until the 15th

century.
B. The earth was believed to revolve around the sun until the 15th

century.
D. Not until the 15th century the earth was believed to revolve around the sun.
89. More people are infected with HIV this year than they were 50 years ago.
A. Not as many people were infected with HIV 50 years ago as they are

this year.
B. Less people arc infected with HIV this year than they were 50 years ago.
C. Few people are infected with HIV this year than 50 years ago.
D. There are fewer people infected with HIV this year as they were 50 years ago.
90. Why didn't you read the instruction?
A. You should read the instruction.
B. You had to read the instruction.
C. You had better to read the instruction.
D. You should have read the instruction.
91. It's very kind of you to give me a lift.

A. How kind are you to give me a lift!


B. I appreciate your giving me a lift.
C. I appreciate you to give me a lift.
D. It's kind for you to give me a lift.
92. The storm blew the roof off the house.
A. The stoiiii caused the roof of the house blow off.
B. The storm made the roof of the house be blown off.
C. The house had its roof blown off in the storm.
D. The house had its roof blown off by the storm.
93. He did all out of his kindness.
A. All that he did was due to his kindness.
B. All that he did were due to his kindness.
C. He did all for his kindness.
D. What he did was owing to his kindness.

94. If you don't work hard, you will not have any chance of success.
A. Your chance of success depends on how do you work.
B. Whether or not you succeed depends on your working. hard.
C. Unless you work hard, you will not be likely to have any chance of success.
D. If you work hard, you will have any chance of success.

95. He forgot her birthday until his friend phoned him.


A. It wasn't until his friend phoned him did he remember her birthday.
B. Without his friend he didn't remember her birthday.
C. Not until his friend phoned him did he remember her birthday.
D. If his friend didn't phone him, he didn't remember her birthday.

II.

Choose the best answer among A, B, C or D that is best itten based on following
words and phrases.

96. Your handwriting / difficult / read /. You / should / letter / type.


A. Your handwriting is difficult to read. You should have letters typed.
B. Your handwriting is difficult to read. You should send your letters without typing.
C. Your handwriting is difficult to read. You should have your letters typed.
D. Your handwriting is difficult to read. You should have a letter typed.
97. I / he / sure / he / tell / truth / . He / never / tell / lie / life.
A. I am sure he told truth. He has never told a lie in his life.
B.

I am sure he has told the truth. He has never told a lie in life.

C.

I am sure he told the truth. He never tells a lie in his life.

D. I am sure he told the truth. He has never told a lie in his life.

98. // wet / through / If/ know / rain / I / take / umbrella.


A. I am wet through. If I had known it rained, I would have taken my umbrella.
B.

I am wet through. If I had 'known of the rain, I would have taken my umbrella.

C.

I am wet through. If I had known it would rain, I would have taken my umbrella.

D. I am wet through. If I knew it would rain, I would take my umbrella.

99. Never / put off/ tomorrow / you / do / today.


A. Never put off until tomorrow what you can do today.
B.

Never put off to tomorrow what you can do today.

C.

Never put off for tomorrow what you can do today.

D. Never to put off until tomorrow what you can do today.

100. My father / tired / city / want / quiet village.


E.

A. My father is tired from living in the city so he wants to live in a quiet

F.

hday. B. My father is tired of living in the city so he wants a quiet village.

G. My father is tired from living in city so he wants to stay in a quiet village.


H. My father is tired of living in the city so he wants to live in a quiet village.

English objective test 19


Time allowed: 90 min.
Part A. PHONETICS
I. Choose the word in each group that has the underlined part pronounced differently
from the rest.
1.

A. clean

B. death

C. head

D. heavy

2.

A. vision

B. apprehension

C. emission

D. pension

3.

A. cosmetic

B. conventional

C. continental

D. element

4.

A. delicate

13. intimacy

C. military

D. dramatic

5.

A. guidance

B. gesture

C. get-together

D. graduate

II. Choose the word that has different stress pattern from the rest in each group.
6.

A. transition

B. traveller

C. unpleasant

D. supportive

7.

A. vocational

B. uncertainty

C. solidify

D. occupation

8.

A. safeguarding B. satellite

C. reclaiming

D. moderate

9.

A. preservation

B. restoration

C. orientation

D. miraculous

10.

A. reduction

B. intensive

C. disposal

D. undergrowth

Part B. LEXICO - GRAMMAR


I. Choose the correct word form (A, B, C or D) which best fits the space in eac of the
following sentences.
11.

It was only after he'd been......for the third time that she asked for a divorce.
A. faithless

12.

C. unfaith

D. failing

Management offered employees a two per cent pay rise, but the union fount this offer
A. unaccepted

13.

B. unfaithful
B. accepted

C. unacceptable

D. acceptable

The teacher helped the student pass her exam. She was most.........and boug1-1 her a
present.
A. appreciated B. appreciatively

C. appreciating

D. appreciative

14. I know you feel fed up, but dont let one failure ... you. It was your first interview, after
all, therell be more.
A. discourage

B. courage

C. encourage

D. miscourage

15. Lack of water and agricultural land impose the most serious ..... on Australias future
development.
A. limits

B. limitations

C. limit

D. limiting

16. This hotel gives ..... treatment to people who stay in it regularly.
A. preferable

B. preferential

C. preferred

D. preference

17. We should apply new technology to ..... our country into a modern nation.
A. form

B. transforming

C. transform

D. forming

18. Right at the beginning of their ..... programme the Japanese realized th importance of
an iron and steel industry.
A. modern

B. moderning

C. modernizing

D. modernization

19. The criminal stayed in .... until the police had passed.
A. concealation

B. conceal

C. concealment

D. concealness

20. At what temperature does alcohol become a ......?


A. solid
II.

B. solidarity

C. solidity

D. solitary

Choose the best word (A, B, C or 1)) to complete each of the following sentences

21 Laser beams which are useful in both medicine and industry were first ......... in science
fiction stories 30 years ago.
A. used

B. predicted

C. written

D. said

22. The public is highly suspicious of the oil industry whose profits have been increasing.

In spite of the energy............................


A. crisis

B. short

23. Carbohydrates which are ........

C. deficient

D. insufficient

of carbon, hydrogen and oxygen are organic

compounds.
A. consisted

B. included

C. composed

D. made

24. Hoping that he would pass the course, he stayed up all night studying for final exam:

unfortunately. he..........................and missed the test.


A. slept

B. slept over

C. overslept

D. slept through

25. It was the longest film I've ever seen: it ....... three hours.

A. lasted

B. stayed

C. finished

D. completed

26. We were so late, we.........had time to catch the train.

A. nearly

B. almost

C. hardly

D). simply

27. He asked if i -c would to share the room.


A. accept

B. consider

C. agree

D. approve

28. If you wish to learn a new language you must............ class regularly.
A. follow

B. present

C. attend

D. assist

29. It takes a great deal of............ for the class to make a trip abroad.
A. work

B. organization

C. expense

D. business

30. They agreed to call off the strike on that all those who had been dismissed were
reinstated.
A. terms

B. demand

C. request

D. condition

C. in

D. of

31. These goods are free ........... duty.


A. from

B. with

32. The travel ........... plans holidays for people.


A. accountant

B. applicant

C. astronaut

D. agency

33. As the fat man sat down, the deck chair............. under him with a loud noise.
A. fell

B. sank

C... dropped

D. collapsed

C. seller

D. producer

34. A person who uses goods is a.............


A. buyer

B. consumer

35. We saw a man..............in the street yesterday.


A. drive

B. staggering

C. recycling

D. strolled

III. Choose the correct preposition (A, B, C or D) which best fits each of thefollowing
sentences.
a.

She felt so guilty (36) .......... not telling the truth. She knew that because
this she had been found guilty (37)........ murder.

b.

How could she have been so unfaithful (38). him when he had such faith (39)

. her?
c.

She criticizes everything (40) ........ him. She criticizes him (41)...not b&-- ________

ambitious enough, and she's even critical (42) .................

the

way

he

eats

.-

breakfast cereal!
d.

He may be patient (43) ...... his own children.

e.

Coal has suffered a decline (44)........importance since World War II.

f.

It is undeniable that the Five-Year Plans have resulted (45) some very great
achievements.

36. A. of

B. about

C. with

D. for

37. A. of

B. about

C. with

D. for

38. A. with

B. for

C. to

D. at

39. A. with

B. in

C. at

D. to

40. A. of

B. in

C. on

D. about

41. A. for

B. about

C. on

D. from

42. A. on

B. with

C. of

D. at

43. A. for

B. with

C. of

D. at

44. A. in

B. of

C. at

D. for

45. A. from

B. to

C. of

D. in

IV. Choose the correct verb form which best fits each numbered space in the
following sentences.
1.

Hello Paul! How are you? I (46) ................ you for ages! What you (47) ...... since
I last (48) .............. you.

2.

I (49) ............... forward to spring (50) .......... Winter seems (51) ........ with us for
months, and I can't stand (52) ........................ up when it's still dark outside. Soon
some flowers should be coming out. That'll be nice.

3.

Travelers since the time of Marco Polo (53) ................. on the Chinese custom of
making use of every bit of land which (54)................

Recently WHO announced that the disease of smallpox (55) ....................


parts of the world.

46. A. haven't seen

C. hadn't seen
47. A. did you do

C. you did
48. A. had seen

C. have seen
49. A. look

B. didn't see
I). haven't been seeing
B. had you done
D. have you been doing
B. saw
D. was seeing
B. have looked

in

most

C. am looking
50. A. arriving

C. to arrive

D. was looked
B. arrive
D. arrived

51 A. to be

B. having been

52. A. to get

B. getting

C. to have got
53. A. remarked

D. get
B. had remarked

C. have remarked

D. has remarked

54. A. could be cultivated

B. could cultivate

C. can cultivate
55. A. almost wiped off
C. almost wipes off

D. can be cultivate
B. had almost wiped off
D. had almost been wiped off

V. Decide which of the underlined part A, B, C or D is incorrect in standard English.


56. The number of people contracting AIDS, a dead virus disease, is escalating
A
B
C
D.
every year.
56. To use a calculator, one can easily convert kilos to pounds and Celsius to
A
B
C
D
Fahrenheit.
57. Before hand calculators were common, math students usually carried a
A
B
slide-rule, it is a type of calculating ruler.
C
D
58. The manner of election the president in the USA is written in the Constitution.
A
B
C
D
56. A second region in which manufacturing has expanded rapidly is the Pacific
A
B
C
Coast, specially California.
D
56. Most pubs have two drink rooms, called bars the public and the saloon bar,
A
B
C
which is more comfortable and slightly more expensive.
D
57. No alcoholic drinks may be served to young people under 18 and no children
A
B
C
under 16 are allowed in the bar.
D
63. When they got there, the room was crowded with people and a MP was making
A
B
C
D
a speech.
64. How all the documents disappeared remains mystery.
A
B
C
D
65. In spite of high unemployment, a few British people will accept jobs as
A
B
C
D
living in Servants.

Part C: READING
I. Choose the best option (A, B, C or D) to fill each numbered blank in the following

passage.
It isn't always easy these days to (66)........ between people from different (67) ........of
Britain. (68)........... because British people (69) ........ about the country more than they used to.
Northerners come and live in the south. Southerners and East Anglicans go to the North, and lots of
Scots, Welsh and Irish come and live in England. More people (70) ....... in London and the

south than in any other (71) ...... That is why many Southerners do not really have a special
(72) ....... except the Cockneys from the East End of London.
The North still has a (73) ....... of its own, and Southerners who settle in the North usually take

(74) ........ the Northern way of life. Their children soon (75) ...... up the accent from their
school friends.
As the English are such a mixed people, local customs and accents in England vary a
great deal and local (76) ........is still strong in some (77) ....... of the country. The closer one
gets to London, the less one (78) ...... such differences, for London is a melting-pot. People
from all over Britain arid from all over the world (79)

....... into the giant city. London

tends to "melt down" and smooth out strong accents and provincial customs. very year the
influence of London (80)

further

and

further

into

the

country,

north,

south, east and west, but particularly into the south and southeast.
66.

A. realise

B. distinguish

C. classify

D. know

67.

A. areas

B. places

C. parts

D. zones

68.

A. That's

B. It's

C. This is

D. They are

69.

A. come

B. arrive

C. move

D. reach

70.

A. arrive

B. settle

C. come

D. stay

71.

A. region

B. regions

C. part

D. places

72.

A. voice

B. speech

C. accent

D. speak

73.

A. character

B. characteristic

C. characterization D. characterizing

74.

A. up

B. in

C. on

D. off

75.

A. set

B. take

C. build

D. pick

76.

A. pride

B. thought

C. tradition

D. preservation

77.

A. places

B. spots

C. zones

D. parts

78.

A. recognises

B. notices

C. watches

D. know

79.

A. go

B. reach

C. pour

D. get

80.

A. spreads

B. fill

C. widen

D. reach

IL

Read the passage and then choose one answer for each of the following questions.
The diseases connected to smoking are a big problem. Doctors think that the annual
medical cost for lung cancer, heart disease, and other illnesses connected to smoking is
between 12 and 35 million pounds.
And smoking costs society money in other ways. Between 27 and 61 billion pounds
are spent each year on sick days when people don't go to work, on wages that you don't
get when you don't go to work, and on work lost at the company when you are sick.
This money counts the wages from people who die of cancer at young age and stop
paying taxes. This does not count fire started by cigarettes, which kill fifteen hundred
people yearly and injure another four thousand. Smoking costs every man, woman and
child in the UK from one hundred and ten to two hundred and fifty pounds yearly in
insurance cost, that comes to one hundred and sixty to four hundred and ten pounds. If
everyone stopped smoking, a family of four could have tip to one thousand six hundred
and forty pounds a year more.

Smoking will also cause other problems. People who don't smoke will live longer, and so
they will take money from the government when they are old. But they will also work
for more years and pay more taxes.
In the end, the value of a non smoking nation is not in pounds. The good health of people
is the true value for us all.
81. if everyone stopped smoking, all the United Kingdom
A. would have more money

B. would live longer

C. would have less money

D. would have no more problems.

82. Every year companies lose........ because of the diseases connected to smoking.

A. money

B. work

C. wages

D. time

83. If everyone stopped smoking, a family of four could have.........more each year.
A. from 110 to 260 pounds
B. from 1,340 to 1,430 pounds
C. from 160 to 410 pounds

D. 1,640 pounds

84. The true value for the UK of not smoking is ....................


A. more working
B. more taxes
C. good health D. more money
85. This text is about
A. taxes which are not paid by smoke
B. diseases that smokers get
C. how much smoking costs the UK
D. how much the UK get if everyone stopped smoking

Part D. WRITING
I. Choose the answer (A, B, C or D) which is closest in meaning to the original sentence.

86. You have disobeyed; you will therefore be punished.


A. As a result of disobedience; you will be punished.
B. As a result of your disobedience, you will be punished.
C. You will be punished because of not obedience.
D. If you don't obey, you will be punished.

87. His real identity will always remain a secret.


A. No one knows his real identity.
B. No one must know his real identity.
C. No one will ever know his real identity.
D. His real identity hasn't been known.

88. He never suspected that she was a witch.


A. Never has he suspect that she was a witch.
B. He used to think she was not a witch.
C. ever did he suspect that she was a witch.
D. What he thought was she was not a witch.

89. Are they likely to pass Proficiency?


A. Is there any likelihood of passing Proficiency?
B. Is there any likelihood to pass Proficiency?
C. Is there any likelihood of their passing Proficiency?
D. Will they be likely to pass Proficiency?

90. Whenever she went to Paris she bought a new dress.


A. When she went to Paris she used to buy a new dress.
B. She bought a new dress if she went to Paris.
C. She never went to Paris without buying a new dress.
D. She bought a new dress on the way to Paris..

91. If you are ever in the neighbourhood, please drop in and see us.
A. Should you ever be in the neighbourhood, please drop in and see us.
A. We would like you to drop in and see us if you come to the neighbourhood.
B. Would you ever be in the neighbourhood, please drop in and see us.
C. Will you ever be in the neighbourhood, please drop in and see us.

92. It is not certain that Jones will get the job.


A. Jones won't be likely to get the job.
A. Jones certainly doesn't get the job.
B. It is open to question whether Jones will get the job.
C. Jones may not be getting the job.

93. We weren't surprised by his success.


A. We had no surprise at his success.
A. We were not surprised of his success.
B. It came as no surprise to us that he was successful.
C. Not surprised that he was successful.

94. She admitted that she had taken the ring.


A. She admitted to take the ring.
A. She admitted to have taken the ring.

C.. She admitted she stole the ring._


D. She admitted having taken the ring.
95. It was only when he saw her in private that he told her
A. Only when he saw her in private that he told her everything,
B. He didn't tell her everything until he saw her in private.
C. Not until he saw her in private did he tell her all thing.
D. It was not until he saw her in private that he told her evcrythi"g.

II. Choose one sentence (A, B, C or D) that is best written based on the words rovided.
96. rain / every day / have / spend / most / time / indoors.
A. It rained everyday so we had to spend most of time indoors.
B. It rained every day so we had to spend most of our time indoors.
A. It rained nearly every day so we had to spend most time indoors.
C. It rained every day so we have spent most of our time indoors.

97.opinion / violent flM / not / show / television.


A. In my opinion,. violent films should not be shown on television.
B. In my opinion, violent films should not show on television.
C. In my opinion, violent films cannot show on television.
D. In my opinion, violent films should be not shown on television.

98. If/ smoke / great chance / you / die / lung cancer.


A. If you smoke, there is a great chance that ynu will die of luno cIncer.
B. If you smoke, there is great chance that you will die from lung cancer.
C. If you smoke, there is a great chance for you to die of lung cancer,
D. If you smoke, there is a great chance you die from lung cancer.

99.it / important / brother / stop / smoke / once.


A. It is important for your brother to stop to smoke at once.
B. It is important for your brother to stop to smoke at once.
C. It is important that your brother stop smoking at once.

D. It is important that your brother will stop smoking at once.


100. The / long / he listen / more attentive / become.
A. The longer he listened, the more attentively he became.
B. The longer he listened, the more he became attentive
C. The longer he listened, the more attentive he would become.
D. The longer he listened, the more attentive he had become.

English Objective test 20


Time allowed: 90 min.
Part A. PHONETICS
I. Choose theword in each group that has the underlined part pronounced differently
from the rest.
1.

A. Sensible

B. scalpel

C. surgery

D. satellite

2.

A. assure

B. relaxed

C. obstacle

D alternative

3.

A. account

B. mountain

C. roundabout

D. routine

4.

A. how

B. towel

C. vowel

D. rowing

5.

A. villa

B. violet

C. cricket

D. signal

II.

Choose the word that has different stress pattern from the rest in each group.

6.

A. telephonist

B. ultimate

C. necklace

7.

A. geometric

B. constitution

C. unaccountable D. parental

8.

A. geologist

B. material

C. temperate

D. courageous

9.

A. alcoholic

B. continental

C. engineering

D. philosophy

10.

A. artistic

B. convertible

C. pull over

D semester

D. moonlight

Part B. PHONETICS
I. Choose the word or phrase (A, B, C or D) which best completes each sentence..
11.

We're in good time; there's ....... to hurry.


A. unnecessary

12.

D. stolen

B. over

C. off

D. to

B. hers

C. herself

D. her own

B. where

C. that

D. it

on't....................him to arrive early. He's always late.


A. think

17.

C. lost

There was a small room into........we all crowded.


A. which

16.

B. missed

She was singing an old Spanish folksong, a favourite of.........


A. her

15.

D. impossible

The man who lives opposite us sometimes comes........for a CUP of coffee.


A. on

14.

C. no need

My brother had his camera........from his car in the office car-park.


A. robbed

13.

B. no purpose

B. judge

C. attend

D. expect

The hotel receptionist said she would ..... what she could do about the dripping tap
immediately.

A. find
18.

C. see

He will do the work and then send you the


A. sum

19.

B. try
B. note

D. look

for it.
C. addition

D. bill

I had a ., which I couldn't explain, that something terrible was going to


happen.
A. sense

20.

She seems very confident but you


A. could

21.

B. through
B. aim
B. make

C. might

D. ought

C. down

D. away

C. target

D. focus

C. enjoy

D. live

She has bought a new.................near the park.


A. accommodation B. housing

25.

never judge by appearances.

Do you think it's possible to ................. on nothing but fruit and vegetables?
A. eat

24.

D. view

TV advertising in the late afternoon tends to..................young children.


A. point

23.

B. should

C. feeling

All his plans for starting his own business fell ..................
A. in

22.

B. thought

C. flat

D. rental

They had to walk up a very .................. hill every day.


A. steep

B. stepped

C. straight

D. sharp

II. Choose the correct form of the word (A, B, C or D) to complete each of the following
sentences.
26.

Don't worry ! Our new product will keep your bathroom clean and..................
A. odourless

27.

D. odourlessly

B. knowledged

C. knowledgeable D. unknowledged

There is too much...................in this world.


A. greed

29.

C. odourful

My teacher is very..................
A. known

28.

B. odour

B. greediness

C. greedy

D. greedness

Mr Smith is a . Person.If he says he will do something, you know that he will


do.
A. depending

30.

D. independent

B. Theorily

C. Theoricly

D. Theoretically

He lost in the election because he is a weak and


A. indecisive

32.

C. dependent

there arc black holes in space.


A. Theorically

31.

B.dependable

B. undecisive

leader.

C. undeciding

The car in front braked...........and I crashed into it.

D. undecided

A. unexpectingly
33.

C. unexpectedly D.expecting

The police car collided with a fire


A. engineering

34.

B. expectantly
B. engineer

C. engine

D.enginer

The elderly tourists do not find the thought of climbing the hill
A. appealed

B. appealing

C. unappealingectedly D. expecting

D. appeal
35.

We need to..............................more of our waste and glass.


A. recycle

36.

B. cycle
B. frightful
B. respectably

C. respectful

D. respectable

B. representative C. representation D. presenting

are unpleasant, but it will be nice when we get into the new house.
A. Removals

40.

D. frightening

This statue is a lifelike ..........of Christ Jesus.


A. presentation

39.

C. frightened

It is not .................................. to be drunk in the street.


A. respecting

38.

D. cyclone

The.........................................horse ran away from the fire.


A. fright

37.

C. tricycle

B. Movements C. Removements D. Moves

Please accept this cheque as a of your services.


A. recognisation

B. recognition

C. recognising D. recognisemeni

III. Decide which of the underlined part in each line is incorrect in standard English
41.

If you ever come in a house in Japan, you must remember to take off y shoes.
A
B
C
D

42.

Otherwise, this would damage the fine straw mats which cover on floors.
A

41. The rooms in almost Japanese houses are usually large. In the middle of the rooom
A

41.
42.

there maybe a low table with small flat cushions around it. Many houses have
A
B
C
D
no other furnitures in their rooms. Perhaps you will see a bowl of flowers or
A
B
C
D

46. long silk painting on one of the walls. Visitors are invited a small cup of green tea

A B

47. You may be surprised to see that there are not bedrooms. The Japanese unroll

A
B
C
D
46. their beds and put them on the floor when they feel tired. Japanese people take bath A
B
take bath.
C
D
47. before their evening meals. Most houses have one large bath for the whole families. A
B
C
D
48. Although, no one washes in the bath ! They wash themselves before they go into
A
B
C
D
the big bath. The water is very hot. But the Japanese are used to having hot baths. After
the bath, they put on a loose robe and eat their evening meal.
Part C. READING
T.

Choose the preposition which best fits each numbered space in the following
passage.

My sense of direction is not very good, and I easily get lost, one day, (51) ..... ten years ago, I was
walking (52) ...... the mountains (53) ..... France and Italy when the weather began to turn bad. I
started to make my way (54)....... downhill as I did not want to be caught (55) ...... a storm. But
after a few hundred meters I realized that I was not sure soon I was completely lost. I called as
loudly as I could, but of course there was nobody close enough to hear me. So I crawled (58)........
a hole (59) ....... two rocks and waited for the storm to go (60) .......
51.

A. in

B. at

C. during

D. about

52.

A. on

B. in

C. at

D. into

53.

A. on

B. in

C. between

D. at

54.

A. of

B. back

C. to

D. toward

55.

A. in

B. by

C. at

D. with

56.

A. about

B. of

C. for

D. with

57.

A. down

B. to

C. back

D. up

58.

A . to

B. into

C. through

D. across

59.

A . with

B. of

C. below

D. between

60.

A . by

B. over

C. past

D. through

II. Read the article below and cirele the letter next to the word that best fits each space.
The BBC is based at Broadcasting House in London, although it has (61) ...... in many other parts

of the country. It is (62) ....... by a board of governors (63) ...... by the government but once
appointed this board has complete freedom and the government cannot (64) ....... Everyone who
(65)......or rents a television set has to pay a yearly (66) .......There is no (67)...... on BBC radio or
television, and it is (68) ...... the sale of licences (69) ...... the BBC gets most of its money. An
annual licence for a colour set costs twice as much as for a black and white set.
The BBC is not the mouthpiece of the government. On the (70)......, all the major political parties
have equal rights to give political (71) ......., and each party sometimes complains that the BBC is
prejudiced (72) ...... it.
There arc four radio channels, each of which "specialises". Radio 1 has mainly pop music; Radio
2 has light music, comedy, sport; Radio 3 has classical and twentieth century music, (73) ...... on
serious (74) ......, ancient and modern. Radio 4 specialises in the spoken word the main nev
reports, talks and discussions, plays, etc. In addition the BBC has local radio (75) ...... in many
cities and districts. The BBC also broadcasts news al information about Britain to countries all
over the world. This World Serv. provides programmes in forty different languages, as well as
English.
The BBC has two television channels: BBC 1 and BBC 2. BBC 2 (76) ...... more serious
programmes than BBC 1 documentaries and discussion= adaptations of novels into plays and
serials, operas and concerts. BBC programmes (77) ...... largely of lighter plays and series,
humour and but there are also some interesting documentaries. BBC 2 is watched lly 10 per cent
of all (78)......
Radio and television programmes ......... weekly periodical The Listener, in which a
selection of radio and TV talks are (80) ......
61. A. offices

B. rooms

C. buildings

D. studios

62. A. controlled

B. directed

C. led

D. operated

63.

A. elected

B. operated

C. appointed

D. voted

64.

A. decide

B. interfere

C. do

D. instruct

65.

A. gets B. buys

C. owns

D. hires

66.

A. money

B. licence

C. sum

D. bill

67.

A. advertising

B. sport

C. entertainment

D. film

68.

A. by B. from

C. with

D. at

69.

A. what

B. and

C. that

70.

A. fact B. reality

C. other

D. contrary

D. which

71.

A. broadcasts

B. opinion

C. show

77.

A. at

C. with

I). to

73.

A. talks

B. speech

C. speaking

D. lecture

74.

A. titles

B. subjects

C. things

D. matters

75.

A. office

B. buildings

C. stations

D. studio

76.

A. shows

B. performs

C. provides

D. offers

77.

A. consist

B. compose

C. include

D. conclude

78.

A. viewers

13. spectators

C. watchers

D. audience

79.

A. a

C. another

D. the

80.

A. made

B. printed

C. broadcast

B. against

B. one

D. point

D. shown

III. Read the passage and then choose one answer for each of the following sentences.
As more women in the United States move up the professional ladder, more are finding it
necessary to make business trip alone. Since this is new for many, some tips are certainly in
order. If you are married, it is a good idea to encourage your husband and children to learn to
cook a few simple meals while you are away. They will be much happier and probably they will
enjoy the experience. If you will be eating alone a good deal, choose good restaurants. In the end,
they will be much better for your digestion. You may also find it useful to call the restaurant in
advance and state that you will be eating alone. You will probably get better service and almost
certainly a better table. Finally, and most importantly, anticipate your travel needs as a
businesswoman; this starts with light weight luggage. Take a folding case inside your suitcase; it
will come in extremely handy for dirty clothes, as well as for business documents and papers you
no longer need on the trip.
And make sure you have a briefcase so that you can keep currently required papers separate.
Obviously, experience helps, but you can make things easier on yourself from the first by careful
planning so that right from the start you can really have a good trip.
81. Who is the author's intended audience?
A. Working women who have no time for cooking.
B. Husbands and children of working women.
C. Working women who must travel on their own.
D. Hotel personnel who must cater for working women.

82. Which of the following can he inferred from the passage?


A. greater percentage of women are advancing professionally in the US than previously.

B. Professional men refuse to accompany their female colleagues on business trip.


C. Each year there arc more female tourists in the United States.
D. Business women become successful by showing a willingness to travel alone.

83. In this passage, what advice does the author have for married women?
A. Stay at home and take care of your family.
B. Encourage your husband and kids to be happy and have fun while you are away.
C. Help your family learn to prepare food for themselves.
D. I lave your whole family take cooking classes together.

84. Why are better restaurants especially preferable for frequent travellers?
A. The food is usually better for your health.
B. The tables are better.
C. You can call ahead for reservations.
D. You will not have to eat alone.

85. .Why is lightweight luggage important for the travelling businesswomen?


A. It provides space for dirty clothes.
B. It can double as a briefcase.
C. It is usually big enough to carry all business documents.
D. It allows mobility.

Part D. WRITING
I.

Choose the answer (A, B, C or D) which is closest in meaning to the original


sentence.

86. The minister's popularity suffered as a result of the scandal.


A. The scandal had a negative effect on the minister's. popularity.
A. The scandal had bad effect to the minister's popularity.
B. The scandal influenced on the minister's popularity.
B. The scandal effected the minister's popularity.

87. The counsellor answered every question ,frankly


A. The counsellor was frank on the way he answered every question.
B. The counsellor gave frank answers to every question.
C. The counsellor gave a frank answer to every question.
D. The counsellor was frank at the way of answering every question.

88. They had to wait for twelve hours before their flight left.
A. Only after a 12-hours wait did their flight leave.
B. Their flight left when they had to wait for 12 hours.
C. Their flight left 12 hours after they had waited.
D. Only after a 12-hour wait did their plane leave.

89. It was difficult for Susan to get used to the new lift there.
A. Susan found it hardly to get used to the new life there.
B. Susan hardly could get used to the new life there.
C. Susan could hardly get used to the new life there.
D. Susan was difficult to get used to the new life there.

90. The company has decided to replace this model.


A. It is the intention of the company to replace this model.
B. It is the company intention to replace this model.
C. The company has intended replacing this model.
D. The company's intention has been replace this model.

91. It was careless of you to leave the window open last night.
A. It was your fault to leave the window open last night.
B. You were too careless to leave the window open last night.
C. You shouldn't leave the window open last night.

D. You shouldn't have left the window open last night.

92. Though my house was cheaper than Norman's, it was bigger and moreattractive.
A. Norman's house may be more expensive but it was smaller and less attractive than

mine.
A. Norman's house may have been more expensive but it was smaller and less attractive

than mine.
B. Despite my house was cheaper, it was bigger and more attractive than Norman's.

D.Despite my house cheaper, it was bigger and more attractive than Norman's.
93. Please would you send me flirt her details of the job advertised?
A. I would be grateful if you send further details of the job advertised.
B. I should be grateful if you send me further details of the job advertised.
C. Could you send to me further details of the job advertised?
D. I think you should send me details of the job advertised.
94. The rate of inflation has Allen steadily during recent months.
A. There has been a steady decline of the rate of inflation during recent months.
B. There has been steady fall in the rate of inflation during recent months.
C. There has been a steady decline in the rate of inflation during recent months.
D. There has been a steady rate fall in inflation during the recent months.

95. Her husband died. When she heard the news, she fainted.
A. On hearing the news of her dead husband she fainted.
B. When she heard the deadly news of her husband she fainted.
C. When hearing the news from her dead husband she fainted.
D. On hearing the news of her husband's death she fainted.

II.

Choose one sentence (A, B,C or D) that is best written based on the words provided.

96. Your car / park / here / last two hours / and that / wrong.
A. Your car has parked here for the last two hours and that is wrong.
B. Your car has been parked here for the last two hours and that is wrong.
C. Your car has been parking here for the last two hours and that is wrong.
D. Your car has parked here last two hours and that is wrong.

97. We / arrive / airport / time / cup qf tea / before / plane leave.


A. We arrived at the airport on time to have a cup of tea before the plane left.
B. We arrived in the airport in time to have a cup of tea before the plane left.
C. We arrived in the airport in time and had a cup of tea before the plane left.
D. We arrived at the airport in time to have a cup of tea before the plane left.

98. demand / the report / not / photocopy.


A. We demanded that the report was not photocopied.
B. We demanded the report not to be photocopied.
C. We demanded that the report not be photocopied.
D. We demanded the report to not be photocopied.

99.I / angry / myself/ make such / stupid mistake.


A. I was angry at myself for making such a stupid mistake.
B. I was angry with myself about making such a stupid mistake.
C. I was angry about myself for making such a stupid mistake.
D. I was angry with myself for making such a stupid mistake.

100. uncle / make / mind / go / Canada / vacation.


A. My uncle has made his mind to go to Canada for his vacation.
A.My uncle has made up his mind to go to Canada for his vacation.
B. My uncle has made up his mind to go to Canada for vacation.
C. My uncle has minded going to Canada for a vacation.

English Objective Test 21.


Time allowed : 90 min.

Part A. PHONE C
I.

Choose one word A, B, C or D whose underlined part is pronounced differently from the
others.

1.

A. transition

B. obstacle

C. automatic

D. access

2.

A. basic

B. absorb

C. disease

D. preserve

3.

A. surgery

B. sanguine

C. regret

D. regard

4.

A. pension

B. prevention

C. element

D. metropolis

5.

A. pollution

B. sulphur

C. brunch

D. product

II.

Choose one word A, B, C or D that has a different stress pattern from the others.

6.

A. founder

B. reject

C. outwit

D. involve

7.

A. volcanic

B. potential

C. mechanic

D. mineral

8.

A. supervisor

B. delicacy

C. occupation

D. intimacy

9.

A. environment B. disappointment C. deficiency

D. investigate

10.

A. vacancy

D. possession

B. satellite

C. property

Part B. LEXICO - GRAMMAR


I.
11.

Choose the most suitable pronoun A, B, C or D to complete the sentences.


My thanks to my family, without.............none of this would have been possible.
A. whom

12.

D. whose

B. whom

C. that

D. which

Anyone . wants to help should leave their contribution in this box.


A. whose

14.

C. that

There were a lot of survivors, the majority of were in pain.


A. who

13.

B. which

B. that

C. whom

Wasn't there some German author beginning with a K in

D. which
novels -"- individuals got

lost in wonderful mazes?


A. Which
15.

B. who

C. whose

D. whom

I was interested to see that those ........... felt strong about the issue were getting very
worked up.
A. that

16.

B. who

C. whom

Much of...........has been said will soon be forgotten.

D. which

A. what
17.

B. whoever

C. whatever

D. however

B. which

C. this

D. what

We spent a weekend in Venice...............we decided to get engaged.


A. when

20.

D. these

I had a holiday in Rome last year, ......... is where it all began.


A. that

19.

C. those

You can say............you like. However, there is no substitude for hard work.
A. whichever

18.

B. which

B. where

C. whose

D. why

They just talked about...................they had first met.


A. when

B. that

C. whatever

D. whenever

II.

Choose the most suitable word /phrase to fill in each space.

21.

The doctor's mistakes in judgment............the death of the patient.


A. resulted from

22.

B. worn out

C. used up

D. wasted away

B. came up for C. came up with D. came out of

.......... the invention of the steam engine, most forms of transport were horse-drawn.
A. Similar to

25.

D. led to

The teacher asked a difficult question, but finally Ted..........a Good answer.
A. came up to

24.

C. brought to

You'll have to buy some new shoes as these are ............


A. broken out

23.

B. broke off

B. Prior to

C. In addition to D. With reference to

It is important that you reply to our letter without ..........


A. pause

B. fault

C. delay

D. stoppage

15. The name of the book was on the .............. of my tongue, but I just could not think of

it.
A. peak
15.

26.

D. tip

A. keep up with

B. keep away from

C. keep in touch with

D. put up with

We went to the railway station to........... our friends.


B. see ... off

C. see ... out

We are prepared to overlook the error on this occasion


A. in the light of

28.

C. point

- I had to work hard to .. the other students.

A. get...off
27.

B. top

13. thanks to

C. with a view to

D. set ... out


your previous good work.
D. with regard to

I could tell at a ....................................... that nothing had changed between Barbara and
Edward.
A. glimpse

29.

B. blink

C. wink

I). glance

It is common knowledge that more than of the globe's surface is water.


A. two-third

B. two-thirds

C. second-three

D. second-thirds

30.

The new technology allows products to be made with components than before.
A. fewer

B. less

C. little

D. few

33. Rumor has it that of workers will be made redundant so as to keep the company

alive.
A. a great number B. a great amount C. a great sum

D. a great deal

33. According to our latest survey,........... students are working part-time.

A. the most

B. most

C. most of

D. the most of

46. They show great skills in ........ aspect of school life.

A. most every

B. almost all

C. most all

D. almost every

47. According to the principal, of the teachers are interested in eliminating the mid-term

tests.
A. no

B. none

C. no one

D. not one

48. It is a brilliant idea. .. only trouble is that we don't know how much it will cost.

A. An
49.

B. The

C. A

D. 0

will you be able to find such a good secretary like her, I am sure.
A. No anywhere

B. Anywhere

C. Everywhere

D. Nowhere

50. She'll be coming tonight, ........... I don't know exactly when.

A. provided that

B. even if

C. although

D. however

51. Buy me a dozen of eggs on the way back,...........

A. do you

B. may you

C. will you

D. can you

III. Choose the correct verb form to be used in each numbered blank.
Man has made great strides in all the fields of science, particularly medicine
For instance, research work in the laboratories at last (41) (conquer) ....... poliomyelitis,
one of the most devastating diseases. Although the Salk vaccine (42) (not be) ....... one
hundred percent effective, it (43) (decrease) .......the cases of polio considerably.
Tuberculosis once (44) (know) .......as the white plague (45) (study)

intensively.

As

matter of fact, it is curable if it is detected in its early stages. We still have cancer (46) (deal)

....... with, but research workers and doctors all over the world are striving to find a way to
prevent and cure it.(47) (Judge) .......from past experience we can expect that encouraging
news (48)(issue)....... from time to time.

Heart disease, the greatest killer of mankind, is now in the process of being
overcome. An example of the techniques that (49) (develop) ....... is that of heart
massage: when a heart stops nowadays while the patient is under anesthesia, the

doctor opens the chest, massages the heart and revives the patient. Even a few years
ago, such an operation would (50) (be) ....... inconceivable.
Yes, science, which is just another term for knowledge, is helping mankind.
41. A. conquered

B. is conquering

C. has conquered

D. has been conquering

42. A. is not

B. hasn't been

C. hadn't been

D. won't be

43. A. is decreasing

B. has decreased

C. decreased

D. will decrease

44. A. know

B. knew

C. known

D. to know

45. A. will be studied

B. is studied

C. has studied

D. is being studied

41. A. dealing

B. to deal

C. dealt

D. being dealt

42. A. Being judged

B. Judging

C. To judge

D. Having judged

41.

A. will be issued

B. has been issued C. is issued

43. A. had developed

D. will issue

B. developed

C. are developing

D. have been developed

44. A. be

B. have been

C. has been

D. be being

IV. Choose the correct preposition to fill in each space.


45.

Before they hire anyone, they check up.......his background.


A. about

B. on

C. for

D. with

41. Since it's important, would you attend .........that matter right away?

A. of
53.

B. with

C. to

D. by

Now that Anne has graduated, she is no longer dependent

for

financial

support.
A. on
54.

B. with on

C. to about

D. to on

B. for

C. with

D. to

Don't take ........the question, come.......the point.


A. about into

57.

D. of

He does not show much consideration........ his wife's wishes.


A. at

56.

C. from

Why did you quarrel .........your friends ...... such a small matter?
A. with about

55.

B. in

B. round for C. around to

D. about to

I wouldn't give away that ring ....... anything in the world, so once and don't ask me
again.
A. to

58.

B. for

C. by

D. with

I'd say there were 5,000 people there..........a guess.


A. at

B. for

C. with

D. on

59.

If this plan is satisfactory ........ you. I will present it to the committee.


A. to

60.

B. of

D. in

That man's personality makes him unfit........any higher position.


A. with

V.

C. with

B. to

C. about

D. for

Decide which of the four underlined words/phrases contains a mistake.

(61) Each of the four types of human tooth are suited for a specific purpose.

(62) Most bacteria have strong cell walls much like that of plants.

(63) The electric toaster was one of the earliest appliance to be developed for the

A
B
C
kitchen.
D
(64) Despite most mushrooms are edible, some species cause serious poisoning.
A
B
C
D
(61) A basic knowledge of social studies, such as history and geography, are
A
B
C
considered a basic part of the education of every child.
D
(62) Political science, alike the other social sciences. is not an exact science.
A
B
C
D
(63) The most widely used material for package consumer goods is cardboard.
A
B
C
D
(64) Some plants and insects exhibit so high degree of interdependence that the
A
B
elimination of one results in the elimination of the other.
C
D
(69) Water polo is a game in which is played in the water bv two teams, each with
A
B
C
D
seven players.
(70) The principles used in air-conditioning are basically the same as those used
A
B
C
by the human body to cool himself.
D
Part C. READING
I. Read the following article and answer questions 71 80 by choosing the
corresponding letter A, B, C, or D.
Researchers in the field of psychology have found that one of the best ways to make an
important decision, such as choosing a university to attend or a business to invest in, involves
the utilization of a decision worksheet. Psychologists who study optimization compare the
actual decisions made by people to theoretical ideal decisions to see how similar they are.

Proponents of the worksheet procedure believe that it will yield optimal, that is, the best
decisions. Although there are several variations on the exact format that worksheets can take,
they are all similar in their essential aspects.
Worksheets require defining the problem in a clear and concise way and then listing all
possible solutions to the problem. Next, the pertinent considerations that will be affected by
each decision are listed, and the relative importance of each consideration or consequence is
determined. Each consideration is assigned a numerical value to reflect its relative
importance. A decision is mathematically calculated by adding these values together. The
alternative with the highest number of points emerges as the best decision.
Since most important problems are multifaceted, there are several alternatives to choose
from, each with unique advantages and disadvantages. One of the benefits of a pencil and
paper decision-making procedure is that it permits people to deal with more variables than
their minds can generally comprehend and remember. On the average, people can keep
about seven ideas in their minds at once. A worksheet can be especially useful when the
decision involves a large number of variables with complex relationships. A realistic example
for many college students is the question "What will I do after graduation?" A graduate might
seek a position that offers specialized training, pursue an advanced degree, or travel abroad
for a year.
A decision-making worksheet begins with a succinct statement of the problem that will also
help to narrow it. It is important to be clear about the distinction between long-range and
immediate goals because long-range goals often involve a different decision from short-range
ones. Focusing on long-range goals, a graduating student might revise the question above to
"What will I do after graduation that will lead to a successful career?"
71. What does the passage mainly discuss?
A. A tool to assist in making complex decisions.
B. A comparison of actual decisions and ideal decisions.
C. Research on how people make decisions.
D. Differences between long-range and short-range decision making.

72. The word "essential" in paragraph 1 is closest in meaning to .........


A. introductory

B. changeable

C. beneficial

D. fundamental

73. Of the following steps, which occurs before the others in making a decision
worksheet?
A. Listing the consequences of each solution.
B. Calculating a numerical summary of each solution.
C. Deciding which consequences are most important.

D. Writing down all possible solutions.

74. According to decision-worksheet theory, an optimal decision is defined as one that


A. has the fewest variables to consider.
B. uses the most decision worksheets.
C. has the most points assigned to it.
D. is agreed to by the greatest number of people.

75. The author organizes paragraph 2 by .


A. describing a process.
B. classifying types of worksheets.
C. providing historical background.
D. explaining a theory.

76. The author states that "On the average, people can keep about seven ideas in their
minds at once" (paragraph 3) to explain that.....................
A. most decisions involve seven steps. z
B. human mental capacity has limitations.
C. some people have difficulty making minor as well as major decisions.
D. people can learn to keep more than seven ideas in their minds with practice.

77. The word "succinct" in paragraph 4 is closest in meaning to


A. creative

B. satisfactory

C. personal

D concise

78. Which of the following terms is defined in the passage?


A. Proponents (para. 1)

B. Optimal (para. 1)

C. Variables (para. 3)

D. Long-range goals (para. 4 )

79. The word "it" in paragraph 4 refers to...................................


A. worksheet

B. problem -

C. distinction

D. decision

80. The word "revise" in paragraph 4 is closest in meaning to . .


A. ask

B. explain

C. change.

D. predict

II. For questions 81 90, choose the best word/phrase to fill in the blank.
Have you ever asked yourself what you are working for? If you have ever had the time to
consider this taboo question, or put it to others in moments of weakness or confidentiality, you
(81)....... well have heard some or all of the (82) ....... It's the money, of course, some say with
a smile, as if explaining something to a child. Or it's the satisfaction of a task well-done, the
sense of achievement behind the clinching of an important (83) ....... I worked as a bus

conductor once, and I can't say I (84) ....... the same as I staggered along the swaying gangway
trying to (85) ....... out tickets without falling over to someone's lap. It's the company of other
people

perhaps, but if that is the (86) ......., what about farmers? Is it the

conversation in the farmyard that keeps them captivated by the job? Work is power and a
sense of status say those (87) ....... have either attained these elusive goals, or feel aggrieved
that nobody has yet recognized their leadership qualities. Or we can blame it all on someone
else, the family or the taxman, I suspect, and I say this under my breath, that most of us work
rather as Mr. Micawber lived, hoping for something to (88) ....... up. We'll win the pools, and
tell the boss what we really think. We'll scrape together the (89) .......and open that little shop
we have always dreamed of, or go round the world, or spend more time in the garden. One
day, we'll get that (90) ....... we deserve, but until then at least we have something to do. And
we are so busy doing it that we won't have time to wonder why.
(81)

A. might

B. can

C. will

D. should

(82)

A. below

B. rest

C. following

D. latter

(83)

A. deal

B. position

C. job

D. engagement

(84)

A. enjoyed

B. wished

C. hoped

D. felt

(85)

A. make

B. turn

C. issue

D. give

(86)

A. one

B. case

C. question

D. problem

(87)

A. people

B. must

C. who

D. may

(88)

A. move

B. tur

C. ease

D. end

(89)

A. resources

B. opportunities

C. riches

D. money

(90)

A. ambition

B. station

C. vocation

D. promotion

Part D. WRITING
I. Choose one sentence A, B, C or D that is best written based on the words provided.
91. Should/you/want/know/further information/holiday/not hesitate/ask.//
A. Should you want to know further information about the holiday, not to hesitate to

ask.
A. Should you want to know further information of the holiday, don't hesitate to ask.
B. Should you want to know further information about the holiday, don't hesitate

asking.
B. Should you want to know further information about the holiday, don't hesitate to

ask.
92. No sooner/I/tell her the news/burst out/cry.//
A. No sooner did I tell her the news than she burst out crying.

B. No sooner had I told her the news than she burst out crying.
C. No sooner had I told her the news that she burst out crying.
D. No sooner I told her the news than she burst out crying.

93. You/entitle/drive a car/ your driving license.//


A. You are not entitled to drive a car without your driving license.
B. You are not entitled to drive a car with your driving license.
C. You do not entitle to drive a car without your driving license.
D. You are not entitled to driving a car without your driving license.

94. Privatization/think/beneficial/in that/promote competition.//


A. Privatization is thought to be beneficial in that it promotes competition.
B. Privatization is thought as beneficial in that it promotes competition.
C. Privatization is thought to be beneficial in that promotes competition.
D. Privatization thought to be beneficial in that it promotes competition.

95. No point/join/club/full/eccentrics.//
A. There is no point in joining that club because it seems to be full with eccentrics.
B. There is no point of joining that club because it seems to be full of eccentrics.
C. There is no point in joining that club because it seems to be full of eccentrics.
D. It is no point in joining that club because it seems to be full of eccentrics.

II.

Choose one sentence A, B, C or D that has the same meaning as the original one.

96. We couldn't have managed without my father's money.


A. If it were not for my father's money, we couldn't have managed.
B. If it hadn't been for my father's money, we couldn't have managed.
C. With my father's money, we could have managed.
D. If my father had left us some money, we could have managed.

97. I only recognized him when he came into the light.


A. Not until he came into the light that I recognized him.
B. Not until did he come into the light that I recognized him.
C. Not until he came into the light did I recognize him.
D. Only when he came into the light that I recognized him.

98. You must submit articles for the magazine by June 16th.
A. Articles must he submitted for the magazine long before June 16th.
B. The final date for you to submit articles to the magazine is June 16th.
C. The only day that you can submit articles for the magazine is June 16D. The final date for you to submit articles for the magazine is June 61h.

99. I left without saying goodbye as I didn't want to disturb the meeting.

A. Rather than disturb the meeting, I left without saying goodbye.


B. Rather than disturbing the meeting, I left without saying goodbye.
C. I'd rather leave without saying goodbye than disturbing the meeting.
D. On leaving without saying goodbye, I didn't want to disturb the meetir_.:

100. I had only just put the phone down when the boss rang hack.
A. Hardly did I put the phone down when the boss rang back.
B. Hardly had I put the phone down when the boss ram), back.
C. Hardly had I put the phone down that the boss rang back.
D. Hardly I put the phone down when the boss rang back.

English Objective test 22


time allowed: 90 min
Part A. PHONETICS
I. Choose one word A, B, C or D whose underlined part is pronounced differently from
the others.
1.

A. associate

B. abnormality

C. operation

D. grateful

2.

A. educational

B. gesture

C. explanation

D. appreciate

3.

A. honesty

B. hydrogen

C. horrible

D. harmful

4.

A. treatment

B. beam

C. deafness

D. release

5.

A. personal

B. disposal

C. atmosphere

D. preserve

II. Choose one word A, B, C or D that has a different stress pattern from the others.

6.

A. absorb

B. exhaust

C. focus

D. resource

7.

A. introduce

B. attendance

C. encounter

D. employment

8.

A. tolerate

B. company

C. mineral

D. organic

9.

A. investigate

B. miraculous

C. environment

D. disappointment

10.

A. technology

B. particular

C. intimacy

D. accessible

Part B. Lexico Grammar


I. Choose the Most suitable phrasal verb A, B, C or D to complete the following
sentences.

11.

The meeting didn't ............. until late.


A. end up

11.

B. break up

B. adds up to C. feels up to

up to

B. get up to

C. come down to D. get down to

B. follow up

C. drop off

a break up

It's no good pretending,-you've got to ..............reality.


A. bargain for B. come up against C. face up to

16.

17.

A. bringing about

B. getting up to

C. coming up with

D. getting round to

You should always have an alternative plan to ...............


C. feel up to

D. fall back on

B. asked after me

C. asked me after D. called up me

The school examination for eleven-year olds was...........some years ago.


A. followed up

20.

B. ask for

When they ............, I had to report to the nearest barracks.


A. called me up

19.

D. get down to

What were you two. just now in the garden?

A. bring about
18.

come down to

At first, Tim insisted that he was right, but then began to..............
A. back down

15.

D.

The hotel didn't.............. my expectation.


A. come

13.

D. fall through

In the end it all .......... a question of trust.


A. vets round to

12.

C. come about

B. drawn up

C. carried out

D. done away with

When I took over the business I got more than...............


A. asked after

B. bargained for

C. drew up....

D. came in for

I. Choose the most suitable word/phrase A, B, C, or D to complete each sentence.


21. It is important to build and maintain a fire in such fearful weather since you will die of
cold once the fire goes
A. away

B. down

C. off

D. out

22. Hemingway made of his own experience of life to 1A,Tite many successful anti-war
novels such as For Whom The Bell Tolls or A Farewell To Arms.

A. fun

B. use

C. a point

D. a habit

23. It is . your own sake that you work hard to prepare for the coming exam.
A. by

B. of

C. for

D. on

24. The experts of WHO are trying their best to play. the effects of SARS on human
beings.
A. down

B. out

C. up

D. with

25. It is . human beings to decide the fate of many endangered species.


A. for
26.

B. up to

C. up on

D. of

Many people often irresponsibly throw garbage on the street as a ...........

of

convenience.
A. result
27.

B. fact

C. matter

D. problem

After graduating from high schools, almost every student makes great ......... to pass
the entrance exam to university.
A. progress

B. attempts

C. impressions D. improvements

28. It is highly recommended that language learners in general and English learners

should study the language in the native community so as to master it.


A. on the other hand B. in short
29.

C. on the whole

D. in particular

the development in medical technology, many serious diseases have been


successfully treated.
A. Thanks to

B. In spite of C. Because of

D. In view of

30. People should take.......... of their own health to maintain a happy life.

A. notice

B.. care

C. account

D. advantage

31. According to surveys, the *majority of Britons want capital punishment


restored.

A. the most

B. most

C. most of

D. the most of

32. I suppose you just go to the speaking club improve your communicative ability.

A. so as

B. for

C. in order to

D. so that

33. Nowadays, due to the increasing unemployment rate, young graduates have
......opportunities to find a job.

A. little

B. a little

C. few

D. a few

34. You'll get a free month's subscription, you renew your membership by the end of
May.

A. unless

B. however C. although

D. provided

35. It seems that.......... of the students arc interested in going to the art museum.

A. no

B. no one -

C. none

D. nobody

36. It takes ........ of time to master a language.

A: a great deal

B. a great number

C. a great many -

D. a great quantity

37. The reason so many species are becoming extinct is that people have
destroyed their habitats through deforestation.
A. which

B. when

C. what

D. why

38. What a dangerous thing to do! You..........have been killed.

A. should

B. can

C. must

D. might

39. Buy me a newspaper on your way back, ,.......,?

A. do you

B. will you

C. have you

D. may you

40. February 14th is Valentine's Day......... people exchange roses and chocolates as a sign

of love.
A. which

B. when

C. where

D. what

III. Choose the most suitable verb form A, B, C, or D to complete each sentence.
41. Many environmentalists have warned drivers not

leaded petrol since it is harmful

to the environment.
A. use

B. to use

C. using

D. used

42. By the time human beings become fully aware of the importance of the ozone layer, it

-........................................................... seriously damaged.


A. will be
43

B. will have been

C. is

D. has been

.. the environment is a challenging task which requires the cooperation of


all nations in the world.
A. Protecting

B. Protect

C. To protect

D. To be protecting,

44. Left-hand traffic, a custom in Britain only, dates back to the days when English

people went to and fro on horseback.


A. existed

B. exists

C. existing

45. Jack London used the knowledge and experience

D. to exist
during the time he joined the

Alaskan 'gold rush' as the materials for many of his literature works.
A. gained.

B. being gained

C. gaining

D. to again

46. It is high time chemical factories stopped their unprocessed garbage into the sea.

A. release

B. to release

C. releasing

D. to be releasing

47. There is a world of difference between the way animals with each other and that of

mankind.

48

A. to communicate

B. communicate

C. communicating

D. communicated

very hard from the beginning, she finally passed the exam with the
highest mark.
A. Work

49.

B. Working

C. To work

D. Worked

The celebration of Tet holiday, like many other Vietnamese traditions, are said
from China.
A. originate

B. originating

C. to originate

D. originated

49. Men often shake hands when first ....... to each other.

A. introduce

B. to introduce

C. introducing

D. introduced

IV. Choose the correct preposition A, B, C, or D to complete the passage.


Women in the legal profession are still fallinu, behind their male colleagues, according to
figures (51) ........the NSW Judicial Commission. A discussion paper (52) ........ gender
bias and the law released yesterday found women were underrepresented (53) ........
senior legal ranks, comprising less than 10% of NSW judges.
The Commission also found that only 23% of lawyers (54) ....... private law firms are
women and women held only 10% (55) .......bar positions.
"Disappointingly, this paper finds that women lawyers have been denied opportunities to
earn the same amount of money (56) ....... their male colleagues", Minister for the status
of women, Kerry Chikarovski said. She said one (57) ....... five (58) ....... the women
interviewed said they had resigned (59) ....... their position or changed the job as a result
(60) ....... sex discrimination.
51. A. in

B. by

C. from

D. of

52. A. on

B. about

C. of

D. in

53. A. by

B. in

C. with

D. over

54. A. in

B. from

C. for

D. of

55. A. out of

B. in

C. of

D. with

56. A. like

B. for

C. with

D. as

57. A. of

B. in

C. from

D. within

58. A. among

B. from

C. of

1). out of

59. A. from

B. in

C. to

D. over

60. A. from

B. of

C. to

D. in

V. Decide which of the four underlined words/phrases A, B, C, or D contains a istake.


(61) Mary usually arrives at the office at nine o'clock, but because the storm, she

was two hours late.


D
(62) Danny spent such enjoyable vacation in Europe this summer that he plans to

return as soon as he saves enough money.


D
(63) Electric milking machines have made dairy farming a much easier job than
A

it once did.
D
(63) Tulip tree, the tallest broadleaf plant in the eastern United States, may reach

height of over 200 feet.


D
(65) There are approximately 600 different species of trees native of the continenth
A

United States.
(66)Jane must have called her sister last night, but she arrived home too late to ca her.
A

(67)Unlike competitive running, race walkers must always keep some portion of
A

their feet in contact with the ground.


D
(65) The various parts of the body require so different surgical skills that many
A

surgical specialties have developed.


D
(66) No sooner had he been appointed to the post when the new editor fell ill.
A

(67) Not longer are contributions to the advancement of industry made primarily
A
by individuals.

Pat C. READING
I. Read the following article and answer questions 71 80 by choosing A, B, C, or D.
Ranked as the number one beverage consumed worldwide, tea takes the lea over coffee in
both popularity and production with more than 5 millia metric tons of tea produced annually.
Although much of this tea is consumed in Asian, European and African countries, the United
States drinks its fair share. According to estimates by the Tea Council of the United States, tea
is enjoyed by no less than half of the U.S. population on any given day. Black tea or green tea
iced, spiced, or instant tea drinking has spurred a billion-dollar business with major tea
producers in Africa and South America and throughout Asia.
Tea is made from the leaves of an evergreen plant, Camellia sinensis, which grows tall and
lush in tropical regions. On tea plantation, the plant is kept trimmed to approximately four
feet high and as new buds called flush appear, they are plucked off by hand. Even in today's
world of modern agricultural machinery, hand harvesting continues to be the preferred
method. Ideally, only the top two leaves and a bud should be picked. This new growth
produces the highest quality tea.
After being harvested, tea leaves are laid out on long drying racks, called withering racks, for
18 to 20 hours. During this process, the tea softens and becomes limp. Next, depending on
the type of tea being produced, the leaves may be crushed or chopped to release flavor, and
then fermented under controlled conditions of heat and humidity. For green tea, the whole
leaves are often steamed to retain their green color, and the fermentation process is skipped.
Producing black teas requires fermentation during which the tea leaves begin to darken. After
fermentation, black tea is dried in vats to produce its rich brown or black color.
No one knows when or how tea became popular, but legend has it that tea as a beverage was
discovered in 2737B.C. by Emperor Shen Nung of China when leaves from a Camellia
dropped into his drinking water as it was boiling over a fire. As the story goes, Emperor Shen
Nung drank the resulting liquid and proclaimed the drink to be most nourishing and
refreshing. Though this account cannot be documented, it is thought that tea drinking
probably originated in China and spread to other parts of Asia, then to Europe, and ultimately
to America colonies around 1650.
With about half the caffeine content as coffee, tea is often chosen by those who want to
reduce, but not necessarily eliminate their caffeine intake. Some people find that tea is less
acidic than coffee and therefore easier on the stomach. Others have become interested in tea
drinking since the National Cancer Institute published its findings on the antioxidant
properties of tea. But whether tea is enjoyed for its perceived health benefits, its flavor, or as

a social drink, teacups continue to be filled daily with the world's mos: popular beverage.
71. Why does the author include statistics on the amount of tea produced, so:: and
consumed?
A. To show the expense of processing such a large quantity of tea.
B. To. explain why coffee is not the most popular beverage worldwide.
C. To demonstrate tea's popularity.
D. To impress the reader with factual sounding information.

72. Based on the passage, what is implied about tea harvesting?


A. It is totally done with the assistance of modern agricultural machinery.
B. It is no longer done in China.
C. The method has remained nearly the same for a long time.
D. The method involves trimming the uppermost branches of the plant.

73. What does the word "they" in paragraph 2 of the passage refer to?
A. tea pickers

B. new buds

C. evergreen plants

D. tropical regions

74. Which of the following is NOT true about the tea production process?
A. Black tea develops its dark color during fermentation and final dryilL:
B. Green tea requires a long fermentation process.
C. Green tea is often steamed to keep its color.
D. Black tea goes through two drying phases during production.

75. The word "documented" in paragraph 4 could be best replaced by w'f-- the
following word?
A. ignored

B. proved

C. stored

D. kept

76. According to the passage, what is true about the origin of tea drinking:
A. It began during the Shen Nung dynasty.
B. It may have begun some time around 1650.
C. It is unknown when tea first became popular.
D. It was originally produced from Camillia plants in Europe.

77. The word "eliminate" in paragraph 5 could be best replaced by whi following
word?
A. decrease

B. increase

C. reduce

D. remove

78. According to the passage, which may be the reason why someone would choose to drink
tea instead of coffee?
A. Because it's easier to digest than coffee.
B. Because it has a higher nutritional content than coffee.
C. Because it helps prevent cancer.
D. Because it has more caffeine than coffee.

79. Where in the passage does the author mention research conducted on the beneficial effects
of tea drinking?
A. In paragraph 1

B. In paragraph 2

C. In paragraph 4

D. in paragraph 5

80. What

best describes the topic of this passage?

A. Tea consumption and production.


B. The two most popular types of tea.
C. The benefits of tea consumption worldwide.
D. How tea is produced and brewed.

II.Choose the best word/phrases A, B, C, or D to complete the following passage.


Probably the most famous film commenting on twentieth-century technology- is Modern
Times, (81) ....... in 1936. Charlie Chaplin was motivated to make the film by a reporter who,
while interviewing him, (82) ....... to describe working conditions in industrial Detroit.
Chaplin was told that healthy young farm boys were lured to the city to work on automotive
assembly lines. Within four or five years, these young men's health was (83) ....... by the stress
of work in the factories. Scenes of factory interiors account for only about (84) ....... of the
footage of Modern Times, but they (85) ....... some of the most pointed social commentary as
well as the most comic situations. No one who has seen the film can ever forget Chaplin
vainly trying to (86).......... the fast-moving (87).......... belt, almost losing his mind in the
process. Another popular scene involves an automatic feeding machine brought to the
assembly line (88)....... workers need not interrupt their labor to eat. The feeding machine
malfunctions, hurling food at Chaplin who is strapped into his position on the assembly line
and cannot escape. This serves to illustrate people's utter (89) ....... in the face of machines that
are meant to serve their basic needs.

Clearly, Modern Times has its faults, but it remains the best film treating
technology within a social context. It does not offer a radical social message,
but it does (90) reflect the sentiments of many who feel they. are victims of an overmechanized world.
(81)

A. make

B. making

C. made

D. to make

(82)

A. forced

B. required

C. happened

D. managed

(83)

A. enhanced

B. ruined

C. shattered

D. broke

(84)

A. first third

B. first three

C. one three

D. one third

(85)

A. contain

B. consist

C. compose

D. make

(86)

A. keep face with

B. keep pace with C. keep track of D. keep time of

(87)

A. conveying

B. conveyed

C. convey

D. conveyor

(88)

A. so that

B. therefore

C. in order to

D. thus

(89)

A. meaninglessness B. meaningfulness C. helplessness D. helpfulness

(90)

A. exactly

B. accurately

C. nearly

D. approximately

Part D. WRITING
I.

Choose one sentence A, B, C or D that is best written based on the words


provided.

91. He/ catch/ red-handed/ he/ have/ no choice/ confess.//


A.He caught red-handed so he had no choice but to confess.
B. He was caught red-handed so that he had no choice but to confess.
C.He was caught red-handed so he had no choice of confessing.
D.He was caught red-handed so he had no choice but to confess.

92. Hardly/ I/ water/ garden/ begin/ rain.//


A.Hardly did I water the garden when it began to rain.
B. Hardly had I watered the garden when it began to rain.
C.Hardly 1 watered the garden than it began to rain.
D.Hardly did I water the garden than it had begun to rain.

93. It/ common knowledge/ the Olympic Games/ hold/ every four years.//
A.It is common knowledge that the Olympic Games are held every four years.
B. It is a common knowledge that the Olympic Games are held every four years.
C. It is common knowledge that the Olympic Games is held every four years.
D. It is a common knowledge that the Olympic Games is held every four years.

94. Gone! days/ when! you/ buy/ three-course meal/ under $1.001!

A. Gone are the days when you bought a three-course meal for under $1.00.
B. Gone to the days when you could buy a three-course meal for under $1.00.
C. Gone are the days when you could buy a three-course meal for under $1.00.
D. Gone are the days that you could buy a three-course meal for under $1.00.

95. New machinery/ put in/ this year/ with a view/ increase output.!!
A. New machinery has been put in this year with a view of increase output.
B. New machinery has been put in this year with a view to increasing, output.
C. New machinery has been put in this year with a view to increase output.
D. New machinery has been put in this year with a view of inereasine. output.

H.

Choose one sentence A, B, C or D that has the same meaning as the original one.

96. Alice and Charles did not decide to move to a bigger house until after the birth of their
second child.
A. Only when Alice and Charles had their second child that they decided to move to a

bigger house.
B. Only when Alice and Charles had their second child did they decide to move to a

bigger house.
A. Only when had Alice and Charles had their second child than they decided to move to

a bigger house.
B. Only when Alice and Charles had their second child,they decided to move to a bigger

house.
97. Customs officials are stopping more travelers than usual this week.
A. An increased number of travelers stopped by customs officials this week.
B. The increased number of travelers are being, stopped by customs officials this week.
C. An increased number of travelers are being stopped by customs officials this week.
A. The increased number of travelers is being stopped by customs officials this week.

98. The demand was so great that they had to reprint the book immediately.
A. So great the demand was that they had to reprint the book immediately.
B. So great was the demand so they had to reprint the book immediately.
C. So great was the demand that they had to reprint the book immediately.
D. So great the demand was, they had to reprint the book immediately.

99. When the Minister was asked about the strike, he declined to comment.
A. On he was asked about the strike, the Minister declined to comment.
B. On being asked about the strike by the Minister, he declined to comet
C. On being asked about the strike, the Minister declined to comment.
D. On asked about the strike, the Minister declined to comment.

100. I'd like you to wait until they bring out a new model.
A. I'd rather you waited until they bring out a new model.
B. I'd rather you to wait until they bring out a new model,
C. I'd rather you wait until they bring out a new model.

D. I'd rather you waiting until they bring out a new model.

ENGLISH OBJECTIVE 23
Time allowed: 90 min
Part A. PHONETICS
I.

Choose one word A, B, C, or D whose underlined differently from the others..

1.

A. deficiency

B. delicate

C. conspicuous

D. discard

2.

A. measure

B. treasure

C. eardrum

D. deafness

3.

A. supervisor

B. university

C. invaluable

D. eruption

4.

A. honorable

B. honesty

C. historic

D. heir

5.

A. element

B. employment

C. gesture

D. extra

II.

Choose one word A, B, C or D that has a different stress pattern from the others.

6.

A. emission

B. encounter

C. impudence

D. outnumber

7.

A. logical

B. mechanic

C. ancestry

company

8.

A. apprenticeship B. automatic

C. contaminate

D. Australia

9.

A. guidance

B. favor

C. pension

D. disease

10.

A. promotion

B. satellite

C. property

D. concentrate

Part B. LEXICO - GRAMMAR


I. Choose the most suitable word A, B, C, or D to fill in each space.
11. People living abroad are not ........... to enter for this competition.
A. enabled.

B. permissible

C. capable

D. eligible

12. Unfortunately, our local cinema is on the..........of closing down.

A. verge

B. hint

C. edge

13. For elderly people, one of the problems.............by

D. threat
rising

prices

is

the

continual increase in heating bills.


A. given

B. posed

C. pressed

D. forced

14. The price they offered for my car was so low that I ......... it down.

A. brought
15.

C. turned

D. shouted

When his business failed, he started again from....................


A. scratch

16.

B. called

B. blank

C. introduction D. beginning

The number of tickets available is ................. by the size of the stadium.


A. caused

B. related

C. determined

D. associated

17.

My................... on life has changed a lot since leaving university.


A. outlook

18.

15.

B. ring

C. set

D. wind

B. mock

C. fake

D. unreal

B. except

C. other

D. apart

B. priceless

C. worthless

D. invaluable

B. arrested

C. judged

D. charged

B. In the time of C. By reason of D. In the event of


B. taken of

C. taken over

D. taken in

His wife's death was a terrible shock and it took him a long time to ................. it.
B. come through C. go over

D. get over

He lives a(n) ................throw from the station.


A. stone's

29.

D. ought

Mr. Jones has...................painting since he retired.

A. get round
28.

C. should

a fire, hotel guests are asked to remain calm.

A. taken up
27.

B. would

Choose the most suitable phrase A, B, C, or D to complete each sentence.

A. As a result of
26.

D. condense

Whoever....................of speeding may be fined from $100 up to $1,000.


A. convicted

II.

C. reduce

The ring is only made of plastics; it is quite ...................


A. valuable

25.

B. decrease

There was nothing special about his clothes....................from his flowery tie.
A. but

24.

D. fault

The..................exam in January prepared pupils for the real thing in June.


A. false

23.

C. mistake

Don't forget to .................... the alarm clock for six o'clock tomorrow morning.
A. put

22.

B. lack

I wish you'd tell me what I.................- do in this difficult situation.


A. shall

21.

D. purpose

It would be helpful if you could ..................the report into three or four pages.
A. resume

20.

C. approach

He was always finding..................with his daughter's friends.


A. blame

19.

B. attitude

B. stick's

C. arrow's

D. apples

A good friend will always..................you when you are in trouble.


A. stand by

B. stand up

C. stand for

D. stand up against

30.

It's time we had a(n) ....... talk with each other as you used to be a good friend.
A. eye-to-eye

31.

32.

B. face-to-face

I'm fed up with doing tile same thing every weekend. Let's go away .......
A. as an adjustment

B. for a change

C. as an alternative

D. for an amendment

I usually buy my clothes ....... It's cheaper than going to a dressmaker.


A. off the peg

33.

C. heart-to-heart D. cheek-to-cheek

B. on the house C. in public

It's not surprising that he became a writer because he always longed to see his name
A. in type............................................B. in print

34.

B. Almost all

B. In spite of

C. Most all

D. Almost every

C. Provided that D. As soon as

....... all my efforts, I will not have the report ready by Friday.
A. In spite

37.

D. in edition

....... you have the money in your account, you can withdraw up to $100 a day.
A. Unless

36.

C. in letters

The new principle is agreed upon by..student at this school.


A. Most every

35.

D. on the shelf

B. No matter

C. However I make D. Despite

....... of workers, who went on the strike last week, were dismissed by the Board of
Managers.

38.

A. A great number

B. A great amount

C. A great sum

D. A great deal

The hotel didn't ....... my expectations.


A. come up to

39.

B: get up to

C. come up with D. get up with

....... we get to the theatre, the play will have started.


A. As long as

B. Until

C. By the time

D. Whenever

III. Choose the correct preposition A, B, C, or D to complete each sentence.


40.

I will keep your application ............... file for the time being.
A. in

41.

B. with

C. on

D. at

We can judge the success of your scheme only by taking . account the financial
benefits over the next few years.
A. into

42.

B. out

C. from

D. over

The 10% discount is only applicable . items costing over $100.


A. for

B. to

C. against

D. on

44.

My parents do not approve........their children smoking.


A. about

45.

C. for

D. into

B. in in

C. in to

D. for for

B. to of

C. at with

D. to with

He was struck......... by cancer at the age of thirty nine.


A. down

50.

B. in

Hang on........that end of the box very tightly. Don't let go ........ it.
A. at to

49.

B. above under C. over above D. on with

Her eyes was filled ........ tears and she was trembling...... oold
A. with with

48.

D. to

Now, she is engaged...........decorating the new house in North Plains.


A. with

47.

C. of

You will get $50 .........and.........you usual salary.


A. over with

46.

B. on

B. on

C. at

D. out

I'd prefer not to play golf today. I really don't feel up .......... it.
A. for

B. with

C. to

D. in

IV. Decide which of the four underlined parts A, B, C, or D contains a mistake.


(51) According to most psychological studies, body language expresses a
speaker's A
B
C
emotions and attitudes, and it also tends to affect the emotions and attitudes
of the listen.
D
(52) A smile can be observed, described, and reliably identify, it can also be elicited
A
B
C
and manipulated under experimental conditionc.
D
(53) American painter Georgia O'Keeffe is well known as her large paintings of
A
flowers in which single blossoms are presented as if in close-up.
B
C
D
(54) Compare with the jagged estuaries of the Atlantic coast, the Pacific coast seems
A
B
C
almost uniformly straight.
(55) Most authorities consider both dreaming while sleep and daydreaming to be forms of
fantasy.
(56) Genetic engineering is helping researchers unravel the mysteries of previously A B
incurable diseases so that they can get to its root causes and find cures.
C
D
(57) What makes for human skeleton hard and strong is the presence of the metallic
A
B
C
D
element calcium.
(58) To people from temperate climates, tropical butterflies may seem incredible big.
A
B
C
D

(59) Chicago is the third largest publishing center in the United States, exceeding A
A
B
C
only by New York and San Francisco.
D
(60) Water is the only substance that occur at ordinary temperatures in all three
A
B
C
states of matter: solid, liquid and gas.
D
V. Choose the most suitable verb form A, B, C, or D to complete the following
passage.
Society has changed in many ways since the introduction of computers, and people's
lives at home and at the office (61 influence) .................. Most people are working for
fewer hours per week than they (62 use to)

and

manufactures

and

advertising

agencies (63 become) .................. much more interested in how people spend this extra
leisure time. One recent report stated that, although the number of hobbies (64 not increase)
.................., each hobby had become much more specialized.
A second finding is that nowadays, many managers would rather spend time with their
families than (65 stay) ..................late in the office everyday. Some companies now make
managers (66 take).................. their annual .holidays even if they don't want to, because this
(67 - lead) to quite an improvement their performance if they have some rest.
In spite of these changes, some people are working harder than ever before. The standard
of exams (68 - get) .................. higher, and increased competition (69 make).................. it
harder to get into university than it was 20 years ago. School children and students now have
to work so hard that in many cases, they (70 work).................. longer hours than their
parents.
61. A. were influenced
C. are influenced
62. A. are using to
C. have been used to
63. A. are becoming
C. became
64. A. haven't increased
C. didn't increase

B. have been influenced


D. have influenced
B. are used to
D. used to
B. have been becoming
D. had become
B. hadn't increased
D. aren't increasing

65. A. staying

B. to stay

C. stay D. to staying

66. A. take

B. to take

C. to have taken

67. A. will have led

B. leads C. has led

D. is leading

D. taking

68. A. is getting

B. has been getting

C. gets D. has got

69. A. will make

B. is making

C. has been making D. made

70. A. have worked

B. will work

C. work

D. were working

Part C. READING
Read the following article and answer questions 71 80 by circling A, B, C, or D.
ENVIRONMENTAL ACTIVISTS
Paul Watson is an environmental activist. He is a man who believes that he must do
something, not just talk about doing something. Paul believes in protecting endangered
animals, and he protects them in controversial ways. Some people think that Watson is a hero
and admire him very much. Other people think that he is a criminal.
On July 16th, 1979, Paul Watson and his crew were on his ship, which is called the Sea
Shepherd. Watson and the people who work on the Sea Shepherd were hunting on the Atlantic
Ocean near Portugal. However, they had a strange prey; instead of hunting for animals, their
prey was a ship, the Sierra. The Sea Shepherd found the Sierra, ran into it and sank it. As a
result, the Sierra never returned to the sea. The Sea Shepherd, on the other hand, returned to
its home in Canada. Paul Watson and his workers thought that they had been successful.
The Sierra had been a whaling ship, which had operated illegally. The captain and crew
of the Sierra did not obey any of the international laws that restrict whaling. Instead, they
killed as many whales as they could, quickly cut off the meat, and froze it. Later, they sold the
whale meat in countries where it is eaten.
Paul Watson tried to persuade the international whaling, commission to stop the Sierra.
However, the commission did very little, and Paul became impatient. He decided to stop the
Sierra and other whaling ships in any way that he could. He offered to pay $25,000 to anyone
who sank any illegal whaling ship, and he sank the Sierra. He acted because he believes that
the whales must be protected. Still, he acted without the approval of the government;
therefore, his actions were controversial.
Paul Watson is not the only environmental activist. Other men and women are also
fighting to protect the Earth. Like Watson, they do not always have the approval of their
governments, and like Watson, they have become impatient. Yet, because of their concern for
the environment, they will act to protect it.
(Adapted from "Ecofighters" by Eric Schwartz, OMNI)
71. According to the reading, an environmental activist is someone who...........

A. runs into whaling ship.


B. does something to protect the Earth.
C. talks about protecting endangered species.
D. is a hero, like Paul Watson.

72. When something is controversial, ..........


A. everyone agrees with it.

B. everyone disagrees with it.

C. people have different ideas about it.

D. people protect it.

73. The members of a ship's crew are...........


A. the men and women who work on the ship.
B. the people who work on airplanes.
C. all of the people on a ship, including the passengers.
D. the people who own the ship.

74. The main idea of paragraph one is that...........


A. Paul Watson is a hero to some people.
A. activists arc people who do something.
B. Paul Watson is a controversial environmental activist.
B. Paul Watson does not believe in talking.

75. The Sea Shepherd was hunting ............


A. the Atlantic Ocean

B. whales

C. the Sierra

D. Portugal

76. The author implies that Paul Watson lives in.............


A. Portugal.

B. a ship on the Atlantic.

C. the Sierra.

D. Canada.

77. The captain and the crew of the Sierra were acting illegally because.........
A. they were not obeying international laws.
B. they were whaling.
C. they were killing and selling whales.
D. All of the above are correct.

78. In paragraph 3, the phrase "and froze it" refers to .................


A. whale meat

B. the Sierra

C. whales

D. the Sierra crew

79. The main idea of paragraph three is that............


A. the Sierra sold whale meat in some countries.
B. the people on the Sierra did not obey international laws.
C. the people on the Sierra killed as many whales as they could.
D. whaling is illegal according to international law.

80. Watson ran into the Sierra because...........

A. he wanted to stop the ship's crew from whaling.


B. he was impatient with the government's actions.
C. he wanted to protect the whales from the whalers.
D. All of the above are correct.

II.

Choose the best word/phrase A, B, C, or D to complete the following passage.

INFLUENCES OF TELEVISION
Television has changed the lifestyle of people in every industrialized country in the
world. In the United States, where sociologists have studied the effects, some interesting
observations have been mad Television, although not essential, has become a(n) (81)
. part of most people's lives. It has become a baby-sitter, an initiator of
conversations, the major transmitter of culture, a keeper of traditions. Yet when what
can be seen on TV in one day is critically analyzed, it becomes evident that television is
not a teacher but a sustainer. The poor (82). Of programming does not elevate
people into greater (83).., but rather maintains and encourages the status quo.
The (84).. reason for the lack of quality in American television is related to both the
history of TV programming development and the economics of TV. Television in America
began with the radio. Radio companies and their sponsors first (85..

with television.

Therefore, the close relationship, which the advertisers had with radio programs became the
system for American TV. Sponsors not only paid money for time within programs, but many
actually produced the programs. Thus, (86) . from the capitalistic, profit-oriented
sector of American society, television is primarily (87) with reflecting and attracting
society rather than (88).. and experimenting with new ideas. Advertisers want to attract
the largest viewing audience possible; to do so requires that the programs be entertaining
rather than challenging.
Television in America today remains, to a large (89). with the same
organization and standards as it had thirty years ago. The hope for some evolution and true
achievement toward improving society will require a change in the (90) system.
(81) A. integral

B. mixed

C. fractional

D. superior

(82) A. quantity

B. quality

C. effect

D. product

(83) A. preconception

B. knowledge

C. understanding

D. feeling

(84) A. adequate

B. unknown

C. inexplicable

D. primary

(85) A. experimented

B. tried

C. did

D. made

(86) A. going

B. leaving

C. coming

D. getting

(87) A. interested

B. concerned

C. worried

D. connected

(88) A. reflecting

B. innovating

C. attracting

D. entertaining

(89) A. extent

B. degree

C. size

D. amount

(90) A. total

B. full

C. entire

D. complete

Part D. WRITING
I. Choose one sentence A, B, C or D that is best written based op the words provided.
91. Factory/close down/account/economic depression.//
A. The factory had to close down on account for the economic depression.
B. The factory was closed down on account of the economic depression.
C. The factory had to close down on account of economic depression.
D. The factory had to close down on account of the economic depression.

92. gold rush/people/go/Australia/in the hope/find gold.//


A. During the gold rush, many people went Australia in the hope of finding gold.
A. During the gold rush, many people went to Australia in the hope of finding gold.
B. During the gold rush, many people went to Australia in the hope to find gold.
B. During the gold rush, many people went to Australia in the hope for finding gold.

93. you/fancy/go somewhere else/ complete change?


A. Do you fancy going somewhere else for a complete change?
B. Do you fancy going somewhere else as a complete change?
C. Do you fancy to go somewhere else for a complete change?
D. Are you fancy going somewhere else for a complete change?

94. If/we/cut down/unnecessary expenses/we/make/profit/last year//


A. If we cut down on unnecessary expenses, we would make a profit last year.
B. If We had cut down on unnecessary expenses, we would make a profit last year.
C. If we had cut down on unnecessary expenses, we would have made a profit last year.
D. If we cut down on unnecessary expenses, we would have made a profit last year.

95. I/be used/get up early/don't mind/leave/5 o'clock/morning.//


A. I am used to get up early so I don't mind leaving at 5 o'clock in the morning.
B. I am used to getting up early so I don't mind to leave at 5 o'clock in the morning.
C. I am used to getting up early so I don't mind leaving at 5 o'clock in the

morning.
D. I am used to getting up early so I don't mind leaving by 5 o'clock in the

morning.
II.

Choose one sentence A, B, C or D that has the same meaning as the original one

96. It was his incompetence, which led to their capture.

A. If he hadn't been so incompetent, they wouldn't have been captured.


A. If he hadn't been so incompetent, they wouldn't be captured.
B. If he weren't so incompetent, they wouldn't be captured.
C. Due to his incompetence, they would have been captured.

97. Everyone started complaining the moment the announcement was made.
A. No sooner had the announcement been made that everyone started to complain.

A. No sooner had the announcement been made than everyone started to complain.
A. No sooner was the announcement made than everyone started to complain.
A. No sooner had everyone started to complain than the announcement was made.

98. My boss works better when he's pressed for time.


A. My boss has less time; therefore, he works better.
B. The less time my boss has, he works better.
C. The fewer time my boss has, the better he works.
D. The less time my boss has, the better he works.

99. The film star wore dark glasses so that no one could recognize him.
A. The film star avoided recognizing by wearing dark glasses.
B. The film star avoided being recognized by wearing dark glasses.
C. The film star avoided to be recognized by wearing dark glasses.
D. The film star avoided being recognized to wear dark glasses.

100. While I strongly disapprove of your behavior, I will help you this time.
A. Despite my strong disapproval of your behavior, I won't help you this time.
A.

Despite my strong disapproval against your behavior, I will help you this time.
B. Despite my strong disapproval of your behavior, I will help you this time.
C. Despite of my strong disapproval of your behavior, I will help you this time.

English Objective Test 24


I. Choose the word whose underlined part is pronounced differently from the others in
each line.
1.

A. psychology

B. syndrome

C. hydrogen

D. typhoon

2.

A. efficient

B. concentrate

C. investigate

D. dedicate

3.

A. refusal

B. superb

C. humanity

D. consumption

4.

A. approach

B. throat

C. broad

D. raincoat

B. ensure

C. endure

D. mature

5. A. secure
II.

Choose the word with the position of the others in each line.
6. A. exposure

B. enterprise

C. adequate

D. horrified

7. A. diversity

B. publicity

C. remarkable

D. personally

8. A. procedure

B. distinguished

C. appropriate

D. telescope

9. A. transportation B. exaggeration

C. productivity D. population

10. A. president

C. suspicion

B. neighbourhood

D. temperate

Part B. LEXICO - GRAMMAR


I.

Choose the word or phrase which best completes each of the following sentences.

11.

When we arrived in Paris it was .......with rain.

A. running. B. dropping

C. pouring

D. falling

12. I should be very.......if you would accept the invitation to the party.

A. thankingB. honourable C. pleased

D. pleasing

13. I have always........that my university was the best in the country.

A. trusted...B. respected C. considered

D. regarded

14. Fier mother asked her to ......the table for the evening meal.

A. place. .B. put out C. serve

D. lay

15. His sister was full of............. for the way in which he had so quickly learned to drive a

car.
A. pride

B. admiration C. surprise

D. jealousy

16. The bus only stops here to . . .passengers.

A. get off

B. pick upC. alight

D. get on

17. No one was able to explain the of the old custom.

A. beginning

B. origin

C. starting point

D. reason

18. No She said that she could not a new dress on her small salary.

A. spend
19.

B. save

C. afford

D. spare

He tripped on the stairs and could not......... falling.

A. resist

B. prevent

C. stop

D. avoid

19. -The child was................ by a lorry on the safety crossing in the main street.

A. knocked

B. run across

C. run out

D. knocked down

20. Hotel rooms must be.................by 10 a.m., but luggage may be left with the porter.

A. vacated
19.

19.

D. left

B. overrun

C. oversee

D. overview

Our house still remains in after the cyclone.


A. good conditions

B. a good condition

C. the good condition

D. good state

This is the biggest exhibition held in Hanoi.


A. so far

20.

C. abandoned

The supervisor's job is to the work of his particular department.


A. overlook

20.

B. evacuated

B. never

C. up to now

D. ever

C. did he keep

D. he keeps

Not once his promises.


A. he is keeping

B. has he kept

II. Choose the underlined word or rase which would not be appropriate in standard
written English.
26. Automation reduces labour costs by cutting the number of workers needing to do a job.
A
B
C
D
27. Harvard College was the first institution of higher learning to be establish in the colonies.
A
B
C
D
28. Today tapping a suspect's phone is not longer allowed unless the investigator have a court
A
B
C
order to do so.
D
29. Randolph is the boy whom I think scored the winning points for the basketball team.
A
B
C
D
30. Each of the luxury cars in the showroom was quickly sold to their new owner.
A
B
C
D
31. If only man had been a bit less greedy, more bird and animal species might have been
A
B
C
D
avoided extinction.
32. Animals and man use the energy finding in food to operate their bodies and muscles.
A
B
C
D
33. The ability to convert raw materials into valuabler commodities is the basis of an
A
B
C
industrial economy and the foundation of a high standard of living.
D
29. Women have done advances in reaching high positions in business, but there are still
A
B
C
few women in government.
D
30. Lumber from redwoods is in great demand because of its straight grain, attractive colour
A
B
C
and durable.

D
III. Choose the correct form of the verb (A, B, r nr D) to complete each of the following
sentences.
1. By the end of last year he had read four Shakespeare plays and by next year he
(36) ......two more.
2. He (37) ......the part now if he (38) ......the producer at the last rehearsal.
3. I wonder why I (39) ...... trouble with the carburettor whenever I decide to go home
by car.
4. whenever he had an important decision to make, he <40) ...... a cigar, supposedly to
calm his nerves.
5. If he (41) ......my advice before, he (42) ......in danger now.
6. It was our fault to keep Mary waiting so long. We (43) ......her in advance.
7. From the day five years ago when they married until today, Torn (44) ...... on his wife to do
everything in the house, and she (45) ...... to give him the easiest home life a man could
possibly have
36.

A. have read

37.

A. would play

B. will have read

C. would be playing
38.

A. had not offended

C. had not been offended


39.

A. always have

C. have always had


40.

A. had lit

C. would be lighting
41.

A. have followed

C. followed
42.

A. would be not

43.

A. had informed

B. is not

C. must have informed


44.

A. relied

B. had relied

45.

A. never failed

C. had never failed


C. will read

D. will be reading

B. were playing
D. would have played

B. didn't offend
D. were not offended
B. am always having
D. have had always B. would have lit
D. would light
B. had followed
D. had been followed
C. would not be D. were not
B. informed
D. should have informed
C. has relied

D. relies

B. has never failed


D. was never fail

IV. Choose the appropriate preposition (A, B, C or D) to complete each of the


following sentences.
46. I rarely buy hardback book they arc so expensive. I always wait till they come

in paperback.
A. up

B. out

C. down

D. off

47. It isn't that woman's turn. Don't let her push..........

A. in

B. through

C. into

D. up

48. Simon fell ............... an Irish girl that he met on holiday. Three months later they

were married.
A. into
49.

I wish the boss would stop getting


A. to

50.

B. with
B. for

C. for

D. down

me. I haven't done anything wrong.


C. up

D. at

The company has just won a new order from the USA so they are going to take
extra workers.
A. on

51.

B. down

The names of the winners were given


A. up

53.

C. to

D. for

The school broke for the holidays at the end of July.


A. out

52.

B. up

B. in

C. up

D. into

on the radio.
D. Out

C. away

1 don't believe that story. I am sure you mak,. it ........


A. from

B. of

C. out

D. up

C. off

D. back

49. My plans for starting a restaurant fell

A. down
50.

B. through

Several children had to stay.from school because of the bus strike.

Part C. READING
I. Choose the correct word form (A, B, C or D) whioh best fits each 'pace the
following passage.
Poachers have struck again at the Mountain Gorilla, one of the world's (56) .......
animals. The gorillas-fewer than 250-live in scattered family groups throughout the
forest on the slopes of three extinct volcanoes. Each group is normally led by a
(57) .......male who may weigh up to 130 kilos and is easily recognized by the stripe of
silver fur which appears when he is fully mature. The poachers speared the male to
death as he tried to defend his family and then made off with a baby gorilla whose
mother is missing and also (58) .......dead.

With so few gorillas left, even one death is a serious blow, especially when a "silverback"
male is lost. With no dominant male to lead them, the rest of the group will probably
spilt up and join other groups. Twenty-five years ago there were perhaps 500 gorillas in
these high mountain forests. Today their numbers have been (59) ....... not just by
poaching but by human (60) .......in all its forms, illegal wood cutting, cattle-grazing and
farming

have

reduced the park to a small strip.


The Government is now trying to protect the gorillas to (61) ....... their survival. One of the
most successful operations has been to (62) ....... a group of wild gorillas to the presence of
tourists, who are (63) .......

to pay handsomely and to endure considerable (64).......

crawling through dense underground for a glimpse of these, our (65)....... living, relatives.
56. A. rarely

B. rarest

C. rareliest

D. rare

57. A. dominating

B. domination

C. dominant

D. dominantly

58. A. presuming

B. presumption

C. presumely

D. presumed

59. A. halved

B. half

C. halfed

D. halfing

60 A. interfere

B. interfering

C. interference

D. interferation

61. A. assure

B. ensure

C. reassure

D. insure

62. A. habit

B. habitate

C. habitual

D. habituate

63. A. preparing

B. prepared

C. preparable

D. preparedly

64. A. hard

B. hardness

C. hardship

D. hardwork

65. A. closest

B. closely

C. closing

D. close

IL Choose the word (A, B, C or D) which hest fits each space in the following passage.
Before 1855 Japan had been virtually (66).......Since 1638 the Japanese islands had been
sealed off from the (67) ....... of the world, although the Japanese leaders, through very limited
(68) .......with Dutch traders, were able to (69) ....... some information about what was going
(70) .......outside. The isolation of Japan for over two hundred years, with no Japanese (71)
....... to leave the country, and no foreigner allowed to land, is one of the most (72) .......
stories in the world history.
What happened since is an even more remarkable story. The impact of Japan on Europe and
America in the late nineteenth century was small compared with the impact of the west on
Japan. The Japanese understood that the (73) ....... of European military, political and
commercial power lay in their (74) ....... industrial technology; they learned what was
happening to China; they decided to learn as much as they could about Western (75) ....... and
organization and to apply that knowledge to (76) ....... their country into a modern, industrial

nation. The results have been (77) ....... Little more than a century after being opened up to the
rest of the world, Japan is the third largest industrial (78) ....... in the world, (79).......only by
the USA and the USSR, and has the highest living (80).......of any Asian nation.
66.

A. unknown

B. unheard

C. unseen

D. non-existence

67.

A. country

B. rest

C. nation

D. part

68.

A. link

B. connect

C. touch

D. contact

69.

A. store

B. reach

C. obtain

D. get

70.

A. there

B. on

C. at

D. to

71.

A. permitted

B. having

C. permitting

D. wanting

72.

A. attracting

B. interesting

C. remarkable

D. moving

73.

A. key

B. secret

C. lesson

D. experience

74.

A. lay

B. laid

C. placed

D. based

75.

A. latest

B. developed

C. good

D. advanced

76.

A. technique

B. technology

C. method

D. way

77.

A. transforming

B. turn

C. change

D. make

78.

A. proud

B. fast

C. amazing

D. good

79.

A. maker

B. producer

C. manufacture

D. producing

80.

A. surpassed

B. passed

C. overcome

D. crossed

III. Read the passage and then choose the answer for each of the following questions.
Compact discs (CDs) have revolutionized the music industry with their surprisingly realistic
sound. The six-inch discs look like thin, plastic sandwiches with aluminium in the centre,
They have digitally recorded material that is read by laser beams, so the sound has none of the
crackling of vinyl records, CDs are also virtually indestructible, lighter, and smaller than
conventional records (LPs). CDs are becoming more widely available than LPs; they're sold
in electronics and video stores that haven't formerly carried records or cassettes. Many record
stores are now cutting their prices on LPs to make room for the new CDs.
There has been a phenomenal growth in the sale of CDs. Sales were up almost 150 percent in
the first half of 1986 as compared to the first half of 1985. While fewer than 6 million CDs
were sold in the US in 1984, there were approximately 50 million sold in 1986. The sales
would likely be even higher were it not for the price: CDs cost nearly twice as much as LPs.
In the near future, however, prices should lower as more production facilities open.
81. The author refers to CDs as "sandwiches" because they...........
A. are light

B. are small

C. are layered

D. dont crackle

82. This passage states that it is difficult to......................


A. play a CD

B. produce a CD

C. record a CD

D. destroy a CD

83. Many record stores are currently......................


A. lowering CD prices
A. raising LP prices
B. lowering LP prices
C. raising CD prices

84. According to this passage, which one of the following is true?


A. Different kinds of stores are selling CDs.
A. More CDs are available than LPs.
B. Stores are selling more CDs than LPs.
B. Stores are losing money on their LPs.

85. The author's- main purpose is to .....................


A . tell how CDs are made
A. discuss the growth of CDs
B.

describe the technology that produces CDs

C.

compare CDs and LPs

Part D. WRITING
I.

Choose one sentence (A, B, C or D) which is closest in meaning to the original

sentence.
86. She acted brilliantly in the school play.
A. She gave a brilliant performance in the school play.
A. She acted the school play brilliantly.
B. She showed the play at school brilliantly.
B. She had a brilliant act in the school play.

87. They were married in the summer of 1999.


A. They got married with each other in the summer of 1999.
B. Their wedding held in the summer of 1999.
C. Their wedding took place in the summer of 1999.
D. Their wedding was celebrated in the summer of 1999.

88. She is proud of being such a good pianist.


A. She prides herself on playing the piano.
B. She prides herself on being a good pianist.
C. She prides herself on being good pianist.
D. She prides herself in her playing the piano.

89. It is certain that the new cuts will worry the staff
A. The new cuts certainly worry the staff.
B. The new cuts will be certainly worry the staff.
C. The new cuts will be bound to worry the staff.
D. The new cuts are bound to worry the staff.

90. I felt that it had been a mistake to write the letter.


A. I regretted ever having written the letter.
B. I felt that I was mistaken to write the letter.
C. I regretted to have written the letter.
D. I felt that it was my fault to write the letter.

91. I wasn't in the office yesterday, .so you must have spoken to my assistant.
A. I wasn't in the office yesterday, so it was my assistant that you spoke to.
B. i wasn't in the office yesterday, so I couldn't speak to you.
C. I wasn't in the office yesterday, so it must have been my assistant a you spoke

to.
D. I wasn't in the office yesterday, so I couldn't have spoken to you.

92. His last letter to me was written three years ago.


A. I haven't heard from him for three years.
B. I last heard of him three years ago.
C. He wrote last to me three years ago.
D. He last wrote to me since three years.

93. We'd better leave them a note, because it's possible they'll arrive later.
A.

We'd better leave them a note as they possibly arrive later.

B.

They'll probably arrive later so that we'd better leave them a note.

C.

We'd better leave them a note in case they arrive later.

D.

If they arrive late, we'd better leave them a note.

94. He was unsuccessful in reaching his goal.


A. He tried in vain to reach his goal.
B. He tried best in reaching his goal but in vain.
C. However he did his best to reach his goal, he was unsuccessful.
D. He was impossible to reach his goal.
95. Thanks to my uncle's support I was able to finish the project.
A. If there weren't my uncle's support, I wasn't able to finish the project.
B. Had it not been for my uncle's support, I wouldn't have been able to finish the
project.

C. Unless my uncle had helped me, I would have been able to finish the project.
D. If it were not for my uncle's support, I wouldn't be able to finish the project.
IL. Choose the sentence (A, B. C or D that is best writtAn based on the words provided.
96. Browns / leave / London / New York / by / end / this MOM h.
A. The Browns will have left London for New York by the end of this month.
B. The Browns will have left London to New York by the end of this month.
C. The Browns will leave London for New York by end of this month.
D. The Browns will have left to London and New York by the end of this month.
97. result / investigation / publish / newspaper / possible.
A. The result of the investigation will publish in newspapers as soon as possible.
B. The result from investigation will be published in newspaper as soon as possible.
C. The result of the investigation will be published in newspaper as soon as possible.
D.The result of the investigation will have been published in newspaper as soon as
possible.
98. I / advise / not / spend / most / time / away / home. .
A. I advise you not spend most of the time away from home.
B..1 would advise you not to spend most of your time away from home.
C. I would advise you not to spend most of time away from home.
D. I advise you not to spend most of your time away from your home.

99.

The country / have / advantage / peace / quiet / but / suffer / disadvantage / cut off.
A. The country has the advantage of peace and quiet but it suffers from the

disadvantage of cutting off.


B. The country has the advantage of peace and quiet but it suffers the disadvantage of
being cut off.
C. The country has the advantage of peace and quiet but it suffers from the
disadvantage of having cut off.
D. The country has the advantage of peace and quiet but it suffers from the
disadvantage of being cut off.
100. She / hurt / learn / close friend /forget / birthday.
A. She was hurt to learn that her close friend had forgotten her birthday.
B. She was hurted to learn that her close friend had forgotten her birthday.
C. She was hurt to learn about her close friend forget her birthday.
D. She was hurt to learn that her closed friend had forgotten her birthday.

26

Anda mungkin juga menyukai